Overseas Class-8 Mental Ability

Download as pdf or txt
Download as pdf or txt
You are on page 1of 192

MENTAL ABILITY INDEX

CLASS-VIII
S.No. CHAPTER NAME Pg.No.
1. CODING & DECODING 01-12

2. DIRECTION SENSE TEST 13-18

3. PAPER FOLDING & PAPER CUTTING 19-22

4. SERIES 23-30

5. NON-VERBAL SERIES 31-34

6. ANALOGY 35-42

7. CLASSIFICATION 43-46

8. NON-VERBAL ANALOGY & 47-54


NON-VERBAL CLASSIFICATION

9. LOGICAL VENN DIAGRAM 55-64

10. BLOOD RELATION 65-70

11. ANALYTICAL REASONING 71-76

www.allenoverseas.com OVERSEAS i
S.No. CHAPTER NAME Pg.No.
12. NUMBER, RANKING & TIME SEQUENCE TEST 77-86

13. ALPHABET TEST 87-92

14. DICE & CUBE 93-112

15. INSERTING THE MISSING CHARACTER 113-124

16. MIRROR & WATER IMAGES 125-138

17. ARITHMETICAL REASONING 139-142

18. ALPHA-NUMERIC SEQUENCE PUZZLE, 143-150


& LOGICAL SEQUENCE OF WORDS

19. FIGURE MATRIX, FIGURE FORMATION, 151-160


CONSTRUCTION OF SQUARES &
GROUPING OF IDENTICAL FIGURES
20. MATHEMATICAL OPERATIONS 161-166

21. SPOTTING OUT THE EMBEDDED FIGURE 167-174

22. DOT SITUATION 175-180

23. COMPLETION OF INCOMPLETE PATTERN 181-184

24. SITTING ARRANGEMENT 185-190

Dear students, we have put in best efforts for making this module highly interesting, easy to understand
BOBB-BC

and free from errors. We welcome your valuable feedback, suggestions or any corrections to further
improve the quality of this module. Kindly mail them to us on [email protected].

ii OVERSEAS www.allenoverseas.com
Mental Ability

CHAPTER 1 CODING & DECODING

A code is a 'system of signals'. This means a coding is a method of transmitting a message between the sender
and the receiver without a third person knowing it.
The coding and decoding is set up to decipher the rule that codes a particular word/message and break the
code to decipher the message.

1.1 Number/symbol coding


In these questions, either numerical code values are assigned to a word or alphabetical code letters are assigned
to the numbers. The candidate is required to analyse the code as per the questions.
The letters and numbers would be correlated to each other in either of the following ways :
(1) Direct coding.
(2) In relation to the position of letters in English alphabet.

1. Direct coding

Solved examples
Ex.1 In a certain code, 1 is coded as A, 2 as B, 3 as E, 4 as L and 5 as T. How is 51243 coded in that letter?
(1) TABEL (2) TABLE
(3) ABLTE (4) TBALE

Sol. Number 1 2 3 4 5

Code A B E L T

So Number 5 1 2 4 3

Code T A B L E

Hence, the answer is (2).


Ex.2 In a certain code, if MALE is coded as 5432, TUBE as 6912, how is MUTE coded in that code?
(1) 5962 (2) 6412
(3) 5692 (4) 6142

Number M A L E T U B E
Sol.
Code 5 4 3 2 6 9 1 2

So, MUTE is coded as 5962

Hence, the answer is (1).

www.allenoverseas.com OVERSEAS 1
Class-VIII

2. In relation to the position of letters in english alphabet

There are 26 letters in the english alphabet. If it is asked to count from the left, then it is begun to count from
A and if it is asked to count from the right then it is begun from Z.

Now to remember the number of positions of letters in the alphabet from the left as well as right, we have the
formula EJOTY and BGLQV
E J O T Y

5 10 15 20 25
It means E in the alphabet from the left is at 5th place, J at 10th place, O at 15th place, T at 20th and Y at 25th place.

Another formula is
B G L Q V

25 20 15 10 5
to remember the number of position of a letter in the alphabet from right i.e. from Z to A.

Solved examples

Ex.3 If MAN is coded as 28, what will be the code number for CHILD?

(1) 25 (2) 49 (3) 36 (4) 64

Sol. i.e. M ® 13

A® 1

N ® 14

On adding all, 13 + 1 + 14 =28

Therefore

C ® 3

H ® 8

I ® 9

L ® 12

D® 4

On adding all,

3 + 8 + 9 + 12 + 4

= 36

Hence answer is (3)

2 OVERSEAS www.allenoverseas.com
Mental Ability

Ex.4 If ZIP = 30, ZAP = 38, then VIP will be equal to –

(1) 34 (2) 43 (3) 53 (4) 64

Sol. Using the pattern,


B G L Q V

25 20 15 10 5
The word ZIP can be

Coded as

Z ® 1

I ® 18

P ® 11

So 1 + 18 + 11 = 30

and ZAP as

Z ® 1

A ® 26

P ® 11

So 1 + 26 + 11 = 38

Similarly, VIP can be coded as

V ® 5

I ® 18

P ® 11

So 5 + 18 + 11 = 34

Hence, the answer is (1).

www.allenoverseas.com OVERSEAS 3
Class-VIII

1. If 43572 stands for WORLD, then 7352 stands for –


(1) DOLR (2) ROLD (3) LORD (4) OLRD
2. If GROWTH stands for 579312, then THROW stands for –
(1) 12793 (2) 21793 (3) 21973 (4) 21379
3. If SLIGHT stands for 426875, then GIST stands for –
(1) 6845 (2) 8645 (3) 4568 (4) 4586
4. If PLOT = 5321, TAKE = 1790, PINK = 5469, What is PLATE?
(1) 35710 (2) 53710 (3) 53701 (4) 53071
5. If BREAD = 72416, AIRY = 1392, YIELD = 23456, What is 6941652?
(1) DREADLY (2) DRAEDLY (3) DREAMLY (4) REDAMLY
6. If PRATAP could be given the code number 1618120116, what code number can be given to NAVIN?
(1) 14122914 (2) 19274651 (3) 24639125 (4) 73957614
7. In a certain code language 'CAT' is written as 3120 and 'DOG' is written as 4157. Then which of the following
is certainly the decoded form of 25144?
(1) BEND (2) BEADD (3) YADD or YND (4) All of the above
8. In a certain code language 'SUN' is written as '54' then 'MOON' is written as
(1) 50 (2) 53 (3) 57 (4) 159
9. If REASON is coded as 5, BELIEVED is coded as 7, then GOVERNMENT will be coded as
(1) 8 (2) 9 (3) 10 (4) 11
10. If Z = 52 and ACT = 48, then BAT will be equal to –
(1) 39 (2) 41 (3) 44 (4) 46
Directions (Q.11 & Q.12): The letter-group in each of these questions is to be codified as per the following
number codes:
Letters L T P M R D H F K V
Number codes 3 0 4 9 2 1 5 7 6 8

11. DHFMTRV
(1) 1579082 (2) 1579028 (3) 1570982 (4) 1570928
12. KHVLRDP
(1) 6853214 (2) 6853124 (3) 6583214 (4) 6538214
13. If ACNE is coded as 3, 7, 29, 11 then BOIL will be coded as
(1) 5, 31, 21, 25 (2) 5, 31, 19, 25 (3) 5, 29, 19, 25 (4) 5, 29, 19, 27
14. If ACT = 8 and LANE = 8 then how will SNAKE code :
(1) 8 (2) 9 (3) 10 (4) 11
15. If 17 • 18 = 306 and 14 • 23 = 330, then 10 • 12 =
(1) 121 (2) 306 (3) 345 (4) 129

Que. 1 2 3 4 5 6 7 8 9 10 11 12 13 14 15
Ans. 3 1 2 2 1 1 4 3 2 4 2 3 2 3 1

4 OVERSEAS www.allenoverseas.com
Mental Ability

1.2 Letter coding


In these questions, the letter in a word are replaced by certain other letters according to a specific rule to form
its code. You are required to detect the coding pattern/rule and answer the questions accordingly.
The letters and codes would be related to each other in either of the following ways :–
(1) Direct coding.
(2) Movement of letters relative to their position in English alphabet.
(3) Reordering of letters.

1. Direct coding
In direct coding particular letters are made codes for particular letters without there being any set pattern. In
direct coding, the code letters occur in the same sequence as the corresponding letters occur in the words.
In questions on direct coding, either the particular codes of letters are given or the codes of two or more words
are given and one is asked to find the codes of given words involving only those letters for which the codes have
already been mentioned.
Solved examples
Ex.1 If in a certain code, O is written as E, A as C, M as I, S as O, N as P, E as M, I as A, P as N and C as S, then how
will COMPANIES be written in that code?
(1) SMINCPAMO (2) SEIACPAMO (3) SEINCPAMO (4) SEINCPMIO
Sol. C O M P A N I E S

S E I N C P A M O
Hence, the answer is (3).
Ex.2 If the word VOTES be written as FNLPK in coded form, how can be STOVE be written in that code?
(1) NFPKL (2) KPLNF (3) NLKPF (4) KLNFP
Sol. We have,
Letter V O T E S

Code F N L P K
So the code for STOVE becomes KLNFP
Hence, the answer is (4).

2. Movement of letters
In these questions, coding/decoding is done by moving the letters backward/forward relative to their positions in
English alphabet.
Solved examples
Ex.3 If TOP is coded as SNO then how is FREEZE coded?
(1) EQDFYG (2) ESDFYF (3) GQFDYE (4) EQDDYD
Sol. T O P
–1 –1 –1
S N O
Clearly each letter in the word is moved one step backward to obtain the corresponding letter of code.
Thus, FREEZE will be coded as
F R E E Z E
–1 –1 –1 –1 –1 –1
E Q D D Y D
Hence, the answer is (4).

www.allenoverseas.com OVERSEAS 5
Class-VIII

Ex.4 In a certain code, 'MONKEY' is written in as XDJMNL, how is 'TIGER' written in that code?
(1) SHFDQ (2) QDFHS (3) SDFHS (4) QDHJS

–1
Sol. –1

MONKEY X D J M N L
–1
–1
–1
–1

i.e. the letter of word are first written in reverse order & then moved one step backward.
So, TIGER becomes REGIT and then
R E G I T
–1 –1 –1 –1 –1
Q D F H S
Hence, the answer is (2).

3. Reordering of letters
In these questions, the alphabets used in the letter and code are same, but reordered using a fixed pattern.
Solved examples
Ex.5 In a code COURAGE is denoted by UOCREGA then JOURNAL is denoted in the same code by –
(1) UOJRLAN (2) OUJRANL (3) OJURANL (4) UOJLANR

Sol. C O U R A G E U O C R E G A

1 2 3 4 5 6 7 3 2 1 4 7 6 5
Similarly, JOURNAL can be coded as
1 2 3 4 5 6 7 3 2 1 4 7 6 5

J O U R N A L U O J R L A N
Hence, the answer is (1).

6 OVERSEAS www.allenoverseas.com
Mental Ability

1. In a certain code "GOODNESS' is coded as HNPCODTR, how is GREATNESS' written in that code?
(1) HQFZUODTR (2) HQFZUMFRT (3) HQFZSMFRT (4) FSDBSQDTR
2. In a certain code, 'COURSE' is written as 'FRXUVH', then how would RACE be written in that code?
(1) HFDV (2) UCFH (3) UDFH (4) UDHF
3. In a certain code 'DIVISION' is written as DVISIOIN'. How is STATES written in that code?
(1) SATETS (2) STAETS (3) SATTES (4) SAETTS
4. In a certain code language 'TELEPHONE', is written as 'ENOHPELET'. How is 'ALIGATOR' written in that code?
(1) ROTAGILE (2) ROTAGILA (3) ROTEGILA (4) ROTAGIAL
5. In a certain code, 'SYSTEM' is written as 'SYSMET'. How is FRACTION' written in that code?
(1) CARFNOIT (2) CARFTION (3) NOITFRAC (4) FRACNOIT
6. In a certain code 'PROSE' is written as 'PPOQE'. How is 'LIGHT' written in that code?
(1) LIGFT (2) LLGFE (3) JIEHR (4) LGGFT
7. In a certain code, 'FORGET' is written as DPPHCU. How would 'DOCTOR' be written in that code?
(1) BPAUMS (2) BPAUPS (3) EMDRPP (4) BPARPP
8. In a certain code, 'EASE' is written as 'GUCG'. How is 'CUT' be written in that code?
(1) UVD (2) DVU (3) VWE (4) EWU
9. In a certain code, 'CERTAIN' is coded as 'XVIGZRM'. How is 'MUNDANE' coded in that code?
(1) NFMWZMV (2) VMZWMEN (3) NFMWZMX (4) NFMXZMV
10. If JAPAN is coded as KCSES, then the code for CASTLE will be
(1) DCIJOB (2) DCJKRO (3) DCKMSG (4) DCVXQK
11. If in any code language NATIONAL is written as MZGRLMZO than how is JAIPUR written in that language.
(1) QZRKFI (2) PZRKFI (3) QZRIFK (4) QARKFI
12. In a certain code, GIGANTIC is written as GIGTANCI. How is MIRACLES written in that code?
(1) MIRLCAES (2) MIRLACSE (3) RIMLACSE (4) RIMLCAES
13. In a certain code SPORADIC is written as QNORDJEB. How is TROUBLES written in that code?
(1) SQTNTFMC (2) TNQSRDKA (3) TNQSTFMC (4) TFQSCMFT
14. If TRIANGLE is code as SSHBMHKF, then SQUARE would be.
(1) RRIASF (2) RPVBSF (3) RRTBQF (4) RPVBSD
15. If NEW DELHI is coded as HGKDCVDM then how will be HARYANA coded in that language?
(1) ZMZXQZG (2) ZMZXQZF (3) ZNZYQZF (4) ZNZYQZH

Q u e. 1 2 3 4 5 6 7 8 9 10 11 12 13 14 15
Ans. 2 3 1 2 1 4 1 3 1 4 1 3 3 3 1

www.allenoverseas.com OVERSEAS 7
Class-VIII

1.3 Substitution
In these questions, some particular words are assigned certain substituted names. Then a question is asked that
is to be answered in the substituted code language.

Solved examples

Ex.1 If 'Blue' is called 'Black', 'Black' is called 'Red', 'Red' is called 'Pink', 'Pink' is called 'Orange' and 'Orange' is called
'Brown', then what is colour of human blood?

(1) Brown (2) Pink (3) Black (4) Red

Sol. Colour of human blood is Red and Red and called Pink.

Hence, the answer is (2).

Ex.2 A 'train' means 'bus', 'bus' means 'tractor', tractor' means 'car', 'car' means 'scooter' 'scooter' means 'bicycle',
'bicycle' means 'moped', which is used to plough a 'field'?

(1) Train (2) Bus (3) Tractor (4) Car

Sol. A tractor is used to plough the field. But bus means tractor. So bus will be used to plough the field.

Hence, the answer is (2).

1. If for 'Cloud' is said 'Rain', for 'Rain' is said 'Tree', for 'Tree' is said 'Axe', for 'Axe' is said 'House' and for 'House' is
said 'Mason', then from which of the following wood can be obtained?

(1) Tree (2) Rain (3) Axe (4) House

2. If air is called green, green is called blue, blue is called sky, sky is called yellow, yellow is called water and water
is called pink, then what is the colour of clean sky?

(1) Pink (2) Sky (3) Water (4) Blue

3. If orange is called ghee, ghee is called soap, soap is called ink, ink is called honey and honey is called orange
then which one of the following is used for cloth washing?

(1) Honey (2) Ghee (3) Orange (4) None of these

4. If dog is called cat, cat is called lion, lion is called ox, ox is called cock, cock is called elephant and elephant is
called donkey then tell a farmer ploughs with which animal?

(1) Dog (2) Lion (3) Donkey (4) Cock

5. If student is called saint, saint is called thief, thief is called politician, politician is called duffer, duffer is called
head, then tell who does the job of robbery during day & night?

(1) Saint (2) Thief (3) Head (4) Politician

8 OVERSEAS www.allenoverseas.com
Mental Ability

6. If air means green, green means blue, blue means sky, sky means yellow, yellow means water and water
means pink, then what is the color of clear sky?

(1) Green (2) Sky (3) Yellow (4) Water

7. If sand means air, air means plateau, plateau means well, well means island and island means sky, then
from where will a woman draw water?
(1) Well (2) Island (3) Sky (4) Plateau

8. If 'yellow' means 'green', 'green' means 'white', 'white' mean 'red', 'red' means 'black', black' means 'blue', 'blue'
means violet', which of the following represent the colour of milk?
(1) Black (2) Violet (3) Green (4) Blue
9. In a certain code language, if pen means eraser, eraser means Book, Book means scale, scale means sharpner
sharpner means duster and Duster means table, then what is the name of the object that is used to clean
the black board in that language?

(1) Duster (2) Sharpner (3) Scale (4) Table


10. If cushion means pillow, Pillow means mat, mat means bedsheet and bedsheet means cover, which will be
spread on the floor?

(1) Cover (2) Bedsheet (3) Mat (4) Pillow

Que. 1 2 3 4 5 6 7 8 9 10
Ans. 3 2 4 4 4 1 4 3 2 4

www.allenoverseas.com OVERSEAS 9
Class-VIII

1.4 Deciphering message codes


In these type of questions, some messages are given in the coded language and the code for a particular word
or message is asked. To analyse such codes, any two messages bearing a common word are picked up. The
common code-word will thus represent that word. Proceeding similarly by picking up all the possible combina
ations of two, the entire message can be decoded and the codes for individual words found.
Solved examples

Ex-1. Read the information given below, and answer the questions that follow.

In a certain code " rem tur dimi wee" means " my pencil was stolen", "soz rem legh ner" means "your pencil is
here", "rem zet tur legh" means " blue pencil was here" and "dimi rem tur soz" means "your pencil was stolen."

i. Which of the following is the code for the word "here"?

(1) tur (2) ner (3) dimi (4) legh

Sol. 1. "rem tur dimi wee" Þ "my pencil was stolen".

2. "soz rem legh ner" Þ "your pencil is here".

3. "rem zet tur legh" Þ "blue pencil was here".

4. " dimi rem tur soz: Þ "your pencil was stolen".

Thus from (2) & (3), we get " rem legh" Þ "pencil here" (though not necessarily in the same order). Further, from
(1), (2), (3) & (4) together, we get " rem" = " pencil". Therefore, we get, " legh" = " here". Hence, the answer is (4).

ii. Which of the following indicates "blue pencil is stolen"?

(1) Zet rem ner dimi (2) Ner legh tur zut

(3) Dimi rem soz ner (4) Zet legh tur rem

Sol. We have, "rem" = " pencil". Now from (3) & (4),

we get "tur" = "was" and

since "legh" = "here", we get, "zet" = "blue".

"soz" = "your". Hence, from (2), we get, "ner" = "is".

Thus, "zet rem ner dimi" = blue pencil is stolen".

Hence, the answer is (1).

10 OVERSEAS www.allenoverseas.com
Mental Ability

1. In a code 'Ni Muk Pik' means 'Serious and Worried', 'In Dik So' means 'Each Any Other' and 'Tur Muk To', means
'Soul and Body' then in that code what is for 'Each Worried'?
(1) In Ni (2) Pik Ni (3) Dik Pik (4) Data inadequate
2. In a certain code language 'Kol Tip Mot' means 'Song Is Admirable', 'Mot Baij Min' means 'Dancing Is Good and
'Tip Nop Baij means 'Song And Dancing', then which of the following stands for 'Good'?
(1) Mot (2) Min (3) Baij (4) Data inadequate
Direction (Q.3 to Q.5) : In a code
(i) 1, 5, 9 means 'You better go' , (ii) 1, 6, 7 means 'better come here',
(iii) 5, 6, 7 means, 'you come here', (iv)1, 5, 6 means, 'better you here',
(v) 3, 7, 9 means, 'come and go'
3. To find the code for 'better' atleast which group is necessary?
(1) (i) and (ii) (2) (iv) and (v) (3) (i) and (iv) (4) (ii) and (v)
4. Which of the following is used for 'and' in the code?
(1) 6 (2) 9 (3) 3 (4) 7
5. Which of the following is used for 'go' in the code?
(1) 1 (2) 5 (3) 7 (4) None of these
Direction (Q.6 & Q.7) : In a certain code, "ze lo ka gi" is a code for "must save some money", "fe ka so ni" is
a code for "he made good money", "ni lo da so" is a code for "he must be good" and "we so ze da", is a code for
"be good save grace".
6. Which of the following is the code for "Must"?
(1) so (2) da (3) lo (4) ni
7. What does the code "ze" stand for?
(1) some (2) save (3) be (4) grace
Direction (Q.8 to Q. 10) : In a code
(i) 'Rip Lub Ja Pit' means 'Kindly Let Me Speak' (ii) 'Sa Tik Lub' means 'Kindly Go Forward'
(iii) 'Pit Sun Ki' means 'Speak With Example' (iv) 'Ja Ha Tik' means 'Let Others Go'.
8. In the code what is for 'Example'?
(1) Ki (2) Pit (3) Sun (4) Data inadequate
9. In the word what is for 'Ha'?
(1) Forward (2) Kindly (3) Go (4) Others
10. To find the code for 'Me' which of the following statements are necessary?
(1) Only (i) and (ii) (2) Only (i) and (iii) (3) Only (ii), (iii) and (iv) (4) All are necessary.

Qu e. 1 2 3 4 5 6 7 8 9 10
An s . 4 2 1 3 4 3 2 4 4 4

www.allenoverseas.com OVERSEAS 11
Class-VIII

Important Notes

12 OVERSEAS www.allenoverseas.com
Mental Ability

CHAPTER 2 DIRECTION SENSE TEST


These questions are designed to test candidate's ability to sense direction. Questions on direction are simpler
than other questions if student possesses the right knowledge of the direction. Confusion is created in the
question by giving frequent right and left turns to a specific direction. Students are, therefore, advised to use the
diagram as given in the fig. for the purpose of sensing direction.
North

N-W N-E
90°
45°
West East

S-W S-E

South
Solved examples
Ex.1 A man is facing west. He turns 45° in the clockwise direction and then another 180° in the same direction and
then 270° in the anti-clockwise direction. Which direction is he facing now?
(1) South (2) North-west (3) West (4) South-west
B 270°

Sol.
45° 180°
A
O

C
D
Clearly, the man initially face in the direction OA. On moving 45° clockwise, he faces in the direction OB. On
further moving 180° clockwise, he faces in the direction OC. Finally on moving 270° anti-clockwise, he faces in
the direction OD, which is South-west. Hence, the answer is (4).
Ex.2 Deepa moved a distance of 75 metres towards the north. She then turned to the left and walking for about 25
metres, turned left again and walked 80 metres. Finally, she turned to the right at an angle of 45°. In which
direction was she moving finally?
(1) North-East (2) North-West (3) South-East (4) South-West
25m
C B
Sol.

80m 75m

A
D

E 45°
Deepa started from A, moved 75 m upto B, turned left and walked 25 m upto C. She then turned left again and
moved 80 m upto D. Turning to the right at an angle of 45°, she was finally moving in the direction DE i.e.
South-west. Hence the answer is (4).

www.allenoverseas.com OVERSEAS 13
Class-VIII

Directions (Ex.3 & Ex.4) : Study the information and answer the questions given below:

On a playing ground Dev, Kumar, Nilesh, Ankur and Pintu are standing as directed below facing the North:

(i) Kumar is 40 m to the right of Ankur. (ii) Dev is 60 m to the South of Kumar.

(iii) Nilesh is 25 m to the West of Ankur. (iv) Pintu is 90 m to the North of Dev.

Ex.3 Who is the North-East of the person, who is to the left of Kumar?

(1) Nilesh (2) Ankur (3) Dev (4) Pintu


Pintu
Sol. 30m

25m 40m N

Nilesh Ankur Kumar


W E

S
60m

Dev
From fig. it is very clear that the person to the left of Kumar is Ankur and Pintu is to the North-East of Ankur.
Therefore, Pintu is our answer. Hence, the answer is (4).

Ex.4 If a boy walks from Nilesh, meets Ankur followed by Kumar, Dev and then Pintu, how many metres has he
walked if he has travelled the straight distance all through?

(1) 215 m (2) 155 m (3) 245 m (4) 185 m

Sol. Following the instructions as given in the question, the total distance covered by the boy

= 25 + 40 + 60 + 90 = 215 m

Hence, the answer is (1).

14 OVERSEAS www.allenoverseas.com
Mental Ability

1. Anoop starts walking towards South. After walking 15 m he turns towards North. After walking 20 m, he turns
towards East and walks 10m. He then turns towards South and walks 5 m. How far is he from his original
position and in which direction?
(1) 10 m, North (2) 10 m, South (3) 10 m, West (4) 10 m, East
2. From a point, Rajneesh started walking East and walked 35 m. He then turned on his right and walked 20 m
and he again turned right and walked 35 m. Finally, he turned his left and walked 20 m and reached his
destination. Now, how far is he from the starting point?
(1)50m (2) 55m (3) 20m (4) 40 m
3. A rat runs 20 m towards East and turns to right, runs 10 m and turns to right run 9 m and again turns to left, runs
5 m and then turns to left, runs 12 m and finally turns to left and runs 6 m. Now, which direction is the rat facing?
(1) East (2) North (3) West (4) South
4. A man goes towards East 5 km, then he takes a turn to South-West and goes 5 km. He again takes a turn
towards North-West and goes 5 km with respect to the point from where he started, where is he now?
(1) At the starting point (2) In the West (3) In the East (4) In the North-East
5. Vijayan started walking towards South. After walking 15 m, he turned to the left and walked 15 m. He again
turned to his left and walked 15 m. How far is he from his original position and in which direction?
(1) 15 m, North (2) 15 m, South (3) 30 m, East (4) None of these
6. Rakesh is standing at a point. He walks 20 m towards the East and further 10 m towards the South, then he
walks 35 m towards the West and further 5 m towards the North, then he walks 15 m towards the East. What
is the straight distance in metres between his starting point and the point where he reached last?
(1) 0 (2) 5 (3) 10 (4) Can't be determined
7. Rahim started from point X and walked straight 5 km west, then turned left and walked straight 2 km and again
turned left and walked straight 7 km. In which direction is he from the point X?
(1) North-East (2) South-West (3) South-East (4) North-West
8. Ravi travelled 4 km straight towards South. He turned left and travelled 6 km straight, then turned right and
travelled 4 km straight. How far is he from the starting point?
(1) 8 km (2) 10 km (3) 12 km (4) None of these
9. Mohan started from point 'A' and proceeded 7 km straight towards East, then he turned left and proceeded
straight for a distance of 10 km. He then turned left again and proceeded straight for a distance of 6 km, and then
turned left again and proceeded straight for another 10 km. In which direction is Mohan from his starting point?
(1) East (2) West (3) North (4) South
10. From point P, Akshay starts walking towards East. After walking 30 m, he turns to his right and walks 10 m. He
then turns to his right and walks for 30 m. He again turns to his right and walks 30 m. How far is he from point
P and in which direction?
(1) Point P itself (2) 10 m, North (3) 20 m, West (4) 20 m, North

www.allenoverseas.com OVERSEAS 15
Class-VIII

11. If P and S interchanges their position & Q and T interchanges their position, then which point is north-west of
R?

P Q

S T

(1) P (2) Q (3) T (4) S


12. Starting from a point, a person walked 12 m North, he turned right and walked 10m, he again turned right and
walked 12 m, then he turned left and walked 5 m. How far is he now and in which direction from the starting
point?
(1) 10 m, towards West (2) 15 m, towards East
(3) 10 m, towards East (4) 5 m, towards West
13. Starting from a point 'S', Mahesh walked 25 m towards South. He turned to his left and walked 50 m. He then
again turned to his left and walked 25 m. He again turned to his left and walked 60 m and reached a point T.
How far Mahesh is from Point 'S' and in which direction?

(1) 10 m, West (2) 25 m, North (3) 10 m, East (4) 25 m, West

14. A man starts from his house and walks 10 km in South direction, then he turns right and goes 6 km, again he
turns right and goes 10 km and finally turns right and goes 6 km. At what distance is he from the starting point
and in which direction?

(1) 2 km, North (2) 3 km, South (3) At the starting point (4) 4 km, East

15. 'A' walks 10 m towards East and then 10 m to his right. Then every time turning to his left, he walks 5, 15 and
15 m, respectively. How far is he now from his starting point?

(1) 5 m (2) 10 m (3) 15 m (4) 20 m

16. Village Chimur is 20 km to the North of village Rewa. Village Rahate is 18 km to the East of village Rewa.
Village Angne is 12 km to the West of Chimur. If Sanjay starts from village Rahate and goes to village Angne,
in which direction is he from his starting point?

(1) North (2) North-West (3) South (4) South-East

17. A started from a place. After walking for 1 km, he turns to the left, then walking for 1/2 km, he again turns
to left. Now, he is going Eastward direction. In which direction, did he originally start?

(1) West (2) East (3) South (4) North

18. A is to the South-East of C, B is to the East of C and North-East of A. If D is to the North of A and North-West
of B, in which direction of C is D located?

(1) North-West (2) South-West (3) North-East (4) South-East

16 OVERSEAS www.allenoverseas.com
Mental Ability

19. Ms. A goes for her morning walk at 6 O'clock towards sun for 2 km, then she turns to her right and walks 3 km.
She again turns to her left and walks 2 km, finally she turns to her left to walk another 6 km. In which direction
is she moving and at what distance from the last turn, she is standing?

(1) 6 km, East (2) 9 km, East (3) 5 km, North-East (4) 6 km, North

20. A watch reads 4.30. If the minute hand points East, in what direction will the hour hand point?

(1) South-East (2) North-East (3) North (4) North-West

21. Revati goes towards South-East a distance of 7 km, then she goes towards west a distance of 14 km. From here
she goes towards North-West a distance of 7 km and finally she goes a distance of 4 km towards east. How far
is she now from the starting point?

(1) 3 km (2) 4 km (3) 10 km (4) 11 km

22. Pravin walked 30 metres towards East, took a right turn and walked 20 metres, again took a right turn and
walked 30 metres. How far was he from the starting point?

(1) 30 metres (2) 80 metres (3) 50 metres (4) 20 metres

23. Ankit started walking towards North. After walking 30 m, he turned towards left and walked 40 m. He then
turned left and walked 30 m. He again turned left and walked 50 m. How far is he from his original position?

(1) 50 m (2) 40 m (3) 20 m (4) 10 m

24. A man is facing North-West. He turns 90° in clockwise direction and then 135° in the anticlockwise direction.
Which direction is he facing now?

(1) East (2) West (3) North (4) South

25. From his house, Vishal went 15 km to the North. Then he turned west and covered 10 km. Then, he turned South
and coverd 5 km. Finally, turning to East, he covered 10 km. How far and in which direction is he from his house?

(1) 10 km East (2) 10 km West (3) 10 km North (4) 10 km South

26. If A # B means B is at 1 metre to the right of A; A $ B means B is at 1 metre to the north of A, A B means
B is at 1 metre to the left of A; A Î B means B is at 1 metre to the south of A and first person from the left is
facing North, then according to P # R $ A U, in which direction is U with respect to P?

(1) East (2) West (3) North (4) South

27. A tourist drives 10 km towards east and turns to the right hand and drives 3 km. Then he drives towards west
(turning to his right) 3 km. He then turns to his left and drives 2 km. Finally he turns to his right and travels 7 km.
How far is he from his starting point and in which direction would he be?

(1) 10 km, East (2) 5 km, South (3) 8 km, West (4) 5 km, West

28. It is 3'o' clock in a watch .If the minute hand points towards the north -East ,then the hour hand will point towards
the ............... .

(1) South (2) South-West (3) North-West (4) South-East

www.allenoverseas.com OVERSEAS 17
Class-VIII

29. Vijay walks 10 km towards North. From there, he walks 6 km towards South. The he walks 3 km towards east.
How far and in which direction is he now with reference to starting point?

(1) 5km West (2) 5 km North-East (3) 7 km East (4) 7 km West

30. A direction pole is situated on the crossing. Due to an accident the pole turned in such a manner that the
pointer which was showing East, started showing south. One traveller went to the wrong direction thinking it to
West. In what direction actually he was travelling?

(1) East (2)West (3)North (4) South

Que. 1 2 3 4 5 6 7 8 9 10
Ans. 4 4 2 1 4 2 3 2 1 4
Que. 11 12 13 14 15 16 17 18 19 20
Ans. 4 2 1 3 1 2 1 3 3 2
Que. 21 22 23 24 25 26 27 28 29 30
Ans. 3 4 4 2 3 3 2 4 2 3

18 OVERSEAS www.allenoverseas.com
Mental Ability

PAPER FOLDING &


CHAPTER 3 PAPER CUTTING

3.1 Folding a Transparent Sheet


In such type of problems a figure of transparent sheet carrying a design on it is given. There is a dotted line on
this sheet. This sheet has to be folded along the dotted line.
A candidate requires to identify a figure from given options, that looks similar to the folded sheet.
Solved example
Ex.1 A square transparent sheet X, with a design and a dotted line on it is given. Choose the correct figure from the
options which represents the sheet X after folding sheet along the dotted line.

(1) (2) (3) (4)

Sol. Clearly, the right half of the sheet X is put on the left half. The combination of the design in left half and mirror
image of the design in the right half will appear on the folded sheet. So the sheet will then appear as shown in
figure (3). Hence, the answer is (3).

3.2 Cutting/Punching a Folded Paper


In such type of problems a paper is folded twice or more than twice. Then one or more pieces of it are cut. After
this the paper is unfolded. In this sitution the paper has as many cuts or holes on it as folded. So it contains a
pattern.
A candidate requires to identify a figure from given four options, that shows the similar paper sheet as the
pattern made.
Usually, the paper sheet is folded along the dotted lines marked on it. And arrows show the directions of the folds.
Ex.2 Figures X and Y respectively shows the two consecutive folds of the paper. Figure Z shows the cut on the folded
paper. Choose one figure form the four options that is similar to the unfolded form of the sheet Z.

X Y Z

(1) (2) (3) (4)

Sol. In figure (X), the square sheet of paper has been folded along the vertical line of symmetry so that the right half
of the sheet overlaps the left half.
In figure (Y), the sheet is folded further to a quarter.
In figure (Z), a square has been punched in the folded sheet Y.
Clearly, the punched square will be created in each quarter of the paper.
Thus, when the paper is unfolded, four square punches will appear symmetrically over it and the paper will then
appear as shown in figure (4).
Hence, the answer is (4).

www.allenoverseas.com OVERSEAS 19
Class-VIII

Directions (Q.1 to Q.11) : In each of the following problems, a square transparent sheet (X) with a pattern is
given. Figure out from amongst the four alternatives as to how the pattern would appear when the transparent
sheet is folded at the dotted line.

1.
(1) (2) (3) (4)

2.

(1) (2) (3) (4)


3.

(1) (2) (3) (4)


4. D D
D A
A
A D
A

A
D

(1) (2) (3) (4)


5.

(1) (2) (3) (4)


6.

(1) (2) (3) (4)


7.

(1) (2) (3) (4)


8.

(1) (2) (3) (4)


9.

(1) (2) (3) (4)

20 OVERSEAS www.allenoverseas.com
Mental Ability

10.

(1) (2) (3) (4)

11.

(1) (2) (3) (4)

Direction (Q.12 to Q.19) : There are three forms, X, Y and Z of a sheet. Figures X and Y respectively show
the two consecutive folds of the sheet. And the figure Z shows punch on the folded sheet. Choose one figure
from the four options (1), (2), (3) and (4), that is similar to the unfolded form of the sheet?

12. (1) (2) (3) (4)


X Y Z

13. (1) (2) (3) (4)


X Y Z

14. (1) (2) (3) (4)

15. (1) (2) (3) (4)


X Y Z

16. (1) (2) (3) (4)


X Y Z

17. (1) (2) (3) (4)


X Y Z

www.allenoverseas.com OVERSEAS 21
Class-VIII

18. (1) (2) (3) (4)


X Y Z

19. (1) (2) (3) (4)


X Y Z

20. A square paper is folded in a particular manner and a bunch is made. When unfolded the paper appears
as given below :

Question Figure Answer figures

(1) (2) (3) (4)

Que. 1 2 3 4 5 6 7 8 9 10 11 12 13 14 15
Ans. 2 1 3 2 2 4 3 2 1 1 2 4 2 1 3
Que. 16 17 18 19 20
Ans. 3 2 2 4 4

22 OVERSEAS www.allenoverseas.com
Mental Ability

CHAPTER 4 SERIES
A series is a sequence of number/alphabet or both which follow a particular rule. Each element of series is
called term. We have to analyse the pattern find the missing term or next term to continue the pattern.

Types of Series

Number Alphabet Alpha-Numeric Continuous


Series Series Series Pattern
Series

4.1 Number series


Number series is a form of number is a certain sequence, where some numbers are missing in that series, we
need to observe first and the find accurate number to that series of numbers.
Solved examples
Directions : Find the missing term in each of the following series.
Ex.1 2, 3, 5, 7,?, 13, 17
(1) 9 (2) 21 (3) 25 (4) 11
Sol. Clearly, the given series is consecutive prime number
Hence, the answer is (4).
Ex.2 4, 6, 12, 14, 28, 30,?
(1) 32 (2) 60 (3) 62 (4) 64
Sol. The given sequence is a combination of two series:
(I) 4, 12, 28,? and (II) 6, 14, 30, .....
Now, the pattern followed in each of the above two series is : + 8, + 16, + 32, .....
So, missing number = (28 + 32) = 60.
Hence, the answer is (2).
Ex.3 1,3,3, 6, 7, 9,?, 12,21
(1) 10 (2) 11 (3) 12 (4) 13
Sol. Clearly, the given sequence is a combination of two series;
(I) 1, 3, 7,?, 21 and (II) 3, 6, 9, 12
The pattern followed in I is + 2, + 4, ..... and the pattern followed in II is + 3.
So, missing number = 7 + 6 = 13.
Hence, the answer is (4).
1 3 5 7
Ex.4 Which fraction comes next in the sequence , , , ,?
2 4 8 16
9 10 11 12
(1) (2) (3) (4)
32 17 34 35
Sol. Clearly, the numerators of the fractions in the given sequence form the series 1, 3, 5, 7, in which each term is
obtained by adding 2 to the previous term.
The denominators of the fractions form the series 2, 4, 8, 16, i.e., 21, 22, 23, 24.
So, the numerator of the next fraction will be (7 + 2) i.e. 9 and the denominator will be 2 5 i.e. 32.
9
Thus, the next term is . Hence, the answer is (1).
32

www.allenoverseas.com OVERSEAS 23
Class-VIII

Directions (Q. 1 to Q.17) : In each of the following questions, a number series is given with one term missing.
Choose the correct alternative that will continue the same pattern and replace the question mark in the given
series.
1. 1, 9, 25, 49,?, 121
(1) 64 (2) 81 (3) 91 (4) 100
2. 4, 7, 12, 19, 28,?
(1) 30 (2) 36 (3) 39 (4) 49
3. 11, 13, 17, 19, 23, 25,?
(1) 26 (2) 27 (3) 29 (4) 37
4. 6, 12, 21,?, 48
(1) 33 (2) 38 (3) 40 (4) 45
5. 2, 5, 9,?, 20, 27
(1) 14 (2) 16 (3) 18 (4) 24
6. 6, 11, 21, 36, 56,?
(1) 42 (2) 51 (3) 81 (4) 91
7. 10, 18, 28, 40, 54, 70,?
(1) 85 (2) 86 (3) 87 (4) 88
8. 120, 99, 80, 63, 48,?
(1) 35 (2) 38 (3) 39 (4) 40
9. 22, 24, 28,?, 52, 84
(1) 36 (2) 38 (3) 42 (4) 46
10. 4832, 5840, 6848,?
(1) 7815 (2) 7846 (3) 7856 (4) 7887
11. 10, 100, 200, 310,?
(1) 400 (2) 410 (3) 420 (4) 430
12. 0, 2, 8, 14,?, 34
(1) 20 (2) 23 (3) 24 (4) 25
13. 28, 33, 31, 36,?, 39
(1) 32 (2) 34 (3) 38 (4) 40
14. 125, 80, 45, 20,?
(1) 5 (2) 8 (3) 10 (4) 12
15. 2, 15, 41, 80,?
(1) 111 (2) 120 (3) 121 (4) 132
16. 6, 17, 39, 72,?
(1) 83 (2) 94 (3) 116 (4) 127

24 OVERSEAS www.allenoverseas.com
Mental Ability

17. 325, 259, 204, 160, 127, 105,?


(1) 94 (2) 96 (3) 98 (4) 100
Direction (Q. 18 to Q.24) : One term in the series is wrong. Find the wrong term.
18. 24, 60, 120, 210, 336, 492
(1) 60 (2) 336 (3) 492 (4) 24
19. 28, 84, 112, 196, 308, 504, 872
(1) 112 (2) 196 (3) 308 (4) 872
20. 300, 421, 592, 815, 1104, 1465
(1) 421 (2) 1104 (3) 815 (4) 592
21. 0, 8, 7, 16, 14, 24, 22, 32
(1) 22 (2) 14 (3) 24 (4) 32
22. 1, 11, 22, 34, 47, 61, 76, 93
(1) 93 (2) 1 (3) 34 (4) 76
23. 8, 4, 4, 6, 12, 28, 90
(1) 8 (2) 4 (3) 12 (4) 28
24. 438, 487, 447, 476, 460, 469
(1) 447 (2) 438 (3) 476 (4) 469

Que. 1 2 3 4 5 6 7 8 9 10 11 12 13 14 15
Ans. 2 3 3 1 1 3 4 1 1 3 4 3 2 1 4
Que. 16 17 18 19 20 21 22 23 24
Ans. 3 1 3 4 4 1 1 4 1

www.allenoverseas.com OVERSEAS 25
Class-VIII

4.2 Alphabet series


In this type of questions, a series of single, pairs or groups of letters is given. The terms of the series form a
certain pattern as regards the position of the letters in the English alphabet. The candidate is required to
decipher this pattern and accordingly find the missing term or the wrong term in the given series.
Solved examples
Ex.1 EZ, DX, CV, ?, AR, ZP
(1) CS (2) AM (3) BT (4) TG
Sol. First and second letters follow a sequence of – 1 and – 2 respectively.
Hence the answer is (3)
Ex.2 DIE, XCY, RWS, ?
(1) LQN (2) QMP (3) LMS (4) LQM
Sol. First, second and third letters of each group follow a sequence of – 6 series.
Hence the answer is (4)
Ex.3 Find the next term in the series : KAD, NCF, QEH,?
(1) TGJ (2) RJN (3) SOV (4) QGN
Sol. Clearly, we observe the following pattern :
+3 +3 +3
The first letters follow the pattern + 3 i.e. K ¾¾¾ ® N ¾¾¾ ® Q ¾¾¾ ® T
+2 +2 +2
The second letters follow the pattern + 2 i.e. A ¾¾¾ ® C ¾¾¾ ® E ¾¾¾ ® G
+2 +2 +2
The third letters follow the pattern + 2 i.e. D ¾¾¾ ® F ¾¾¾ ® Hs ¾¾¾ ®J
Thus, the missing term is TGJ. Hence, the answer is (1).

Directions (Q.1 to Q.20) : In each of the following questions, various terms of an alphabet series are given

with one or more terms missing as shown by (?). Choose the missing terms out of the given alternatives.

1. R, U, X, A, D,?

(1) F (2) G (3) H (4) I

2. T, R, P, N, L,?,?

(1) J, G (2) J, H (3) K, H (4) K, I

3. B, D, F, I, L, P,?

(1) R (2) S (3) T (4) U

4. U, B, I, P, W,?

(1) D (2) F (3) Q (4) Z

26 OVERSEAS www.allenoverseas.com
Mental Ability

5. H, I, K, N,?

(1) O (2) Q (3) R (4) S

6. Z,?, T,?, N,?, H,?, B

(1) W, Q, K, E (2) W, R, K, E (3) X, Q, K, E (4) X, R, K, E

7. A, G, L, P, S,?

(1) U (2) W (3) X (4) Y

8. A, D, H, M,?, Z

(1) T (2) G (3) N (4) S

9. A, I, P, V, A, E,?

(1) E (2) F (3) G (4) H

10. Z, U, Q,?, L

(1) I (2) K (3) M (4) N

11. Z, W, S, P, L, I, E,?

(1) B (2) D (3) F (4) K

12. Z, Y, X, U, T, S, P, O, N, K,?,?

(1) H,G (2) H, I (3) I, H (4) J, I

13. AB, DEF, HIJK,?, STUVWX

(1) LMNO (2) LMNOP (3) MNOPQ (4) QRSTU

14. AI, BJ, CK,?

(1) DL (2) DM (3) GH (4) LM

15. AZ, GT, MN,?, YB

(1) JH (2) SH (3) SK (4) TS

16. GH, JL, NQ, SW, YD,?

(1) EJ (2) FJ (3) EL (4) FL

www.allenoverseas.com OVERSEAS 27
Class-VIII

17. AZ, CX, FU,?

(1) I R (2) I V (3) JQ (4) KP

18. DF, GJ, KM, NQ, RT,?

(1) UW (2) YZ (3) XZ (4) UX

19. ajs, gpy,?, sbk, yhq

(1) dmv (2) mve (3) oua (4) qzi

20. PMT, OOS, NQR, MSQ,?

(1) LUP (2) LVP (3) LVR (4) LWP

Que. 1 2 3 4 5 6 7 8 9 10

Ans. 2 2 3 1 3 1 1 4 4 4

Que. 11 12 13 14 15 16 17 18 19 20

Ans. 1 4 3 1 2 4 3 4 2 1

28 OVERSEAS www.allenoverseas.com
Mental Ability

4.3 Alpha-numeric series


This type of questions is just a jumbled form of questions of Type I and Type II, which you have just read. Here,
the terms of the given series are a combination of letters and numerals, which move according to a set pattern.
Study the following examples:
Solved examples
Ex. U32, V29, __, X23, Y20
(1) W26 (2) W24 (3) Y21 (4) T26
Sol. In this series, the letters progress by 1; the numbers decrease by 3.
Hence answer is (1)

Directions (Q.1 to Q.7) : In each of the following questions, a letter-number series is given with one or more
terms missing as shown by (?). Choose the missing term out of the given alternatives.
1. B2E, D5H, F12K, H27N ?
(1) J58Q (2) J56Q (3) J57Q (4) J56P
2. B3M, E7J, H15G, K31D ?
(1) N65A (2) O63A (3) N63A (4) N632
3. 5X9, 8U12, 11R15, 14O18, ?
(1) 17L21 (2) 17K21 (3) 17M21 (4) 17L23
4. P3, ?, J9, G12, D15
(1) K6 (2) M6 (3) M8 (4) L8
5. 18Z, 17Y, 16X, 15W,?
(1) 14V (2) 14U (3) U14 (4) V15
6. 55V, 66W, 77X,88Y, ?
(1) 99X (2) 99Z (3) 999Z (4) 99U
7. AB98, CD87, EF76, GH65, ?
(1) IJ54 (2) JK54 (3) IJ53 (4) JK43

Que. 1 2 3 4 5 6 7
Ans. 1 3 1 2 1 2 1

www.allenoverseas.com OVERSEAS 29
Class-VIII

4.4 Continuous pattern series


This type of questions usually consists of a series of small letters which follow a certain pattern. However, some
letters are missing from the series. These missing letters are then given in a proper sequence as one of the
alternatives. You are required to choose this alternative as the answer.
Solved examples
Ex. aab _ aaa _ bba _
(1) baa (2) abb (3) bab (4) aab
Sol. We proceed step by step as shown below:
The first blank space should be filled in by 'b' so that we have two a's followed by two b's. Hence pattern is aabb/
aaabbb/aa. Thus, our answer is 'baa'.

Directions (Q.1 to Q.10) : Which sequence of letters when placed at the blanks one after the other will
complete the given letter series ?
1. _ bca _ ca _ c _ b _
(1) aabbc (2) abbbc (3) aabcc (4) abbac
2. a_baa_aa__ab
(1) a a a a (2) b a a a (3) b b a a (4) a b b a
3. a _ bbc _ aab _ cca _ bbcc
(1) bacb (2) acba (3) abba (4) caba
4. aab_aaa_bba_
(1) b a a (2) a b b (3) b a b (4) a a b
5. _aabb_abba_b
(1) b a b (2) a b a (3) b b a (4) b a a
6. a_baa_baa_ba
(1) a a b (2) b a b (3) b b a (4) b b b
7. _baa _ba_aab_
(1) b a b a (2) b b a a (3) a b b b (4) b b a b
8. abc _ d _ bc _ d _ b _ cda
(1) bacdc (2) cdabc (3) dacab (4) dccbd
9. _bc__bb_aabc
(1) acac (2) babc (3) abab (4) aacc
10. a _ bc _ a _ bcda _ ccd _ bcd _
(1) abddbd (2) acbdbb (3) adbbad (4) bbbddd

Que. 1 2 3 4 5 6 7 8 9 10

Ans. 4 4 2 1 4 4 3 3 1 3

30 OVERSEAS www.allenoverseas.com
Mental Ability

CHAPTER 5 NON-VERBAL SERIES


The word "series" is defined as anything that follows to form a specific pattern or in continuation of a given
pattern or sequence. In this type of non-verbal test, two sets of figure state the problems. The sets are called
problem figures and answer figures, Each problem figure changes in design from the preceding one. The
answer figure set contains 4 figures marked (1), (2), (3), (4). You are required to choose the correct answer
figure, which would best continue the series.
Solved examples
Direction : Study the problem figures and try to establish the relationship between them. From the answer figures
marked (1), (2), (3) and (4) pick out the figure which would complete the series.
Problem Figures Answer Figures

Ex.1

(A) (B) (C) (D) (E) (1) (2) (3) (4)


Sol. It is clear from here that figures rotate clockwise and anti-clockwise making an angle of 90° alternatively and
one figure is deleted in every alternate block. Hence the correct answer figure is (4).

Problem Figures Answer Figures


Ex.2

(A) (B) (C) (D) (E) (1) (2) (3) (4)

Sol. Here deletion of design in the figure takes place with the change in the figures. Arrow is deleted in the block
skipping alternate block with the reversal in design. Similarly, base is also reversed with alternate shifting of
shaded portion. Hence, the correct answer figure is (4).
Problem Figures Answer Figures

Ex.3

(A) (B) (C) (D) (E) (1) (2) (3) (4)


Sol. The problem figure (D) is reverse of problem figure (A) and problem figure (E) is reverse of figure (B). Therefore
answer figure will be reverse of figure (C). Hence, the correct answer figure is (3).
Ex.4 PROBLEM FIGURES ANSWER FIGURES

?
A B C D (1) (2) (3) (4)
Sol. Clearly, two rectangles are added to the figure in each step so as to form stairs. There should be five rectangles
in fig. (C). Hence, the answer is (2).

www.allenoverseas.com OVERSEAS 31
Class-VIII

Ex.5 PROBLEM FIGURES ANSWER FIGURES

?
A B C D (1) (2) (3) (4)
Sol. Three, four, five..... line segments are added to the figure sequentially at the lower end of the figure. Clearly,
there should be nine line segments in fig. (C). Hence, the answer is (1).

Directions (Q.1 to Q.10) : In each of the following questions, there is a set of four figures labelled A, B, C and
D called the Problem Set followed by a set of four other figures labelled 1,2,3 and 4 called the Answer Set. Fig.
(C) contains a question mark. Select a suitable figure from the Answer Set which will substitute this question mark so
that a series is formed by the figures A, B, C and D taken in order. The number of the selected figure is the answer.

1.

A B C D (1) (2) (3) (4)

2.
× C = =
C = C = C C C
A B C D (1) (2) (3) (4)

3.

A B C D (1) (2) (3) (4)

4. ×
× × ×
× ×
A B C D (1) (2) (3) (4)

5.

A B C D (1) (2) (3) (4)

32 OVERSEAS www.allenoverseas.com
Mental Ability

6.

?
A B C D (1) (2) (3) (4)

7.

A B C D (1) (2) (3) (4)

8.
z z
A B C D (1) (2) (3) (4)

9.

A B C D (1) (2) (3) (4)

10.

A B C D (1) (2) (3) (4)

Directions (Q.11 to Q.15) : Each of the following questions consists of five figures marked A, B, C, D and E
called the Problem Figures followed by four other figures marked 1, 2, 3 and 4 called the Answer Figures.
Select a figure from amongst the Answer Figures which will continue the same series as established by the five
Problem Figures.

11.

(A) (B) (C) (D) (E) (1) (2) (3) (4)

C
K
12.
K

C
C

K
C
K

K
K

K
K

C K
C

(A) (B) (C) (D) (E) (1) (2) (3) (4)

www.allenoverseas.com OVERSEAS 33
Class-VIII

13. = C
S O O = O = #
T S # T # #
(A) (B) (C) (D) (E) (1) (2) (3) (4)

14.

(A) (B) (C) (D) (E) (1) (2) (3) (4)

15. 1 1 1 1 1 1 1 1 1

4 4 4 4 4 4 4 4 4
(A) (B) (C) (D) (E) (1) (2) (3) (4)

Que. 1 2 3 4 5 6 7 8 9 10
Ans. 3 4 2 4 3 4 3 4 4 4
Que. 11 12 13 14 15
Ans. 4 3 3 2 4

34 OVERSEAS www.allenoverseas.com
Mental Ability

CHAPTER 6 ANALOGY
'Analogy' means 'correspondence'.
In questions based on analogy, a particular relationship is given and another similar relationship has to be
identified from the alternatives provided. Analogy tests are, therefore, meant to test a candidate's overall
knowledge, power of reasoning and ability to think concisely and accurately. Below are given some common
relationships which will help you to detect most analogies better:
Common relationships
1. Country and capital :
Ex. Afghanistan : Kabul
Kabul is the capital of Afghanistan.
2. State and capital :
Ex. Maharashtra : Mumbai
Mumbai is the capital of Maharashtra.
3. Country and currency :
Ex. India : Rupee
Rupee is the currency of India.
4. Instrument and measurement :
Ex. Barometer : Pressure
Barometer is an instrument used to measure pressure.
5. Quantity and units :
Ex. Length : Metre
Metre is the unit of measuring length.
6. Individual and group :
Ex. Sailors : Crew
A group of sailors is called a crew.
7. Animal and young one :
Ex. Cow : Calf
Calf is the young one of cow.
8. Male and female :
Ex. Horse : Mare
Mare is the female horse.
9. Animal and movement :
Ex. Duck : Waddle
Waddle is the name given to the movement of the duck.
10. Animal/thing and sound :
Ex. Lion : Roar
Roar is the sound produced by a lion.

www.allenoverseas.com OVERSEAS 35
Class-VIII

11. Individual/thing and class :


Ex. Lizard : Reptile
Lizard belongs to the class of Reptiles.
12. Individual and dwelling place :
Ex. Dog : Kennel
A dog lives in a kennel.
13. Animals/things and keeping place :
Ex. Car : Garage
A car is kept in a garage.
14. Games and place of playing :
Ex. Badminton : Court
Badminton is played on a court.
15. Worker and tool :
Ex. Blacksmith : Anvil
Anvil is the tool used by a blacksmith.
16. Tool and action :
Ex. Needle : Sew
A needle is used for sewing.
17. Worker and working place :
Ex. Chef : Kitchen
A chef works in a kitchen.
18. Worker and product :
Ex. Mason : Wall
A mason builds a wall.
19. Product and raw material :
Ex. Prism : Glass
Prism is made of glass.
20. Part and whole relationship :
Ex. Pen : Nib
Nib is a part of a pen.
21. Pair relationship :
Ex. Shoes : Socks
Shoes and socks go together.
22. Study and topic :
Ex. Ornithology : Birds
Ornithology is the study of birds.

36 OVERSEAS www.allenoverseas.com
Mental Ability

23. Word and intensity :


Ex. Anger : Rage
Rage is of higher intensity than Anger.
24. Word and synonym :
Ex. Abode : Dwelling
Abode means almost the same as Dwelling. Thus, Dwelling is the synonym of Abode.
25. Word and antonym :
Ex. Attack : Defend
Defend means the opposite of Attack. Thus, Defend is the antonym of Attack.

1. Completing the analogous pair

In this type of questions, two words are given. These words are related to each other in some way. Another word
is also given. You are required to find out the relationship between the first two words and choose the word from
the given alternatives, which bears the same relationship to the third word, as the first two bear.
Solved examples
Ex.1 Anaemia : Blood :: Anarchy :?
(1) Lawlessness (2) Government (3) Monarchy (4) Disorder
Sol. Anaemia is the state of lack of blood. Similarly, Anarchy is the state of lack of Government. Hence, the answer
is (2).

2. Choosing the analogous pair

In this type of questions, a pair of words is given, followed by four pairs of words as alternatives. The candidate
is required to choose the pair in which the words bear the same relationship to each other as the words of the
given pair bear.
Ex.2 Energy : Joule
(1) Axe : Grind (2) Ammeter : Current
(3) Power : Ampere (4) Resistance : Ohm
Sol. Joule is the unit of measuring energy. Similarly, Ohm is the unit of measuring resistance. Hence, the answer is (4).

3. Number analogy
This section deals with four types of questions:
1. Choosing a number related to a given number in the same manner as the two numbers of another given
pair are related to each other;
2. Choosing a similarly related pair as the given number pair on the basis of the relation between the
numbers in each pair;
3. Choosing a number similar to a group of numbers on the basis of certain common properties that they
possess;
4. Choosing a number set similar to a given number set.

www.allenoverseas.com OVERSEAS 37
Class-VIII

Solved examples
Ex.3 171 : 227 :: 258 : ?
(1) 280 (2) 390 (3) 478 (4) 326
Sol. Clearly, 13 + 2 = 171 and 15 + 2 = 227. So, if the first number is x, the second number is x2 + 2.
2 2

Thus, the relationship is x : x2 + 2.


Hence, the answer is (4).
Ex.4 23 : 72
(1) 8 : 27 (2) 6 : 20 (3) 7 : 10 (4) 9 : 72
Sol. Clearly, the relationship is x : (x × 3 + 3). This relationship exists in (1). Hence, the answer is (1).
Ex.5 Which number is like the given set of numbers? Given Set : (3, 17, 31)
(1) 5 (2) 15 (3) 45 (4) 49
Sol. Clearly, the number in the given set are all prime numbers. 5 is also a prime number and so belongs to the
same group. Hence, the answer is (1).
Ex.6 Which set of numbers is like the given set? Given Set : (48, 24, 12)
(1) (44, 22, 10) (2) (46, 22, 11) (3) (40, 20, 10) (4) (42, 20, 10)
Sol. Clearly, in the given set, the first number is twice the second and the second number is twice the third. A similar
relationship exists between the numbers in the group (40, 20, 10). So, the answer is (3).

4. Alphabet analogy

In this type of questions, two groups of letters related to each other in some way, are given. The candidate is
required to find out this relationship and then choose either a letter-group which is related in the same way to
a third group provided in the question or a pair consisting of similarly related letter-groups.
Solved Examples

Ex.7 ABCD : NPRT : : FGHI :?

(1) KLMN (2) OQRT (3) RTUW (4) SUWY

Sol. Clearly, the first, second, third and fourth letters of the first group are moved 13, 14, 15 and 16 steps forward
respectively to obtain the corresponding letters of the second group.

A B C D F G H I
+13¯ +14¯ +15¯ +16¯ +13¯ +14¯ +15¯ +16¯
N P R T S U W Y

Hence, the answer is (4).

38 OVERSEAS www.allenoverseas.com
Mental Ability

Direction (Q. 1 to Q. 6 ) : In each of the following questions, there is some relationship between the two terms to left
of:: and the Same relationship continues between the two terms to its right. Also in each question, one term either to
the right of:: or to the left of it is missing. This term is given as one of the alternatives given below. Find out the missing
term.
1. Pulp : Paper :: Hemp :?
(1) Rope (2) Basket (3) Cooton (4) Yarn
2. Cub : Tiger :: Fawn :?
(1) Sheep (2) Ass (3) Stag (4) Donkey
3. Shoal : Fish :: Swarm :?
(1) Tigers (2) Snakes (3) Birds (4) Bee
4. Mirror : Reflection :: Water :?
(1) Refraction (2) Immersion (3) Dispersion (4) Attraction
5. Snake : Reptile :: Whale :?
(1) Insect (2) Mammal (3) Money (4) String
6. France : Euro :: Canada :?
(1) Rand (2) Dollar (3) Pound (4) Yaun
Direction (Q.7 to Q.14): A pair of words is given. Find the pair which shows a simialr relationship that
shown by the given pair.
7. Restaurant : Menu
(1) Library : Catalogue (2) Journal : Newspaper (3) College : Account (4) Book : Encyclopedia
8. Light : Darkness
(1) Santiy : Madness (2) Medicine : Patient (3) Anger : Friendship (4) Education : Illiteracy
9. Fragile : Crack
(1) Irreducible : Reduce (2) Pliable : Bend (3) Cemetery : Death (4) Aerial Air
10. Soldier : Regiment
(1) Sailor : Crew (2) Drop : Ocean (3) Flower : Bunch (4) Bounce : Jungle
11. A : BC
(1) D : EG (2) W : UV (3) L : NO (4) H : IJ
12. PRT : UVW
(1) BDF : HGI (2) ACE : FGH (3) LNP : STU (4) GHJ : KLM
13. XYZ : ABC
(1) TUV : DEF (2) UVW : EFG (3) UVW : DEF (4) STU : HGF
14. 9125 : 5219
(1) 9215 : 5214 (2) 1296 : 2098 (3) 4164 : 4641 (4) 4116 : 6114
15. Choose one numbers, which is similar to the number in the given set : 992, 733, 845, 632
(1) 114 (2) 326 (3) 425 (4) 947

www.allenoverseas.com OVERSEAS 39
Class-VIII

Direction (Q. 16 to Q. 22) : In each of the following questions, there is some relationship between the two terms to
left of:: and the Same relationship continues between the two terms to its right. Also in each question, one term either
to the right of:: or to the left of it is missing. This term is given as one of the alternatives given below. Find out the
missing term.
16. Melt : Liquid : : Freeze :?
(1) Ice (2) Condense (3) Solid (4) crystal
17. Paw : cat : : Hoof :?
(1) Horse (2) Lion (3) Lamb (4) Elephant
18. Bank : River : : Coast :?
(1) Flood (2) Waves (3) Sea (4) Beach
19. Ocean : Water : : Glacier :?
(1) Refrigerator (2) Ice (3) Desert (4) Cave
20. Reading : Knowledge : : Work :?
(1) Experience (2) Engagement (3) Employment (4) Experiment
21. Cardiology : : Heart : : Zoology :?
(1) Science (2) insect (3) Eggs (4) None
22. Sky : Ground : : Ceiling :?
(1) Road (2) Ground (3) Floor (4) None
Direction (Q.23 to Q.25) : Choose the correct pair :
23. Elevated : Exalted
(1) Dirty : Filthy (2) Disorderly : Unfaithful
(3) Raise : Commensurate (4) Promoted : Excellence
24. Badminton: court
(1) Hockey : Stick (2) Skating : Rink
(3) Cricket : bat (4) Football : Goal
25. Food : Hungry
(1) Thought : politics (2) Water : River
(3) Rest : Weary (4) wine : Intoxication
Direction (Q.26 to Q.28) : In each of the following Questions, there is a certain relationship between two given
numbers on one side of :: and one number is given on another side of :: while another number is to be found from the
given alternatives, having the same relationship with this number as the numbers of the given pair bear : choose the
best alternative
26. 14 : 9 : : 26 :?
(1) 12 (2) 13 (3) 15 (4) 31

40 OVERSEAS www.allenoverseas.com
Mental Ability

27. 8 : 28 : 27 :?
(1) 55 (2) 63 (3) 64 (4) 65
28. 68 : 130 : :? : 350
(1) 210 (2) 216 (3) 222 (4) 240
Direction (Q.29 & Q.30) : In each of the following Questions, choose that set of numbers from the four
alternatives sets, that is similar to the given set.
29. Given Set : (9, 15, 21)
(1) (10, 14, 16) (2) (7, 21, 28) (3) (5, 10, 25) (4) (6, 9, 12)
30. Given Set : (14, 23, 32)
(1) (15, 23, 31) (2) (14, 19, 24) (3) (13, 21, 29) (4) (12, 21, 30)

Que. 1 2 3 4 5 6 7 8 9 10 11 12 13 14 15
Ans. 1 3 4 1 2 2 1 4 2 1 4 2 3 4 3
Que. 16 17 18 19 20 21 22 23 24 25 26 27 28 29 30
Ans. 3 1 3 2 1 4 3 1 2 3 3 4 3 4 4

www.allenoverseas.com OVERSEAS 41
Class-VIII

Important Notes

42 OVERSEAS www.allenoverseas.com
Mental Ability

CHAPTER 7 CLASSIFICATION
Classification means to assort the items of a given group on the basis of certain common quality they possess and
then sport the stranger out.

These test judge your ability to observe differences and similarities among items. You must determine the
similarity of characteristics in the given terms and then identify the one which does not have same characteristics.
These characteristics can be based on relationship, small and capital letter relationship, vowel and consonant
relationship, repetitions and skipping pattern relationship, letter formation relationship.

1. Choosing the odd word

Solved examples

Ex.1 Find the odd one out :

(1) Biscuits (2) Chocolate (3) Cake (4) Bread

Sol. All except Chocolate are baked items. Hence answer is (2).

Ex.2 Find the odd one out :

(1) Asia (2) Argentina (3) Africa (4) Australia

Sol. All except Argentina are continents, while Argentina is a country. Hence answer is (2).

2. Choosing the odd pair of words

Ex.3 Find out the pair which is different from the other given pairs?

(1) Water : Thirst (2) Talent : Education (3) Food: Hunger (4) Air : Suffocation

Sol. In all other pairs, lack of first causes the second. Hence answer is (2).

3. Choosing the odd numeral

Ex.4 Find the odd one out :

(1) 6 (2) 12 (3) 18 (4) 7

Sol. 7 is the only prime number in the group. Hence answer is (4).

Ex.5 Find the odd one out :

(1) 25631 (2) 33442 (3) 34424 (4) 52163

Sol. In all other numbers, the sum of the digits is 17. Hence answer is (2).

www.allenoverseas.com OVERSEAS 43
Class-VIII

4. Choosing the odd numeral pair/group

Ex.6 Find the odd pair :

(1) 34 - 43 (2) 55 - 62 (3) 62 - 71 (4) 83 - 92

Sol. In all other pairs, second number is 9 more than the first. Hence answer is (2).

5. Choosing the odd letter group

Ex.7 Find the odd letters :

(1) PRT (2) MOQ (3) GEC (4) TVX

Sol. All other groups contain alternate letters of the alphabet in order. Hence answer is (3).

Ex.8 Find the odd letters :

(1) ABDG (2) IJLO (3) MNPS (4) RSUY

Sol. In all other groups, the first, second and third letters are respectively moved one, two and three steps forward
to give the second, third and fourth letters respectively. Hence answer is (4).

Direction (Q. 1 to Q.30): Here we are given a group of items all except one are similar to one another
in some manner. We have to choose this one item which does not fit into the given group.
1. (1) Gold (2) Silver (3) Brozne (4) Platinum

2. (1) Pound (2) Mark (3) Waterloo (4) Yen

3. (1) Mango (2) Potato (3) Onion (4) Ladyfinger

4. (1) Mumbai (2) Bhopal (3) Chennai (4) Kolkata

5. (1) Goa (2) Shimla (3) Mussourie (4) Nainital

6. (1) Kabul (2) Cairo (3) Moscow (4) California

7. (1) Whale (2) Tiger (3) Lion (4) Dog

8. (1) Axe (2) Knife (3) Shovel (4) Saw

9. (1) Needle (2) Pin (3) Nail (4) Hammer

10. (1) Jaguar (2) Baboon (3) Gorilla (4) Chimpanzee

11. (1) Blue (2) Yellow (3) Brown (4) Red

44 OVERSEAS www.allenoverseas.com
Mental Ability

12. (1) Bake (2) Peel (3) Roast (4) Boil

13.

14.

15.

16.

17.

753 703 653 605


18.
(1) (2) (3) (4)

19.

P17A Q19B R21C T25F


20.
(1) (2) (3) (4)

J100 L144 R324 P400


21.
(1) (2) (3) (4)

28MO 27BX 26JP 24DT


22.
(1) (2) (3) (4)

242 482 565 393


23.
(1) (2) (3) (4)

24. (1) Lima (2) Tokyo (3) Beijing (4) New York

www.allenoverseas.com OVERSEAS 45
Class-VIII

25. (1) Coast (2) Harbour (3) Peninsula (4) Oasis


26. (1) 95 (2) 7 (3) 12 (4) 18
27. (1) 325 (2) 639 (3) 426 (4) 286
28. (1) 23-4 (2) 32-9 (3) 44-256 (4) 52-25
29. (1) UNICEF (2) IMF (3) WHO (4) SAARC
30. (1) TWQ (2) PSM (3) WZT (4) EIB

Que. 1 2 3 4 5 6 7 8 9 10 11 12 13 14 15
Ans. 3 3 1 2 1 4 1 3 4 1 3 2 4 3 4
Que. 16 17 18 19 20 21 22 23 24 25 26 27 28 29 30
Ans. 1 4 3 4 4 4 2 3 4 4 2 4 1 4 4

46 OVERSEAS www.allenoverseas.com
Mental Ability

NON-VERBAL ANALOGY &


CHAPTER 8 NON-VERBAL CLASSIFICATION

8.1 Non-Verbal Analogy


Analogy' implies 'Corresponding'. In the problems based on analogy, a pair of related figures is provided and a similar
relationship is to be established between two other figures, by selecting one or both of them from a set of alternatives.
The various types of problems upon Analogy have been discussed with examples and exercises in this chapter.
This type of Analogy involves problems consisting of four figures marked A, B, C and D forming the Problem
Set and four other-figures marked 1, 2, 3 and 4 forming the Answer Set. The figures A and B of the Problem
set are related in a particular manner and a similar relationship is to be established between figures C and D by
choosing a figure from the Answer set which would replace the question mark in fig. (C) or fig. (D).
Solved examples
Directions: Figures A and B are related in a particular manner. Establish the same relationship between
figures C and D by choosing a figure from amongst the four alternatives, which would replace the question mark
in fig. (D).

Ex.1 Which shape or pattern completes the second pair in the same way as the first pair?

×
× × ?
×

×
×
×

(1) (2) (3) (4)

Sol. We can quickly note that in the first pair there is no change of shape and size but the figure is rotating
anticlockwise through 135° (90° + 45°). The same relation is found in the answer figure (1). Hence, the answer
is (1).
Ex.2 Which shape or pattern completes the second pair in the same way as the first pair.

?
(1) (2) (3) (4)

Sol. In the first pair, the second figure is the water image of the first figure. Similarly in the second pair, the required
design will be the water image of the first figure. Hence, the answer is (1).

www.allenoverseas.com OVERSEAS 47
Class-VIII

Ex.3 Which shape or pattern completes the second pair in the same way as the first pair?

?
(1) (2) (3) (4)

Sol. In the first pair, from figure (2) to (1), white circles are reduced to half the number and become concentric
circles. Also the black circles are reduced to half their number and attached to the other circles. Similarly
applying the rule for the second pair, we can observe that the first figure matches with option (3). Hence, the
answer is (3).

Directions (Q.1 to Q.7): Each of the following questions consists of two sets of figures. Figures A, B, C and D
constitute the Problem Set while figures 1, 2, 3 and 4 constitute the Answer Set. There is a definite relationship
between figures A and B. Establish a similar relationship between figures C and D by selecting a suitable figure
from the Answer Set that would replace the question mark (?) in fig. (D).
PROBLEM FIGURES ANSWER FIGURES

1.
?
A B C D (1) (2) (3) (4)
2.
?
A B C D (1) (2) (3) (4)
3.

?
A B C D (1) (2) (3) (4)
4.

?
A B C D (1) (2) (3) (4)
5.
?
A B C D (1) (2) (3) (4)

48 OVERSEAS www.allenoverseas.com
Mental Ability

6.
?
A B C D (1) (2) (3) (4)

7.
?
A B C D (1) (2) (3) (4)

Directions (Q.8 to Q.14): In each of the following questions, a related pair of figures (unnumbered) is followed
by four other pairs of figures numbered as 1, 2, 3 and 4. Out of the four numbered pairs, select the pair that has
a relationship similar to that in the unnumbered pair. The best answer is to be selected from a group of fairly
close choices.

8.

I II I II I II I II I II

(1) (2) (3) (4)

9.

I II I II I II I II I II

(1) (2) (3) (4)

10.

I II I II I II I II I II

(1) (2) (3) (4)

11.

I II I II I II I II I II

(1) (2) (3) (4)

12.

I II I II I II I II I II

(1) (2) (3) (4)

www.allenoverseas.com OVERSEAS 49
Class-VIII

13.

I II I II I II I II I II

(1) (2) (3) (4)

14.

I II I II I II I II I II

(1) (2) (3) (4)

15. Which shape of the figure completes the second pair in a similar way as the first pair?

is to as is to

(1) (2) (3) (4)

Directions (Q.16 & Q.17): Each of the following questions consists of two sets of figures. Figures A, B, C and D
constitute the Problem Set while figures 1, 2, 3, 4 and 5 constitute the Answer Set. There is a definite relationship
between figures A and B. Establish a similar relationship between figures C and D by selecting a suitable figure
from the Answer Set that would replace the question mark (?) in fig. (D).

16.
?
A B C D (1) (2) (3) (4)

17.

?
A B C D (1) (2) (3) (4)

Directions (Q.18 & Q.19) : In each of the following questions, a related pair of figures (unnumbered) is
followed by four other pairs of figures numbered as 1, 2, 3 and 4. Out of the four numbered pairs, select the
pair that has a relationship similar to that in the unnumbered pair. The best answer is to be selected from a
group of fairly close choices.

18.

I II I II I II I II I II

(1) (2) (3) (4)

50 OVERSEAS www.allenoverseas.com
Mental Ability

19.

I II I II I II I II I II

(1) (2) (3) (4)


20. The given question consists of two sets of figures. Figures (i), (ii), (iii) and (iv) constitute the Problem Set. There is
a definite relationship between figures (i) and (ii). Establish a similar relationship between figures (iii) and (iv) by
selecting a suitable figure from the options that would replace (?) in figure (iv).
Problem set

(i) (ii) (iii) (iv)

(1) (2) (3) (4)

Que. 1 2 3 4 5 6 7 8 9 10 11 12 13 14 15 16 17 18 19 20
Ans. 2 1 3 2 4 4 4 3 4 3 1 3 4 4 3 2 2 2 3 3

www.allenoverseas.com OVERSEAS 51
Class-VIII

8.2 Non-Verbal Classification


In the chapter on Classification, we deal with problems of 'Odd-Man-Out' type. In such problems, we are given
a set of figures, such that all, except one have similar characteristics/ features. We are required to select the
figure which differs from all other figures in the given set. Several other types of problems based upon classification
are also discussed in details in this chapter.
Solved examples
Direction : In such type of problems, we are given a set of four figures, out of which all except one are alike in
some manner. We have to select the exclusively different figure in the given set. Following examples will make
understanding easier.

Ex.1

(1) (2) (3) (4)

Sol. Clearly, in all other figures. except fig. (4), the two elements on either side of the line are vertically inverted
images of one another.
Hence the answer is (4).

Ex.2

(1) (2) (3) (4)

Sol. In this case, all the figures, except fig. (4) can be rotated into each other.
Hence the answer is (4).

Ex.3
(1) (2) (3) (4)

Sol. Only in figure 4 all triangles are facing same direction.

52 OVERSEAS www.allenoverseas.com
Mental Ability

Directions (Q.1 to Q.16) : In each problem, out of the four figures marked (1), (2), (3) and (4) three are similar
in a certain manner. However, one figure is not like the other three. Choose the figure which is different from
the rest.

1. 2.

(1) (2) (3) (4) (1) (2) (3) (4)

3. 4.

(1) (2) (3) (4) (1) (2) (3) (4)

5. 6.

(1) (2) (3) (4) (1) (2) (3) (4)

7. 8.

(1) (2) (3) (4) (1) (2) (3) (4)

9. 10.

(1) (2) (3) (4) (1) (2) (3) (4)

11.
A F Z E 12.
(1) (2) (3) (4) (1) (2) (3) (4)

www.allenoverseas.com OVERSEAS 53
Class-VIII

13. 14.
(1) (2) (3) (4) (1) (2) (3) (4)

15. P E I O 16.
, ; . +
(1) (2) (3) (4) (1) (2) (3) (4)

Que. 1 2 3 4 5 6 7 8 9 10
Ans. 4 4 3 3 1 1 4 3 3 2
Que. 11 12 13 14 15 16
Ans. 4 3 3 4 1 4

54 OVERSEAS www.allenoverseas.com
Mental Ability

CHAPTER 9 LOGICAL VENN DIAGRAM

This section deals with questions which aim at analysing a candidate's ability to relate a certain given group of items
and illustrate it diagramatically. Here are a few different types of Venn diagrams with their implications made clear.
Suppose you are given a group of three items. Then,
1. If the items evidently belong to three different groups, the Venn
A B
diagram representing it would be as shown alongside.
Lawyers
Ex. Doctors, Engineers, Lawyers Doctors
C Engineers
These three items bear no relationship to each other. So they are
represented by 3 disjoint figures as shown in fig.
2. If one item belongs to the class of the second and the second
belongs to the class of third, then the representation is in the Hours
C
form of three concentric circles, as shown in fig. B Minutes
Ex. Seconds, Minutes, Hours A
Seconds
Clearly, seconds are a part of minutes and minutes are a part of hours.
So, the Venn diagram would be as shown in the adjoining figure with
circle A representing Seconds, circle B representing Minutes and circle
C representing Hours.
3. If two separate items belong to the class of the third, they are
represented by two disjoint circles inside a bigger circle as
shown in fig. Table A B Chair
Ex. Table, Chair, Furniture
Clearly, table and chair are separate items but both are items of furniture. C Furniture
So, they would be represented as in the adjoining figure with circle A
representing Table, circle B representing Chair and circle C representing
Furniture.
4. If two items belong to the class of the third such that some
items of each of these two groups are common in relationship,
then they are represented by two intersecting circles enclosed
within a bigger circle. Fathers A B Brothers
Ex. Males, Fathers, Brothers
Clearly, some fathers may be brothers and vice-versa. So, fathers and C Males
brothers would be represented by two intersecting circles. Also both
fathers and brothers are males. So, the diagrammatic representation
would be as shown in fig., with circle A representing Fathers, circle B
representing Brothers and circle C representing Males.
5. If two items are partly related to the third, and are themselves
independent of each other they are represented by three
intersecting circles in a line. A B C
Ex. Dogs, Pets, Cats
Clearly, some dogs and some cats are pets. But all the pets are not dogs or Dogs Pets Cats
cats. Also dogs and cats are not related to each other. So, the given items
would be represented as shown in fig. with circle A representing Dogs, circle
B representing Pets and circle C representing Cats.

www.allenoverseas.com OVERSEAS 55
Class-VIII

6. If the three items are partly related to each other, they are
represented as shown in the adjoining figure. Cricket Government
fans Employees
Ex. Cricket fans, Government Employees, Educated Persons
Clearly, some cricket fans may be government employees and some may be A B
educated. Similarly, some government employees may be cricket fans and
C
some may be educated. Also, some educated persons may be cricket fans and
some may be government employees. So, the given items may be represented Educated
as shown in fig. with three intersecting circles denoting the three classes. Persons

7. If one item belongs to the class of second while third item is entirely
different from the two, then they may be represented by the adjoining Human Beings
diagram.
Ex. Engineers, Human Beings, Rats B
A C
Clearly, all engineers are human beings. This would be represented by two
concentric circles. But the class of rats is entirely different from these two.
Thus, these items would be represented as shown in fig. with circle A Engineers Rats
representing Engineers, circle B representing Human beings and circle C
representing Rats.
8. If one item belongs to the class of second and the third item is partly
related to these two, they are represented as shown alongside. Females
Ex. Females, Mothers, Doctors B
Clearly, all mothers are females. This would be represented by two concentric A C
circles. But, some females and some mothers can be doctors. So, the circle
representing doctors would intersect each of the two concentric circles. Thus,
Mothers Doctors
the diagram becomes as shown in fig. with circle A representing Mothers,
circle B representing Females and circle C representing Doctors.
9. If one item belongs to the class of second and the third item is partly
related to the second, they are represented as shown alongside. Males
Ex. Males, Fathers, Children B
Clearly, all fathers are males. This would be represented by two concentric A C
circles. But, some males are children. But, children cannot be fathers. Thus,
the diagram becomes as shown in Fig. with circle A representing Fathers, Children
Fathers
circle B representing Males and circle C representing Children.
10. If two items are partly related to each other and the third item is
entirely different from the two, they are represented as shown
Author
alongside.
Ex. Professor, Author, Children
Clearly, some professors can be authors and vice versa. This would be A B C
represented by two intersecting circles. But the class of children would be
entirely different from these two. Thus, the venn diagram would be as shown Professor Children
in fig. with circle A representing Professors, circle B representing Authors and
circle C representing Children.

56 OVERSEAS www.allenoverseas.com
Mental Ability

Solved Examples
Ex.1 If the items evidently belong to three different groups the Venn diagram representing it would be as shown
below:
Doctors, Engineers, Lawyers

A B

Sol. These three items bear no relationship to each other. So, they are represented by 3 disjoint figures .
Ex.2 If the first word is related to second word and second word is related to third word. Then they will be shown by
diagram as given below:
Unit, Tens, Hundreds

Unit

Tens

Hundreds

Sol. Ten units together make one Tens or in one tens, whole unit is available and ten tens together make one
hundreds.
Ex.3 If two different items are completely related to third item, they will be shown as below:
Pen, Pencil, Stationery

Pen Pencil

Stationery

Ex.4 If there is some relation between two items and these two items are completely related to a third item they will
be shown as given below:
Women, Sisters, Mothers

Sisters Mothers

Women

Sol. Some sisters may be mothers and vice-versa. Similarly some mothers may not be sisters and vice-versa. But all
the sisters and all the mothers belong to women group.

www.allenoverseas.com OVERSEAS 57
Class-VIII

Ex.5 If two items are related to a third item to some extent but not completely and first two items are totally different
they will be shown as given below:
White, Flower, Cloth

Flower White Cloth

Sol. The flower and cloth are different items while some flower may be white. Similarly some cloth may be white.
Ex.6 Study the diagram and identify the people who can speak only one language.

English I L J Hindi
NO
M
K
Tamil

(1) L + M + O (2) K + J + I (3) K (4) I


Sol. The regions represented by the letters K, J and I denote such people who can speak only one language.
Hence, the answer is (2).

Directions (Q.1 to Q.6) : Each of the questions below contains three elements. These three elements may or
may not have some linkage. Each group of the elements may fit into one of the diagrams at (1), (2), (3) and (4).
You have to indicate groups of elements in each of the questions fit into which of the diagrams given below. The
letter indicating the diagram is the answer.

(1) (2) (3) (4)

1. Vegetables, Potato, Cabbage 2. Week, Day, Year


3. Husband, Wife, Family 4. Square, Rectangle, Polygon
5. Bus, Car, Vehicle 6. Anxiety, Intelligence, Strength
Directions (Q.7 to Q.17) : In each of these questions, three words are related in some way. The relationship
among the words in the question can best be represented by one of the four diagrams (1), (2), (3) and (4) given
below. Mark your answer accordingly.

(1) (2) (3) (4)

58 OVERSEAS www.allenoverseas.com
Mental Ability

7. Teacher, Educated, Student 8. Cabinet, Home Minister, Minister


9. Parrots, Birds, Mice 10. Professor, Researcher, Scientist
11. Parents, Mother, Father 12. Musicians, Singers, Women
13. Elephant, Carnivore, Tiger 14. Apple, Mango, Fruit
15. Furniture, Tables, Books 16. Indoor games, Chess, Table tennis
17. Grand-father, Father, Son
Directions (Q.18 to Q.21) : Choose the venn diagram which best illustrates the three given classes in each of
the following questions.

(1) (2) (3) (4)

18. Protons, Electrons, Atoms 19. Paper, Stationary, Ink


20. Science, Physics, Chemistry 21. Atmosphere, Hydrogen, Oxygen
Directions (Q.22 & Q.23) : Each one of the questions below contains three elements. These three elements
may or may not have some linkage. Each group of the elements may fit into one of the diagrams at (1), (2), (3)
and (4). You have to indicate groups of elements in each of the questions fit into which of the diagrams given
below. The letter indicating the diagram is the answer.

(1) (2) (3) (4)

22. Musicians, Instrumentalists, Violinists

23. Sister, Mother, Female

24. Which of the following diagrams indicates the best relation between Moon, Sun and Earth?

(1) (2) (3) (4)

25. Which of the following diagrams indicates the best relation between Engineer, Doctor and People?

(1) (2) (3) (4)

26. Which of the following diagrams indicates the best relation between Lion, Dog and Snake?

(1) (2) (3) (4)

www.allenoverseas.com OVERSEAS 59
Class-VIII

27. Which of the following diagrams indicates the best relation between Elephants, Wolves and Animals?

(1) (2) (3) (4)

Directions (Q.28 to Q.35) : In each of these questions, three words are related in some way. The relationship
among the words in the question can best be represented by one of the four diagrams (1), (2), (3) and (4) given
below. Mark your answer accordingly.

(1) (2) (3) (4)

28. Mother, Homosapien, Woman

29. Men, Rodents, Living beings

30. Chair, Furniture, Pen

31. Fish, Herring, Animals living in water

32. Rings, Ornaments, Diamond rings

33. Vertebrates, Non-vertebrates, Living beings

34. Delhi, India, Asia

35. Singer, Dancer, Actor.

Que. 1 2 3 4 5 6 7 8 9 10 11 12 13 14 15 16 17 18 19 20
Ans. 2 1 2 1 2 3 3 2 4 1 3 1 4 3 4 3 2 2 2 2
Que. 21 22 23 24 25 26 27 28 29 30 31 32 33 34 35
Ans. 2 1 3 3 4 3 2 2 3 4 2 2 3 2 1

60 OVERSEAS www.allenoverseas.com
Mental Ability

Direction (Q.1 to Q.5) : Study the following diagram, information and then answer the questions gives below.

1. If hospital management requires only married trained nurse already working in the hospital for operation
theatre, which part of the diagram should be chosen?
(1) 7 (2) 4 (3) 3 (4) 6
2. Which number represents the married but untrained nurses working in the hospital?
(1) 4 (2) 6 (3) 7 (4) 3
3. Which number represents married trained nurses but not working in the hospital?
(1) 3 (2) 7 (3) 6 (4) 2
4. What is represented by the number 7?
(1) Married but untrained nurses in the hospital (2) Trained nurses
(3) Unmarried trained nurses (4) Married trained nurses
5. Which number represents the trained unmarried nurses working in the hospital?
(1) 6 (2) 5 (3) 7 (4) 4
6. In the given diagram, triangle represents 'girls', rectangle represents 'players' and circle represents 'coach.'
Which part of the diagram represents the girls who are players but not coach?

P Q R T
N

(1) P (2) Q (3) R (4) N


Direction (Q.7 to Q.9) : The diagram given below represent those students who play Cricket, Football and
Kabaddi. Study the diagram and identify

Kabaddi

P Q R
S T Y
Football Z
Cricket

7. The students who play all the three games.


(1) P + Q + R (2) Z + T (3) S + T + Z (4) S
8. The students who plays only football.
(1) Q (2) Z (3) P (4) Y

www.allenoverseas.com OVERSEAS 61
Class-VIII

9. The students who play kabaddi and cricket only.


(1) R (2) T (3) Z (4) Q
10. In the figure given below, square represents doctors, triangle represents ladies and circle represents professors.
By which letter the ladies who are doctor and professor both are represented?

(1) X (2) Y (3) Z (4) A


11. In the given figure, triangle represents healthy people, rectangle represent old persons and circle represent
men. What is the number of those men who are healthy but not old?

(1) 3 (2) 4 (3) 6 (4) 2


Direction (Q.12 to Q.14) : In the following diagram, the circle stands for Professors, the triangle stands for
Surgical Specialists and Medicine Specialists are represented by the rectangle. Answer the questions based on
given information.

P
S
X
Y
Q Z

12. Surgical Specialists who are also Medicine Specialists but not professors are represented by?
(1) Q (2) X (3) Y (4) Z
13. R represents?
(1) Medicine Specialists only (2) Professors only
(3) Surgical Specialists only (4) Medicine and Surgical Specialists only
14. Q represents?
(1) Professors who are neither Medicine nor Surgical Specialists
(2) Professors who are not Surgical Specialists
(3) Medicine Specialists who are neither Professors nor Surgical Specialists
(4) Professors who are not Medicine Specialists

62 OVERSEAS www.allenoverseas.com
Mental Ability

15. Study the diagram and identify the people who can speak only one language.

(1) L + M + O (2) T + H + E (3) T (4) E


16. Terrorists who are criminal but not hard-core are represented by ________.

(1) 2 (2) 3 (3) 4 (4) 1


Direction (Q.17 to Q.20) : Study the following diagram, information given and hence answer each of the
question given below.

10
Persons who take milk
25
20 17 30 40
15 Persons who take coffee
7 20
20
Persons who take tea

The number in a geometrical figure indicates the number of corresponding persons.


17. How many persons take milk?
(1) 100 (2) 82 (3) 92 (4) 122
18. How many persons are there who take only coffee?
(1) 90 (2) 45 (3) 25 (4) 20
19. How many persons take all the three?
(1) 20 (2) 17 (3) 25 (4) 15
20. How many persons are there who take both tea and coffee but not milk?
(1) 22 (2) 17 (3) 7 (4) 20

Que. 1 2 3 4 5 6 7 8 9 10
A ns . 3 3 3 1 4 2 4 3 2 2
Que. 11 12 13 14 15 16 17 18 19 20
A ns . 4 4 3 3 2 4 4 2 4 3

www.allenoverseas.com OVERSEAS 63
Class-VIII

Important Notes

64 OVERSEAS www.allenoverseas.com
Mental Ability

CHAPTER 10 BLOOD RELATION

In this section, problems are based on blood relations. The process of solving these problems (puzzles) depends
upon the deep knowledge of blood relations. The common blood relations are : Father, mother, Grandparents,
Wife, Husband, Son, Daughter, Grandchild, Sister, Brother etc.
Remarks :
1. Relatives on the mother's side are called 'maternal'. For example, mother's brother is called
maternal uncle.
2. Relatives on the father's side are called 'paternal.' For example, father's brother is called paternal
uncle.
3. Assume a relation as paternal relation, unless stated otherwise.
HOW TO SOLVE PROBLEMS
Usually two types of problems are asked : (i) Uncoded (ii) Coded.
(i) To solve uncoded problems, find right gender of the required relation and eliminate all wrong choices if
any. Also find generation gaps to solve quickly.
You can solve by drawing a generation diagram. For this put lower generation below the upper generation
and mention m(or m) and F(or f) with in brackets for male and female respectively.
(ii) To solve coded problems, just take two suitable persons under consideration and move forward or backward
as per question with mentioning their gender and relation.

Type 1. Relation Puzzle

In this type of questions, mutual blood relations of more than two persons are mentioned. The candidate is
required to analyse the given information, work out a family chart and then answer the given questions.

Solved examples

Ex.1 Q is the brother of R.P is the sister of Q.T is the brother of S.S is the daughter of R.Who is aunt of S and T?
(1) R and P (2) R (3) Q (4) P
Sol T is the brother of S, who is the daughter of R. So, T and S are the children of R. Now, P is aunt of S and
T. Hence answer is (4).
Ex.2 A is the son of B. C, who is B's sister has a son D and a daughter E. F is the maternal uncle of D. How many
nephews does F have ?
(1) 0 (2) 1 (3) 2 (4) 3
Sol Clearly, F is the maternal uncle of D means F is the brother of D's mother i.e., F is the brother of C. C is the
sister of B.
So, F is the brother of B who is A's mother. Thus. F is the maternal uncle of A. So, A and D are the nephews
of F i.e., F has two nephews. Hence answer is (3)

www.allenoverseas.com OVERSEAS 65
Class-VIII

Type 2 : Coded Relations


In such questions, the relationships are represented by certain specific codes or symbols such as +, –, ×, ÷,
$, @, , O, § etc.
Solved examples
Ex.3 If 'P + Q' means that P is the mother of Q,
'P ÷ Q' means that P is the brother of Q,
'P × Q' means that P is the son of Q,
'P - Q' means that P is the sister of Q,
Which of the following means that C is the sister of D?
(1) C - P ÷ D (2) P + D ÷ C (3) D × P - C (4) D - C × P
Sol: 'C - P' means that C is the sister of P,
'P ÷ D' means that P is the brother of D.
As C is the sister of P & P is the brother of D, we can say C is the sister of D. Hence answer is (1).
Ex.4 'X is the mother of Y' is represented by 'X @ Y'
'X is the husband of Y' is represented by 'X $ Y'
'X is the sister of Y' is represented by 'X # Y'
'X is the son of Y' is represented by 'X * Y'
If F # J * T $ R @ L, then which of the following is definitely true?
(1) L is the brother of F (2) F is the sister of L
(3) F is the brother of J (4) L is the brother of J
Sol 'F # J' means X is the sister of Y,
'J * T' means J is the son of T,
'T $ R' means T is the husband of R,
'R @ L' means R is the mother of L
Thus, we can definitely say F is the sister of L. Hence answer is (2).

Type 3. Deciphering jumbled up descriptions

In this type of questions, a round about description is given in the form of certain small relationships and
you are required to analyse the whole chain of relations and decipher the direct relationship between the
persons concerned.
Solved examples

Ex.5 A woman introduces a man, as the son of the brother of her mother. How is the man, related to the woman?
(1) Nephew (2) Son (3) Cousin (4) Husband

Sol Brother of mother - Uncle: Uncle's son - Cousin. Hence answer is (3).

66 OVERSEAS www.allenoverseas.com
Mental Ability

1. A is the brother of B, B is the wife of C, C is the son of P, P is the wife of Q. What is Q of B?

(1) Mother-in-law (2) Son-in-law (3) Father-in-law (4) Brother-in-law

2. P is Q's brother, X is P's mother, Y is X's father, Z is Q's son. How is Z related to P?

(1) Cousin (2) Grandfather (3) Nephew (4) Uncle

3. P is Y's brother. Y is Q's father. Q and X are sisters. How is X related to P?

(1) Niece (2) Cousin (3) Aunt (4) Nephew

4. Pointing to a girl John said, "She is the daughter of the only sister of my father." How is John related to the
girl?

(1) Uncle (2) Cousin (3) Father (4) Grandfather

5. A is the son of C; C and Q are sisters; Z is the mother of Q and P is the son of Z. Which of the following statement
is true?

(1) P and A are cousins (2) Q is the maternal grandfather of A

(3) P is the maternal uncle of A (4) C and P are sisters

6. Ravi introduces Raman as the son of the brother of his father's wife. How is Raman related to Ravi?

(1) Son (2) Cousin (3) Uncle (4) Son-in-law

7. P is Q's father's nephew. R is Q's cousin but not the brother of P. How is R related to P if P has only one uncle?

(1) Sister (2) Father (3) Mother (4) Cannot be determined

8. M is the father of O and P is the son of Q. N is the brother of M. If O is the sister of P, how is N related to
Q?

(1) Daughter (2) Brother-in-law (3) Sister-in-law (4) Husband


9. Pointing towards Neha, Dhawal says, "This girl is the daughter of only child of my father." What is the relation
of Dhawal's wife to Neha?
(1) Daughter (2) Aunt (3) Mother (4) Sister
10. Pointing to a man in a photograph, a woman says, "He is the only son of the only daughter-in-law of my only
son's father." How is the man related to the woman?
(1) Son (2) Father (3) Son-in-law (4) Grandson
11. Pointing to a man in the photograph, Priya said, "His mother's daughter is my mother". How is Priya related
to that man?
(1) Nephew (2) Sister (3) Wife (4) Niece

12. If 'P + Q' means 'P is the brother of Q', 'P × Q' means 'P is the father of Q' and ' P – Q' means 'P is the sister
of Q', which of the following relations shows that 'l is the niece of K'?

(1) K + Y × l – Z (2) K × Y + l – Z (3) Z –l × Y + K (4) K + Y + Z – l

www.allenoverseas.com OVERSEAS 67
Class-VIII

13. If A $ B' means 'B is the father of A; 'A # B' means 'B is the mother of A'; 'A*B' means 'B is the sister of A'
and 'A @ B' means 'B is the husband of A', which of the following indicates that N is the grandmother of P'?

(1) P * Q # M $ N (2) P @ Q $ M # N (3) P # Q $ N * M (4) P * Q $ M # N

14. If 'P + Q' means 'P is the daughter of Q'; 'P–Q' means 'P is the brother of Q'; P % Q' means 'P is the father
of Q' and 'P × Q' means 'P is the sister of Q'. Which of the following means 'l is the niece of J?

(1) J – N % C × l (2) l × C – N % J (3) l × C + N – J (4) J + M × C % l

15. If 'A $ B' means 'A is the brother of B';

'A @ B' means 'A is the wife of B';

'A # B' means 'A is the daughter of B' and

'A*B' means 'A is the father of B;

Which of the following indicates that 'U is the father-in-law of P'?

(1) P @ Q $ T # W * U (2) P @ W $ Q * T # U (3) P @ Q $ W * T # U (4) P @ Q $ T # U * W

16. Read the following statements carefully :

(A) A3B means A is the wife of B.

(B) A4B means A is the brother of B.

(C) A5B means A is the father of B.

(D) A9B means A is the sister of B.

Which of the following means F is the mother of K?

(1) F9M4N3K (2) F5M3K (3) F3M5K (4) F3M5N3K

17. If M × N means M is the daughter of N; M + N means M is the father of N; M % N means M is the mother
of N and M – N means M is the brother of N, then P % Q + R – T × K indicates which relation of P to K?

(1) Mother-in-law (2) Sister-in-law (3) Mother (4) Aunt

18. If 'A +B' means that A is the father of B; 'A–B means that A is the wife of B; 'A × B means that A is the brother
of B; 'A ¸ B' means that A is the daughter of B. Then, which of the following is true for P ¸ R + S + Q ?
(1) P is the daughter of Q (2) Q is the aunt of P (3) P is the aunt of Q (4) P is the mother of Q
19. Pointing to a boy, Arun said to Pushpa, "The mother of his father is the wife of your grandfather (Mother's father)".
How is Pushpa related to that Arun?
(1) Sister (2) Niece (3) Cousin (4) Wife
20. The Sharma have three children Sunita, Sanjay and Sheela. Sunita is married to Sonil Mahajan and they have
a son Shoban. Sheela marries Sanjay Bhandari and Vinit and Lily are their Children. Sanjay (Sunita's brother)
is younger to Sunita but elder to Sheela. What is the surname of Shoban?
(1) Bhandari (2) Sharma (3) Mahajan (4) None of these

68 OVERSEAS www.allenoverseas.com
Mental Ability

Directions (Q.21 to Q.25) : Read the following inoformation carefully and answer the questions given below
it : A family consists of six members P, Q, R, X, Y and Z. Q is the son of R but R is not mother of Q. P and
R are a married couple. Y is the brother of R. X is the daughter of P. Z is the brother of P.
21. Who is the brother-in-law of R?
(1) P (2) Z (3) Y (4) X
22. Who is the father of Q?
(1) R (2) P (3) Z (4) None of these
23. How many children does P have?
(1) One (2) Two (3) Three (4) Four
24. How many female members are there in the family?
(1) One (2) Two (3) Three (4) Four
25. How is Q related to X?
(1) Husband (2) Father (3) Brother (4) Uncle
26. P and Q are sisters. P is the mother of R. Q's daughter S is married with T. X is the husband of P. How is Q
related to T?
(1) Mother (2) Sister-in-law (3) Mother-in-law (4) None of these
27. Pointing to a photograph, Rahul said, " She is the daughter of my grandfather's only son". How is Rahul related
to the girl in the photograph?
(1) Father (2) Brother (3) Cousin (4) Can't be determined
Direction (Q.28 & Q.29) : Answer the question based on the following information :
(i) A+B means A is the mother of B.
(ii) A–B means A is the sister of B.
(iii) A*B means A is the father of B.
(iv) ApB means A is the brother of B.
28. Which of the following means Q is the grandfather of P?
(1) P + N * M * Q (2) Q * N * M + P (3) Q p M p N * P (4) Q * N * M – P
29. Which of the following means that N is the maternal uncle of M?
(1) N – Y + A p M (2) N p P – L + E – M (3) M – Y * P – N (4) N p C + F * M
30. Mohit is the brother of Beena. Beena is the wife of Raj. Raj is the brother of Megha. What is the relation of
Megha to Mohit?
(1) Sister (2) Sister-in-law (3) Mother-in-law (4) No specific relation

Que. 1 2 3 4 5 6 7 8 9 10 11 12 13 14 15
Ans . 3 3 1 2 3 2 4 2 3 4 4 1 4 3 4
Que. 16 17 18 19 20 21 22 23 24 25 26 27 28 29 30
Ans . 3 1 3 3 3 2 1 2 2 3 3 2 4 2 4

www.allenoverseas.com OVERSEAS 69
Class-VIII

Important Notes

70 OVERSEAS www.allenoverseas.com
Mental Ability

CHAPTER 11 ANALYTICAL REASONING

The chapter on Analytical Reasoning involves the problems relating to the counting of geometrical figures in a
given complex figure. The systematic method for determining the number of any particular type of figure by the
analysis of the complex figure would be clear from the examples that follow.
Solved examples
Ex.1 How many triangles are there in the following figure?

(1) 12 (2) 14 (3) 15 (4) 13


Sol. The figure in question has been labelled as shown in the following figure.
A B
G
P H
E F

D C
The triangles are as follows.
EAD, EAP, EDP, ABD, AGD, PHD, AGB, GBF, GHF, BDC, AEF, EHD, APF, ABF, BHF
Hence, there are a total of fifteen triangles in the figure.
Ex.2 How many squares are there in the following figure?

(1) 10 (2) 16 (3) 14 (4) None of these


Sol. There are a total of nineteen squares in the figure, which have been marked as shown in the following figure.
A E F G B

T Q H I J L
R
S O M
P N

D U V W C
There are three squares in the upper row EFIH, FGJI and GBIJ.
There are five squares in the middle row – TORS, QHPR, HIOP, IJNO and JLMN.
There are three squares in the lower row – SRUD, RPVU, POWV.
There are a total of six squares three in each of the two rectangles – ABMS, TLCD, AEPS, EGNP, FBMO,
THVD, QIWU, ILCW.
There are two squares in the main rectangle ABCD–AFWD, EBCV.
Hence there are a total of nineteen squares in the main figure.

www.allenoverseas.com OVERSEAS 71
Class-VIII

Direction (Q.1 to Q.7) : Find the minimum number of straight lines used in forming the given figure in each of
the following questions.

1.

(1) 12 (2) 11 (3) 13 (4) 14

2.

(1) 16 (2) 13 (3) 14 (4) 15

3.

(1) 10 (2) 11 (3) 9 (4) 12

4.

(1) 15 (2) 16 (3) 17 (4) 18

5.

(1) 14 (2) 13 (3) 11 (4) 12

6.

(1) 22 (2) 21 (3) 24 (4) 25

72 OVERSEAS www.allenoverseas.com
Mental Ability

7.

(1) 15 (2) 14 (3) 13 (4) 16


8. Identify the number of triangles in the given figure?

(1) 8 (2) 9 (3) 10 (4) 11


9. How many triangles are there in the given figure?

(1) 12 (2) 10 (3) 8 (4) 6


10. How many triangles are there in the given figure ?

(1) 2 (2) 4 (3) 6 (4) 8


11. How many squares are there in the given figure?

(1) 32 (2) 30 (3) 28 (4) 24


12. How many squares are there in the given figure ?

(1) 20 (2) 19 (3) 18 (4) 17

www.allenoverseas.com OVERSEAS 73
Class-VIII

13. How many squares are there in the given figure ?

(1) 8 (2) 9 (3) 10 (4) 12


14. How many rectangles are there in the given figure ?

(1) 10 (2) 9 (3) 7 (4) 5


15. How many parallelograms are there in the given figure ?

(1) 4 (2) 5 (3) 6 (4) 8


16. How many semi - circles are there in this figure ?

(1) 10 (2) 12 (3) 14 (4) 16


17. How many rectangles are there in the given figure ?

(1) 10 (2) 12 (3) 25 (4) 36


Direction (Q.18 to Q.22) : How many squares are there in the given figures ?
18.

(1) 20 (2) 30 (3) 32 (4) 36

19.

(1) 10 (2) 15 (3) 20 (4) 25


20.

(1) 18 (2) 22 (3) 26 (4) 30

74 OVERSEAS www.allenoverseas.com
Mental Ability

21.

(1) 50 (2) 42 (3) 30 (4) 64

22.

(1) 5 (2) 9 (3) 7 (4) None of these


Direction (Q.23 to Q.30) : How many triangles are there in the given figures ?

23.

(1) 20 (2) 32 (3) 14 (4) 36

24.

(1) 30 (2) 40 (3) 45 (4) 60

25.

(1) 10 (2) 12 (3) 14 (4) 11

26.

(1) 19 (2) 21 (3) 24 (4) 22

27.

(1) 12 (2) 16 (3) 10 (4) 14

www.allenoverseas.com OVERSEAS 75
Class-VIII

28.

(1) 22 (2) 16 (3) 20 (4) 18

29.

(1) 10 (2) 14 (3) 12 (4) 11

30.

(1) 10 (2) 14 (3) 22 (4) 24

Que. 1 2 3 4 5 6 7 8 9 10 11 12 13 14 15 16 17 18 19 20
Ans. 1 3 2 1 4 2 4 4 2 4 1 3 4 1 3 4 4 1 2 1
Que. 21 22 23 24 25 26 27 28 29 30
Ans. 3 2 1 3 4 1 1 4 3 4

76 OVERSEAS www.allenoverseas.com
Mental Ability

NUMBER, RANKING &


CHAPTER 12 TIME SEQUENCE TEST

12.1 Number test


In this type of questions, generally a set, group or series of numerals is given and the candidate is asked to trace
out numerals following certain given conditions or lying at specific mentioned positions after shuffling according
to a certain given pattern.

Solved examples

Ex.1 How many such 5's are there in the following number sequence each of which is immediately preceded by 3 or
4 but not immediately followed by 8 or 9?

35954553584567357554523510

(1) None (2) Three (3) Four (4) Five

Sol. As you know, a number which comes after a given number is said to follow it while the one which comes before
the given number precedes it. Thus, the numbers satisfying the given conditions may be marked as follows:

35954 5 53584 5 673 5 7554 5 23 5 10

Clearly, there are five such 5's. Hence, the answer is (4).

Ex.2 In the series given below, how many 8's are there each of which is exactly divisible by its immediate preceding
as well as succeeding numbers?

2838248248682824838286

(1) One (2) Two (3) Three (4) Four

Sol. Clearly, we may mark such sets of 3 numbers, in which the middle number is 8 and each of the two numbers on
both sides of it is a factor of 8, as shown below:

28382 482 4868 282 4838286

So, there are two such 8's. Hence, the answer is (2).

Ex.3 In the series, 6 4 1 2 2 8 7 4 2 1 5 3 8 6 2 1 7 1 4 1 3 2 8 6

how many pairs of successive numbers have a difference of 2 each?

(1) Four (2) Five (3) Six (4) Seven

Sol. Clearly, the pairs of successive numbers having a difference of 2 can be shown as follows :

64 12287 42 1 53 86 2 1 7 1 4 1 3 2 8 6

Thus, there are six such pairs. Hence, the answer is (3).

www.allenoverseas.com OVERSEAS 77
Class-VIII

Ex.4 If it is possible to form a number with the second, the fifth and the eighth digits of the number 31549786, which
is the perfect square of a two-digit even number, which of the following will be the second digit of that even
number?

(1) 1 (2) 4

(3) 6 (4) No such number can be formed

Sol. The 2nd, 5th and 8th digits of the number 31549786 are 1, 9 and 6 respectively.

The perfect square of a two-digit even number, formed using the digits 1, 9 and 6 is 196. And, 196 = 14 2.

So, the required even number is 14. Clearly, its second digit is 4. Hence, the answer is (2).

1. Which is the third number to the left of the number which is exactly in the middle of the following sequence of
numbers?
123456789246897531987654321
(1) 3 (2) 4 (3) 5 (4) 6
2. How many 5's are there in the following number sequence which are immediately preceded by 7 and immediately
followed by 6?
755945764598756764325678
(1) One (2) Two (3) Three (4) Four
3. How many 6's are there in the following number series, each of which is immediately preceded by 1 or 5 and
immediately followed by 3 or 9?
263756429613416391569231654321967163
(1) None (2) One (3) Two (4) Three
4. How many 7's are there in the following number series, each of which is immediately preceded by 6 but not
immediately followed by 4?
74276436753578437672406743
(1) One (2) Two (3) Four (4) Six
5. In the series given below, count the number of 9's, each of which is not immediately preceded by 5 but is
immediately followed by either 2 or 3. How many such 9's are there?
19265938393259293482698
(1) One (2) Three (3) Five (4) Six
Directions (Q.6 to Q.8) : Study the following number sequence and answer the questions given below it:
51473985726315863852243496
6. How many odd numbers are there in the sequence, each of which is immediately followed by an odd number?
(1) 1 (2) 2 (3) 3 (4) More than 4
7. How many even numbers are there in the sequence which are immediately preceded by an odd number but
immediately followed by an even number?
(1) 1 (2) 2 (3) 3 (4) 4

78 OVERSEAS www.allenoverseas.com
Mental Ability

8. How many odd numbers are there in the sequence which are immediately preceded and also immediately
followed by an even number?
(1) 1 (2) 2 (3) 3 (4) 4
9. The positions of the first and the fifth digits in the number 83256479 are interchanged. Similarly the positions
of the second and the sixth digits are interchanged and so on. Which of the following will be the third to the right
of the seventh digit from the right end after rearrangement?
(1) 3 (2) 4 (3) 7 (4) 8
10. If it is possible to make a number which is perfect square of a two-digit odd number with the second, the sixth
and the ninth digits of the number 187642539, which of the following is the digit in the unit's place of that two-
digit odd number?
(1) 1 (2) 7 (3) 9 (4) No such number can be made
Directions (Q. 11 to Q.13) : Answer these questions based on the set of numbers given below:
738 429 156 273 894
11. Which of the following will be the last digit of the second highest number after the positions of the digits in each
number is reversed?
(1) 1 (2) 2 (3) 4 (4) 7
12. What will be the difference between the first digit of the highest number as well as of the lowest number after the
positions of the first two digits in each number are reversed?
(1) 4 (2) 5 (3) 6 (4) 7
13. Which of the following will be the second digit of the third number from the top when they are arranged in
descending order, after the first digit in each number is changed to its next higher digit?
(1) 2 (2) 3 (3) 5 (4) 7
14. In the following series of numbers, how many 1, 3, 7 have appeared together. 7 being in middle and 1,3 on
either side of?
2973173771331738571377173906
(1) 4 (2) 5 (3) 3 (4) More than 5
15. How many such pairs of digits are there in the number 421579368 each of which has as many digits between
them in the number as when they are arranged in ascending order?
(1) None (2) One (3) Two (4) Three
Directions (Q. 16 to Q.18) : In each of the following questions, two rows of numbers are given. The resultant
number in each row is to be worked out separately based on the following rules and the questions below the rows
of numbers are to be answered. The operations of numbers progress from left to right.
Rules:
I. If an odd number is followed by a two-digit even number then they are to be added.
II. If an odd number is followed by a two-digit odd number then the second number is to be subtracted from
the first number.
III. If an even number is followed by a number which is a perfect square of a number then the second number
is to be divided by the first number.

www.allenoverseas.com OVERSEAS 79
Class-VIII

IV. If an even number is followed by a two-digit even number then the first number is to be multiplied by the
second number.
16. 13 11 4
17 13 12
If the resultant of the first set of numbers is multiplied by the resultant of the second set of numbers, what will be
the outcome?

(1) 48 (2) 69 (3) 75 (4) 96


17. 40 30 3600
15 24 17
What will be the sum of the two resultant numbers of the sets of numbers given above?
(1) 22 (2) 25 (3) 28 (4) 42
18. 15 11 20 400
8 12 10
If the resultant of the second set of numbers is divided by the resultant of the first set of numbers, what will be the
outcome?
(1) 19 (2) 92 (3) 196 (4) None of these

Qu e. 1 2 3 4 5 6 7 8 9 10
An s . 2 1 4 2 2 4 3 4 4 2
Qu e. 11 12 13 14 15 16 17 18
An s . 4 4 1 3 3 4 2 4

80 OVERSEAS www.allenoverseas.com
Mental Ability

12.2 Ranking test


In this type of questions, generally the ranks of a person both from the top and from the bottom are mentioned
and the total number of persons is asked. However, sometimes this question is -put in the form of a puzzle of
interchanging seats by two persons.
Solved Examples
Ex.1 Rohan ranks seventh from the top and twenty-sixth from the bottom in a class. How many students are there in
the class?
(1) 31 (2) 32 (3) 33 (4) 34
Sol. Clearly, the whole class consists of :
(i) 6 students who have ranks higher than Rohan;
(ii) Rohan; and
(iii) 25 students who have ranks lower than Rohan, i.e. (6 + 1 + 25) = 32 students.
Hence, the answer is (2).
Ex.2 Manik is fourteenth from the right end in a row of 40 boys. What is his position from the left end?
(1) 24th (2) 25th (3) 26th (4) 27th
Sol. Clearly, number of boys towards the left of Manik = (40 – 14) = 26.
So, Manik is 27th from the left end. Hence, the answer is (4).
Ex.3 In a row of boys facing the North, A is sixteenth from the left end and C is sixteenth from the right end. B, who
is fourth to the right of A, is fifth to the left of C in the row.
How many boys are there in the row?
(1) 39 (2) 40 (3) 41 (4) 42
Sol. Clearly, according to the given conditions, there are 15 boys to the left of A, as well as to the right of C. Also, B
lies between A and C such that there are 3 boys between A and B ; and 4 boys between B and C.
N

A B C
15 3 4 15
So, number of boys in the row = (15 + 1 + 3 + 1 + 4 + 1 + 15) = 40. Hence, the answer is (2).
Ex.4 In a row of 40 girls, when Komal was shifted to her left by 4 places her number from the left end of the row
became 10. What was the number of Swati from the right end of the row if Swati was three places to the right
of Komal's original position?
(1) 22 (2) 23 (3) 25 (4) None of these
Sol. On shifting 4 places to the left, Komal is 10 from the left end of the row. Thus, Komal's original position was
th

14th from the left end.


1 2 3 4 5 6 7 8 9 10 11 12 13 14 15 16 17 ............
K K S

Swati is 3 places to the right of Komal's original position. Clearly, Swati is 17 th from the left end.
Number of girls to the right of Swati = (40 – 17) = 23.
Thus, Swati is 24th from the right end of the row. Hence, the answer is (4).

www.allenoverseas.com OVERSEAS 81
Class-VIII

1. Sam ranked ninth from the top and thirty-eighth from the bottom in a class. How many students are there in the class?
(1) 45 (2) 46 (3) 47 (4) 48
2. A class of boys stands in a single line. One boy is nineteenth in order from both the ends. How many boys are
there in the class?
(1) 27 (2) 37 (3) 38 (4) 39
3. Ajay ranked sixteenth from the top and twenty-ninth from the bottom among those who passed an examination.
Six boys did not participate in the competition and five failed in it. How many boys were there in the class?
(1) 40 (2) 44 (3) 50 (4) 55
4. In a row of boys, Jeevan is seventh from the start and eleventh from the end. In another row of boys, Vikas is
tenth from the start and twelfth from the end. How many boys are there in both the rows together?
(1) 36 (2) 37 (3) 39 (4) None of these
5. In a class of 60, where girls are twice that of boys, Kapil ranked seventeenth from the top. If there are 9 girls
ahead of Kapil, how many boys are after him in rank?
(1) 3 (2) 7 (3) 12 (4) 23
6. Nitin ranks eighteenth in a class of 49 students. What is his rank from the last?
(1) 18 (2) 19 (3) 31 (4) 32
7. Manoj and Sachin are ranked seventh and eleventh respectively from the top in a class of 31 students. What will
be their respective ranks from the bottom in the class?
(1) 20th anld 24th (2) 24th and 20th (3) 25th and 21st (4) 26th and 22nd
8. Bharati is 8 ranks ahead of Divya who ranks twenty-sixth in a class of 42. What is Bharati's rank from the last?
(1) 9th (2) 24th (3) 25th (4) 34th
9. In a row of boys, A is thirteenth from the left and D is seventeenth from the right. If in this row A is eleventh from
the right then what is the position of D from the left?
(1) 6th (2) 7th (3) 10th (4) 12th
10. In a row of 40 children, P is thirteenth from the left end and Q is ninth from the right end. How many children
are there between P and R if R is fourth to the left of Q? '
(1) 12 (2) 13 (3) 14 (4) 15

11. In a class Vidya ranks 7th from the top. Divya is 7 ranks ahead of Medha and 3 ranks behind Vidya. Sushma who
is 4th from the bottom, is 32 ranks behind Medha. How many students are there in the class?

(1) 52 (2) 49 (3) 50 (4) None of these

12. Students line up in a queue in which Ashish stands fifteenth from the left and Sachin is seventh from the right. If
they interchange their places, Sachin would be fifteenth from the right. How many students are there in the
queue?

(1) 21 (2) 22 (3) 29 (4) None of these

82 OVERSEAS www.allenoverseas.com
Mental Ability

Directions (Q.13 to Q.16) : Read the following information to answer these questions:
Consider a group comprising of 4 students - Reena, Beena, Meena and Neena, who stand in a row. Reena and
Beena stand in sixth and seventh positions respectively from the left. Meena and Neena stand in the fourth and
fifth positions respectively from the right. When Beena and Meena exchange their positions, then Beena will be
fifteenth from the left.
13. Originally, Neena's position from the left is
(1) 5 (2) 13 (3) 14 (4) 16
14. Reena's position from the right is
(1) 6 (2) 13 (3) 14 (4) 18
15. If Neena and Reena also exchange their positions between themselves, then after the exchange, Neena's
position from the left will be
(1) 6 (2) 10 (3) 11 (4) None of these
16. After exchange of positions between Beena and Meena, Meena's position from the right is
(1) 5 (2) 10 (3) 12 (4) None of these
17. If Atul finds that he is twelfth from the right in a line of boys and fourth from the left, how many boys should be
added to the line such that there are 28 boys in the line?
(1) 12 (2) 13 (3) 14 (4) 20

18. In a queue of boys Sohan is 9th from the back. Ramesh's place is 8th from the front. Radhey is standing in the
middle of the two. What could be the minimum number of boys standing in the queue?

(1) 8 (2) 10 (3) 12 (4) 14

Qu e. 1 2 3 4 5 6 7 8 9 10
An s . 2 2 4 4 3 4 3 3 2 3
Qu e. 11 12 13 14 15 16 17 18
An s . 1 3 3 2 1 3 2 2

www.allenoverseas.com OVERSEAS 83
Class-VIII

12.3 Time sequence test


Ex.1. Satish remembers that his brother's birthday is after fifteenth but before eighteenth of February whereas his
sister Kajal remembers that her brother's birthday is after sixteenth but before nineteenth of February. On which
day in February is Satish's brother's birthday?

(1) 16th (2) 17th (3) 18th (4) 19th

Sol. According to Satish, the brother's birthday is on one of the days among 16th and 17th February.
According to Kajal, the brother's birthday is on one of the days among 17th and 18th February.

Clearly, Satish's brother's birthday is on the day common to both the above groups, i.e., 17th February.
Hence, the answer is (2).

Ex.2 A bus for Delhi leaves every thirty minutes from a bus stand. An enquiry clerk told a passenger that the bus had
already left ten minutes ago and the next bus will leave at 9:35 a.m. At what time did the enquiry clerk give this
information to the passenger?

(1) 9:10 a.m. (2) 8:55 a.m. (3) 9:08 a.m. (4) 9:15 a.m.

Sol. The next bus will leave at 9:35 a.m. This means that the previous bus had left at 9:05 a.m.

But it happened ten minutes before the clerk gave the information to the passenger.
Thus, the enquiry clerk gave the information at 9:15 a.m.

Hence, the answer is (4).

Ex.3 If the seventh day of a month is three days earlier than Friday, what day will it be on the nineteenth day of the
month?

(1) Sunday (2) Monday (3) Wednesday (4) Friday


Sol. As mentioned, the seventh day of the month is three days earlier than Friday, which is Tuesday.

So, the fourteenth day is also Tuesday and thus, the nineteenth day is Sunday.

Hence, the answer is (1).

1. Kailash remembers that his brother Deepak's birthday falls after 20th May but before 28th May, while Geeta
remembers that Deepak's birthday falls before 22nd May but after 12th May. On what date Deepak's birthday
falls?
(1) 20th May (2) 21st May (3) 22nd May (4) Cannot be determined
2. Sangeeta remembers that her father's birthday was certainly after eighth but before thirteenth of December.
Her sister Natasha remembers that their father's birthday was definitely after ninth but before fourteenth of
December. On which date of December was their father's birthday?
(1) 10th (2) 11th (3) 12th (4) Data inadequate

84 OVERSEAS www.allenoverseas.com
Mental Ability

3. Vikas left home for the bus stop 25 minutes earlier than usual. It takes 20 minutes to reach the stop. He
reached the stop at 10:30 a.m. What time does he usually leave home for the bus stop?

(1) 10:30 a.m. (2) 10:35 a.m. (3) 10:55 a.m. (4) None of these

4. The priest told the devotee, "The temple bell is rung at regular intervals of 45 minutes. The last bell was rung
five minutes ago. The next bell is due to be rung at 7.45 a.m." At what time did the priest give this information
to the devotee?

(1) 7.40 a.m. (2) 7.05 a.m. (3) 7.00 a.m. (4) 6.55 a.m.

5. There are twenty people working in an office. The first group of five works between 8:00 A.M. to 2:00 P.M.
The second group of ten works between 10:00 A.M. to 4:00 P.M. And the third group of five works between 12
noon to 6:00 P.M. There are three computers in the office which all the employees frequently use. During which
of the following hours the computers are likely to be used most?

(1) 10:00 AM. to 12 noon (2) 12 noon to 2:00 P.M.

(3) 1:00 P.M. to 3:00 PM. (4) 2:00 PM. to 4:00 PM.

6. A monkey climbs 30 feet at the beginning of each hour and rests for a while when he slips back 20 feet before
he again starts climbing in the beginning of the next hour. If he begins his ascent at 8:00 a.m., at what time will
he first touch a flag at 120 feet from the ground?

(1) 4 p.m. (2) 5 p.m. (3) 6 p.m. (4) None of these

7. If day after tomorrow is Saturday, what day was three days before yesterday?

(1) Sunday (2) Monday (3) Thursday (4) Saturday


8. If February 1, 2004 is Wednesday, what day is March 3, 2004?
(1) Monday (2) Sunday (3) Saturday (4) Friday
9. If the first day of the year (other than the leap year) was Friday, then which was the last day of that year?
(1) Monday (2) Friday (3) Saturday (4) Sunday
10. How many days will there be from 26th January 2004, to 15th May, 2004 (both days included)?
(1) 110 (2) 111 (3) 112 (4) 113

Que. 1 2 3 4 5 6 7 8 9 10
Ans . 2 4 2 2 2 3 1 3 2 2

www.allenoverseas.com OVERSEAS 85
Class-VIII

Important Notes

86 OVERSEAS www.allenoverseas.com
Mental Ability

CHAPTER 13 ALPHABET TEST

Alphabetical order: Arranging words in alphabetical order implies 'to arrange them in the order as they
appear in a dictionary', i.e., as per the order in which the beginning letters of these words appear in the English
alphabet.
How to arrange the words in alphabetical order?
First consider the first letter of each word. Arrange the words in the order in which these letters appear in the
English alphabet.
Example : Consider the words - Abstract, Principle, Marry, Spring, Frequent.
These words begin with letters A, P, M, S, F respectively.
Their order in English alphabet is A, F, M, P, S.
Thus, the correct alphabetical order of these words is :
Abstract, Frequent, Marry, Principle, Spring
In some cases, two or more words begin with the same letter. Such words should be arranged in the order of
second letters in the alphabet.
Example : Consider the words - Client, Castle, Face, Viper, Dazzle.
Here, as in the above example, the words can be arranged as :

Cli ent ü
ý , Dazzle, Face, Viper..
Castle þ

What remains now is how to arrange 'Client' and 'Castle'. Consider their second letters, i,e., l and a. Arranging
these words accordingly, 'Castle' comes before 'Client'.
Thus, the correct alphabetical order is :
Castle, Client, Dazzle, Face, Viper.
If both the first and second letters of two or more words are the same, arrange these words, considering their
third letters and so on.
Solved examples
Ex.1 If the following four words are arranged in alphabetical order, which word will come at the third place?
(1) Draw (2) Read (3) Play (4) Back
Sol. Arranging the words in alphabetical order, we have: Back, Draw, Play, Read.
Clearly, the word 'Play' comes in the third. Hence, the answer is (3).
Ex.2 Which of the following words will come fourth in the English dictionary?
(1) False (2) Follow (3) Faithfully (4) Fool
Sol. The given words can be arranged in the alphabetical order as :
Faithfully, False, Follow, Fool
Clearly, 'Fool' comes fourth. Hence, the answer is (4).

www.allenoverseas.com OVERSEAS 87
Class-VIII

Ex.3. Arrange the following words according to dictionary arrangement :


1. Epitaxy 2. Episode 3. Epigene 4. Epitome 5. Epilogue
(1) 1, 2, 3, 4, 5 (2) 3, 2, 5, 4, 1

(3) 3, 5, 2, 1, 4 (4) 5, 4, 2, 1, 3
Sol. Clearly, the correct alphabetical order of the given words is :
Epigene, Epilogue, Episode, Epitaxy, Epitome

Thus, the correct sequence is 3, 5, 2, 1, 4. Hence, the answer is (3).


Ex.4 How many such pairs of letters are there in the word INSTRUCTION which have as many letters between them
in the word as in the English alphabet?
(1) One (2) Two (3) Three (4) Four

Sol. Clearly, we have :

OPQRS

INSTRUCTION

HGFED
As depicted above, C and I have five letters between them in the given word as well as in English alphabet; N
and T again have five letters between them and each of the pairs (S and T) and (N and O) have no letter between
them.
Thus, there are four such pairs. Hence, the answer is (4).

Ex.5. How many D's are there in the following series which are immediately followed by W but not immediately
preceded by K?

KDCWKDWNKGDWWDHKVDWZDW

(1) Nil (2) One (3) Two (4) Three


Sol. Clearly, D's satisfying the given conditions can be marked as under:

KDCWKDWNKG D WWDHKV D WZ D W

We observe that such D's are three in number. Hence, the answer is (4).

1. Three of the following groups of letters are like is some way while one is different. Find out which one is different?
(1) ISLOJ (2) AKDGB (3) LUOQM (4) FPILG

2. If the English alphabets are divided into two equal halves from A to M and N to Z, which letter in the later half
would be corresponding to letter J?
(1) Q (2) W (3) X (4) V

88 OVERSEAS www.allenoverseas.com
Mental Ability

3. In the given question, arrange the given words in alphabetical order as in dictionary and choose the one that
comes first.
(1) Guarantee (2) Group (3) Groan (4) Guard
Direction (Q.4 & Q.5) : In each of the following questions one of the words given in the alternatives cannot be
formed by using the letters given in the question. Find out that word.
4. ENCOURAGE
(1) USAGE (2) GREEN (3) ANGER (4) COURAGE
5. CONCENTRATE
(1) TREAT (2) CONCERN (3) TRAIN (4) CENTRE
6. If in the word ELECTROCARDIOGRAPH' the first half of the letters are reversed, second letter from left end is
then prefixed and finally 'S' is suffixed,, then the letter exactly in the middle is _____.
(1) L (2) R (3) D (4) E
7. If the following words are arranged in the alphabetical order, which word will appear at the end?
(1) Olympic (2) Olympia (3) Oval (4) Ovulet
8. In the following alphabet, which letter is fifteenth of the right of the letter which is third to the left of J?
ABCDEFGHIJKLMNOPQRSTUVWXYZ
(1) V (2) U (3) S (4) None of these
9. Two letters in the word 'B O X E S' have as many letters between them as in the alphabet, Which one of those
two comes first in the alphabet?
(1) O (2) B (3) X (4) E
10. If English alphabets are written in reverse order what will be seventh letter to the right of the 12th letter from the
left?
(1) F (2) H (3) S (4) R
11. How many L's are there which do not have R preceding them and also do not have T following them?
ZQSTLRMNQNRTUVXRLTASLTQRSL T
(1) 1 (2) 2 (3) 3 (4) 4
12. What should come next in the letter series given below:
A A B A B C A B C D A B C D E A B C D E F A B C D E F G A B C D E F G __________.
(1) A (2) I (3) H (4) B
13. If the following words are arranged in an alphabetical order, which word will appear in the last ?
(1) Principal (2) Principle (3) Principia (4) Principled
14. How many pairs of letters are there in the word CARROT which have as many letters between them in the word
as in the alphabet?
(1) 1 (2) 2 (3) 3 (4) 4
15. How many pairs of letters are there in the word HORIZON which have as many letters between them in the
word as in the English alphabet?
(1) One (2) Two (3) Three (4) More than three

www.allenoverseas.com OVERSEAS 89
Class-VIII

16. If the English alphabets is written in the reverse order and every alternate letter starting with Y is dropped
towards the right, which letter will be exactly in the middle of the remaining letters of the alphabet?
(1) M (2) O (3) N (4) M or O
17. Which letter should be ninth letter to the left of ninth letter from the right end, if the first half of the English
alphabet is reversed?
(1) I (2) F (3) D (4) E
18. If the order of letters of each of the following words is reversed, then which of the following will be the meaningful
word? If more than one such word can be formed, mark 'S' as the answer and if no such word can be formed,
mark 'X' as the answer.
NAIL, PAIL, RAIL, MADAM, REST
(1) PAIL (2) X (3) MADAM (4) S
19. If it is possible to make meaningful word with the second, third, fifth and eighth letters of the word
PARAGRAPH which of the following will be second letter of that word? If more than one such word can be
made, give X as the answer. If no such word can be made, give M as the answer.
(1) X (2) M (3) G (4) P
20. Number of letters skipped in between adjacent letters in the series decreases by two. Which of the following
series observes this rule?
(1) FQWBG (2) HQXCF (3) TBINO (4) XFMQU
21. Which letter should be fourth to the left of twelfth letter from the right if the second half of the alphabet series
is reversed ?
(1) J (2) K (3) L (4) M
22. If it is possible to make a meaningful word with the second, the fourth, the fifth, the seventh and the eleventh
letters of the word DISTRIBUTION which of the following will be the third letter of that word ? If no such word
can be formed give X as answer.
(1) O (2) I (3) B (4) X
23. If the alphabets were written in the reverse order, which letter will be the fifth letter to the right of the fourteenth
letter from the left.
(1) R (2) I (3) S (4) H
24. Which letter will come exactly between the 10th letter from your left and the 7th letter from your right, without
changing any order in the original from the alphabet?
(1) N (2) P (3) O (4) Q
25. If it is possible to form a word with the first, fourth, seventh and eleventh letters in the word 'SUPERFLUOUS'
write the second letter of that word. Otherwise, X is the answer.?
(1) S (2) E (3) L (4) X
26. In the alphabetical series second half is written first and then the first half is arranged in reverse order. Then
which of the following will be the 5th letter to the left of 16th letter from the right?
(1) S (2) T (3) P (4) F
27. If 1st and 7th, 2nd and 8th, 3rd & 9th letters are interchanged in the word TRANSLATION. What will be the fifth
letter to the right of 9th letter from the right?
(1) T (2) L (3) R (4) S

90 OVERSEAS www.allenoverseas.com
Mental Ability

28. From the word 'CARRIAGE', how many independent words can be made without changing the order of the
letter only once.
(1) One (2) Two (3) Three (4) Four
29. If the first and second letters in the word 'EXCELLENCE' are interchanged, in the sameway third and fourth
letters, the fifth and the sixth letters and so on. Which letter would be the sixth when counted from the right?
(1) C (2) D (3) E (4) L
30. From the word 'POSTSCRIPT', how many independent words can be made without changing the order of the
letters and using each letters only once?
(1) 1 (2) 2 (3) 3 (4) 4
31. Which letter will be 13th to the left of 11th letter from the right in the following series of letters?
WQETRYIPUOAGSDFHLKJBCXMVZN
(1) Y (2) F (3) E (4) V
32. Which of the following words cannot be formed with the help of the letters of 'EXPERIENCE'?
(1) PRICE (2) EXERCISE (3) RICE (4) NICE
33. If you count 21 letters in the English alphabet from the end and 20 letters from the beginning. Which letter will
appear exactly in the middle of the sequence thus formed?
(1) N (2) M (3) L (4) K
34. Which letter is the 9th to the right of the 15th letter from your left in the English Alphabet series?
(1) U (2) V (3) X (4) W
35. If you count 17 letters in the English alphabet from the end and 14 letters from the begining. Which letter will
appear exactly in the middle of the sequence thus formed?
(1) K (2) L (3) M (4) N

Que. 1 2 3 4 5 6 7 8 9 10 11 12 13 14 15
Ans. 3 2 3 1 3 2 4 1 2 2 3 3 4 1 4
Que. 16 17 18 19 20 21 22 23 24 25 26 27 28 29 30
Ans. 3 4 4 2 2 2 3 4 3 2 1 3 2 4 2
Que. 31 32 33 34 35
Ans. 3 2 2 3 2

www.allenoverseas.com OVERSEAS 91
Class-VIII

Important Notes

92 OVERSEAS www.allenoverseas.com
Mental Ability

CHAPTER 14 DICE & CUBE

14.1 Dice
Dice is a three dimensional figure with all of its surfaces numbered. We are giving below few properties of Dice
which will help candidates to solve various problems on Dice.
1. A dice has six surfaces and all of them are numbered from 1 to 6.
2. If the surfaces of dice are unfolded and placed on a plane, the figure of dice so obtained will look like one of the
following figures:
Form I In this case
1
1 lies opposite 5 ;
2 3 4
2 lies opposite 4 ;
5
6 3 lies opposite 6.

Form II In this case

1 2 1 lies opposite 6 ;
3
2 lies opposite 4 ;
4
5 6 3 lies opposite 5.

Form III In this case

1 1 lies opposite 4 ;
2 3
2 lies opposite 6 ;
4
5 6 3 lies opposite 5.

Form IV In this case

1 1 lies opposite 4 ;
2 3 2 lies opposite 5:
4 5
3 lies opposite 6.
6

www.allenoverseas.com OVERSEAS 93
Class-VIII

Form V In this case


1
1 lies opposite 3 ;
2
3 4 2 lies opposite 5 ;
5
6 4 lies opposite 6.

Form VI In this case

+
+ – =
× will be the one of the faces of the cube and it lies opposite 3;
1
– 2 2 lies opposite 4 ;
3 4 =
5
× 1 lies opposite 5.

Form VII In this case

– +
+ ×= will be the one of the faces of the cube and it lies opposite 3;
1
– 2 lies opposite 4 ;
2 3 4 =
5 1 lies opposite 5.
×
Form VIII In this case

1
=
and – × are two faces of the cube that lie opposite to each other..
+ +

– 2
1 lies opposite 3 ;
3 =
× 2 lies opposite 4 ;
4
Solved examples
Category–I
Ex.1 A dice has been thrown four times and produces following results. Which number will appear opposite to the number 3?
6 5 3 2
1 4 1 6
3 2 5 3

(i) (ii) (iii) (iv)


(1) 4 (2) 5 (3) 6 (4) 1
Sol. From the figures (i), (ii) and (iv), we find that numbers 6, 1, 5 and 2 appear on the adjacent surfaces to the
number 3. Therefore, number 4 will be opposite to number 3. Hence, the answer is (1).

94 OVERSEAS www.allenoverseas.com
Mental Ability

Category–II
Ex.2 The figures given below show the two different positions of a dice. Which number will appear opposite to
number 2?
4 5
2 6
3 4

(i) (ii)
(1) 3 (2) 4 (3) 5 (4) 6
Sol. The above question, where only two positions of a dice are given, can easily be solved with the following method.

6
IV

5 4 2
V I III

3
II

1
VI

Fig. X
Step-I The dice, when unfolded, will appear as shown in Fig.X
Step-II Write the common number of both the dice in the middle block. Since common number is 4, hence
number 4 will appear in the central block.
Step-III Consider the Fig. (i) and write the first number in the anti-clockwise direction of number 4, (common
number) in block I and second number in block II. Therefore, numbers 3 and 2 being the first and
second number to 4 in anti-clockwise directions respectively, will appear in blocks I and II, respectively.
Step-IV Consider Fig. (ii) and write first and second number in the anti-clockwise direction to number 4,
(common number) in block III and IV. Hence, numbers 6 and 5 will appear in the blocks III and IV,
respectively.
Step-V Write the remaining number in the remaining block. Therefore, number 1 will come in the remaining
block.
Now, from the unfolded figures we find that number opposite to 6 is 3, number opposite to 2 is 5 and number
opposite to 4 is 1. Hence, the answer is (3).
Category–III
Ex.3 From the following figures of dice, find which number will come in place of '?'.

4 5 1
3 2 ?
2 1 6

(i) (ii) (iii)

(1) 4 (2) 5 (3) 2 (4) 3

www.allenoverseas.com OVERSEAS 95
Class-VIII

Sol. If the above dice is unfolded, it will look like as Fig-I. given below. (Candidate should follow the methods as
explained in the previous example to find the appropriate place of the numbers appearing on the different
surfaces of the dice in the above Fig.)
Now the number in place of '?' can be obtained by making a slight change in the figure as given here.
Now comparing Fig-I. with Fig. (iii) above, we get that number in place of? is 3. Hence, the answer is (4).
4 4
5 2 3 5 2
1 1 3 1 3
6 6
6
(I) (II)
Category–IV
Ex.4 Which of the following dices is identical to the unfolded figure as shown here?

2
5 1
4 3
6

4 1 2 3
3 5 4 4
6 3 6 2

(1) (2) (3) (4)


Sol. From the unfolded figure of dice, we find that number opposite to 2 is 4, for 5 it is 3 and for 1 it is 6. From this
result we can definitely say that figure (2), (3) and (4) can't be the answer figure as numbers lying on the opposite
pair of surfaces are present on the adjacent surfaces. Hence, figure (1) is the answer.

96 OVERSEAS www.allenoverseas.com
Mental Ability

Directions (Q. 1 to Q.5) : In each of the following questions, four positions of the same dice have been shown. You
have to see these figures and select the number opposite to the number as asked in each of the question.
1. Which number is on the opposite surface of number 3?

3 3 2 4
6 1 1 6
5 6 3 1

(i) (ii) (iii) (iv)

(1) 4 (2) 6 (3) 5 (4) 1


2. Which number is opposite to number 5?

3 2 5 6
4 1 4 3
5 6 2 5

(i) (ii) (iii) (iv)


(1) 6 (2) 5 (3) 1 (4) 3
3. Which number is opposite to number 2?

6 4 5 6
4 3 2 5
2 2 3 1

(i) (ii) (iii) (iv)


(1) 4 (2) 6 (3) 1 (4) 3
4. Which letter will be opposite to letter D?

E C D B
C E B A
A D C C

(i) (ii) (iii) (iv)


(1) A (2) B (3) E (4) F
5. Which symbol will come opposite to symbol ¸
–·· –··
= D D =
× = –·· D

(i) (ii) (iii) (iv)


(1) (2) = (3) × (4) D

www.allenoverseas.com OVERSEAS 97
Class-VIII

6. Which number will come opposite to number 2?

5 4
2 2
1 6

(i) (ii)

(1) 5 (2) 1 (3) 6 (4) 3


7. Which will be the number at the bottom, if 5 is at the top?

1 6
2 3
4 2

(i) (ii)

(1) 1 (2) 2 (3) 3 (4) 6


8. All the surfaces of the dice contain different numbers in the form of dots. Consider both the figures of dice and
tell how many dots are contained on the face opposite to the face that contains four dots?

(i) (ii)
(1) 2 (2) 3 (3) 5 (4) 6
9. What is the number of dots on the face opposite to the face that contains 2 dots?

(i) (ii)
(1) 1 (2) 3 (3) 4 (4) 6
10. Which alphabet will come on the opposite surface on which alphabet F appears?
C E D
B C A
A D F

(i) (ii) (iii)


(1) B (2) A (3) C (4) E
11. The different positions of a dice has been shown. What digit will be opposite to digit 4?

3 4 2
4 5 3 2 3 1

(1) 6 (2) 1 (3) 5 (4) 2

98 OVERSEAS www.allenoverseas.com
Mental Ability

12. A diagram has been given which can be folded into a cube. The entries given in the squares indicate the
entries on the face of the cube. In the given question letters has been given. of the four alternative given
below it, you have to find the one that would appear on the face opposite to it in the cube.
Which letter is opposite R?

Q R
S P
U T

(1) P (2) S (3) T (4) U


Directions (Q.13 to Q.15): In each of the following questions, the second die is obtained by rotating the
first die horizontally to right by an angle of 90°. Observe the dice and answer the questions.
13. The faces of this die show numbers 1. 2. 3. 4, 5 and 6. Number on the face opposite to the face showing
4 is

2 2
3 4 5 3

(1) 1 (2) 6 (3) 5 (4) 3


14. The faces of this die show symbols D, #, *, X, p, and O. Symbol on the face opposite to the face showing
o is

D D
o
* # *

(1) A (2) O or X (3) # (4) X


15. The faces of this die show 1, 2, 3, 4, 5 and 6 dots. Number of dots on the face opposite to the face showing
1 dot is

(1) 4 or 6 (2) 4 (3) 6 (4) 2

www.allenoverseas.com OVERSEAS 99
Class-VIII

Directions (Q.16 & Q.17): In each of the following questions, the second die is obtained by rotating the
first die horizontally to left by an angle of 90°. Observe the dice and answer the questions.
16. The faces of this die show numbers 1, 2, 3, 4, 5 and 6. Number on the face opposite to the face showing
2 is

1 1
2 3 3 4

(1) 3 (2) 6 (3) 4 (4) 5


17. The faces of this die show letters A, B, C, D, E and F Letter on the face opposite to the face showing the
letter A is

C C
B A E
A

(1) E (2) D (3) F (4) D or F


Directions (Q.18 to Q.20) : In each of the following questions, the second die is obtained by rotating the
first die vertically to right by an angle of 90°. Observe the dice and answer the questions.
18. The faces of this die show numbers 1. 2, 3. 4. 5 and 6. Number on the face opposite to the face showing
1 is

1
2 3 2
5 3

(1) 4 (2) 6 (3) 5 (4) 2


19. The faces of this die show letters A. B, C. D, E and F. Letter on the face opposite to the face showing the
letter B is

A B
B C
D C

(1) E or F (2) E (3) F (4) C

100 OVERSEAS www.allenoverseas.com


Mental Ability

20. The faces of this die show 1, 2, 3, 4, 5 and 6 dots. Number of dots on the face opposite to the face showing
6 dots is

(1) 5 (2) 1 or 5 (3) 1 (4) 4

Q u e. 1 2 3 4 5 6 7 8 9 10 11 12 13 14 15
Ans. 1 3 3 1 1 4 2 4 1 3 2 2 3 3 1
Q u e. 16 17 18 19 20
Ans. 3 4 3 1 2

www.allenoverseas.com OVERSEAS 101


Class-VIII

Direction (Q.1 to Q.4) : Choose the box that is similar to the box formed from the given sheet of paper (X).

1.

(X) (1) (2) (3) (4)

(1) 1 and 2 only (2) 2 and 4 only (3) 2 and 3 only (4) 1 and 4 only

2.

(X) (1) (2) (3) (4)

(1) 1 and 4 only (2) 3 and 4 only (3) 1 and 2 only (4) 2 and 3 only

F
3. A E
B
C D F F E D
E D
B E BC E
A

(X) (1) (2) (3) (4)


(1) 1 only (2) 2 only (3) 1 and 3 only (4) 1, 2, 3 and 4

4. +

+ +

(X) (1) (2) (3) (4)

(1) 1 only (2) 2 only (3) 3 only (4) 4 only


5. How many dots lie opposite to the face having three dots, when the given figure is folded to form a cube?

(1) 2 dots

(2) 3 dots

(3) 4 dots

(4) 6 dots

102 OVERSEAS www.allenoverseas.com


Mental Ability

6. In the following question, the figure is folded to form a box. Choose from among the alternatives the
box or boxes that can be formed by folding the figure.

X
I. O X II. O X

O X
III. O IV.
O

(1) Both II and IV (2) Both I and II (3) IV only (4) II only
7. Choose from the given options, the box that will be formed when figure is folded.

(1) (2) (3) (4)

Direction (Q.8 to Q.10) : Choose the box that is similar to the box formed from the given sheet of paper (X).

8.

(X) (1) (2) (3) (4)


(1) 1 and 2 only (2) 2 and 3 only (3) 1 and 4 only (4) 1, 2, 3 and 4

9. =

= =
(X) (1) (2) (3) (4)
(1) 1 only (2) 1 and 3 only (3) 1, 3 and 4 only (4) 1, 2, 3 and 4

www.allenoverseas.com OVERSEAS 103


Class-VIII

10.

(X) (1) (2) (3) (4)

(1) 2 and 3 only (2) 1, 3 and 4 only (3) 2 and 4 only (4) 1 and 4 only

Que. 1 2 3 4 5 6 7 8 9 10
Ans. 3 1 2 2 4 3 1 4 3 2

104 OVERSEAS www.allenoverseas.com


Mental Ability

14.2 Cube
Ex.1 Count the number of blocks in the given solid.

(1) 24 (2) 36 (3) 48 (4) 60


Sol. Each layer (say top most) of the solid contains 12 blocks.
There are 3 identical layers in all.
So, the total number of blocks = 3 × 12=36
Hence, the answer is (2).
It is necessary to know basic features of a cubical figure to solve a problem on cube. In the following steps we
have listed out basic features of a cube and all probable situations on which questions on it can be asked in any
examination.
A cube is three dimensional figure, having 8 corners, 6 surfaces and 12 edges. If a cube is painted on all
of its surfaces with any colour and further divided into various smaller cubes, we get following results :
Corner

Edge

Surface
Smaller cubes with three surfaces painted will be present on the corners of the big cube.
Smaller cubes with two surfaces painted will be present on the edges of the big cube.
Smaller cubes with one surface painted will be present on the surfaces of the big cube.
Smaller cubes with no surface painted will be present inside the big cube.

Type-I
When a cube is painted on all of its surfaces with a single colour and then divided into various smaller cubes
of equal size :
If a cube is painted on all of its surfaces with a colour and then divided into smaller cubes of equal size,
then after separation, number of smaller cubes so obtained will be calculated as under :

www.allenoverseas.com OVERSEAS 105


Class-VIII

Cubes with three surface painted


Cubes with two surface painted
Cubes with one surface painted
No. of smaller cubes with three surfaces painted = 8
No. of smaller cubes with two surfaces painted = (n – 2) × 12
No. of smaller cubes with one surface painted = (n – 2)2 × 6
No. of smaller cubes with no surface painted = (n – 2)3
length of edge of big cube
where, n = No. of divisions on the surface of the bigger cube =
length of edge of one smaller cube
Minimum cuts required = (n–1) × 3
Direction (Ex.2 to Ex.6) : A cube is coloured red on all faces. It is cut into 64 smaller cubes of equal
size. Now, answer the following questions based on this statement?
Explanation :

The following figure shows the cube coloured red on all faces, and divided into 64 smaller cubes :

The figure may be analysed by dividing it into four horizontal layers :


In layer I : The four central cubes have only one face coloured, four cubes at the corner have three faces
coloured and the remaining 8 cubes have two faces coloured.
In each of the layer II and III : The four central cubes have no face coloured, the four cubes at the corner have
two faces coloured and the remaining 8 cubes have only one face coloured.

In layer IV : The four central cubes have only one face coloured, four cubes at the corner have three faces
coloured and the remaining 8 cubes have two faces coloured.
Ex.2 How many minimum cuts are required to cut the cube into smaller cubes of 1 cm
(1) 12 (2) 9 (3) 6 (4) 3
Sol. Minimum cuts required = (n–1) × 3
Minimum cuts required = (4–1) × 3 = 9
Hence, the answer is (2).
Ex.3 How many cubes have no face coloured?
(1) 24 (2) 16 (3) 8 (4) 0
Sol. No. of smaller cubes with no surface painted = (n – 2) 3 .
No. of smaller cubes with no surface painted = (4 – 2) 3 .
Thus there are 8 such cubes. Hence, the answer is (3).

106 OVERSEAS www.allenoverseas.com


Mental Ability

Ex.4 How many cubes are there which have only one face coloured?
(1) 4 (2) 8 (3) 16 (4) 24
Sol. No. of smaller cubes with one surface painted = (n – 2) 2 × 6
No. of smaller cubes with one surface painted = (4 – 2)2 × 6
Thus, 24 such cubes. Hence, the answer is (4).
Ex.5 How many cubes have two red opposite face?
(1) 0 (2) 8 (3) 16 (4) 24
Sol. None of the cubes can have its opposite faces coloured red. Hence, the answer is (1).
Ex.6 How many cubes have three faces coloured?
(1) 24 (2) 16 (3) 8 (4) 4
Sol. Four corner cubes in layer I and four corner cubes in layer IV have three faces coloured. Thus there are 8 such
cubes. Hence, the answer is (3).
Type II
If a cube is painted on all of its surfaces with different colours and then divided into various smaller cubes
of equal size.
Direction : A cube of side 4 cm is painted black on one of the pairs of opposite surfaces, blue on another
pair of opposite surfaces and red on the remaining pair of opposite surfaces. The cube is now divided into
smaller cubes of equal side of 1 cm each.
1. No. of smaller cubes with three surfaces painted = 8
(These smaller cubes will have all three surfaces painted with different colours — blue, black and red).
2. No. of smaller cubes with two surfaces painted = 24.
(a) No. of cubes with two surfaces painted with black and blue colours = 8.

Blue
Red
Black

(b) No. of cubes with two surfaces painted with blue and red colours = 8.

(c) No. of cubes with two surfaces painted with black and red colours = 8.

3. No. of smaller cubes with one surface painted = 24.

and out of this —

(a) No. of cubes with one surface painted with black colour = 8.

(b) No. of cubes with one surface painted with blue colour = 8.

(c) No. of cubes with one surface painted with red colour = 8.

www.allenoverseas.com OVERSEAS 107


Class-VIII

Direction : A cube of side 4 cm is painted black on one of the pairs of adjacent surfaces, blue on another
pair of adjacent surfaces and red on the remaining pair of adjacent surfaces. The cube is now divided into
smaller cubes of equal side of 1 cm each.

1. No. of smaller cubes with three surfaces painted = 8

(These smaller cubes will have all three surfaces painted with different colours — blue, black and red).

2. No. of smaller cubes with two surfaces painted = 24.


(a) No. of cubes with two surfaces painted with black and blue colours = 8

Blue
Red
Black

(b) No. of cubes with two surfaces painted with blue and red colours = 8
(c) No. of cubes with two surfaces painted with black and red colours = 8
3. No. of smaller cubes with one surface painted = 24.
and out of this —
(a) No. of cubes with one surface painted with black colour = 8
(b) No. of cubes with one surface painted with blue colour = 8
(c) No. of cubes with one surface painted with red colour = 8

Direction (Q.1 to Q.5) : Count the number of cubes in the given solid in each of the following questions.

1.

(1) 8 (2) 7 (3) 6 (4) 5

2.

(1) 12 (2) 11 (3) 10 (4) 15

108 OVERSEAS www.allenoverseas.com


Mental Ability

3.

(1) 10 (2) 15 (3) 18 (4) 21

4.

(1) 15 (2) 13 (3) 16 (4) 14

5.

(1) 27 (2) 22 (3) 25 (4) 24

6. Count the number of cubes in the following figure.

(1) 30 (2) 32 (3) 31 (4) 33


7. How many unit cubes are there in the figure?

(1) 52 (2) 53 (3) 54 (4) 56

www.allenoverseas.com OVERSEAS 109


Class-VIII

8. Number of unit cubes in the given figure is_____.

(1) 25 (2) 26 (3) 27 (4) 28


9. The minimum number of colours required to paint all the sides of a cube that no two adjacent faces may
have the same colours is
(1) 1 (2) 2 (3) 3 (4) 4
Direction (Q.10 to Q.14) : A cube is painted blue on all faces and then cut into 125 cubes of equal size.
Answer the following questions based on this statements.

10. How many cubes are not painted on any face?

(1) 8 (2) 16 (3) 27 (4) 36

11. How many cubes are painted on one face only?

(1) 16 (2) 32 (3) 48 (4) 54

12. How many cubes have only two faces painted?

(1) 12 (2) 24 (3) 36 (4) 48

13. How many cubes have atleast two faces painted?

(1) 24 (2) 36 (3) 40 (4) 44

14. How many cubes have only three faces painted?

(1) 4 (2) 8 (3) 12 (4) 18

Direction (Q.15 to Q.18) : A solid cube of each side 6 cms has been painted red, blue and black on pairs of
opposite faces. It is then cut into cubical blocks of each side 2 cms.

15. How many cubes have no face painted?

(1) 0 (2) 4 (3) 8 (4) 1

16. How many cubes have only one face painted?

(1) 8 (2) 16 (3) 6 (4) 4

17. How many cubes have only two faces painted?

(1) 8 (2) 16 (3) 18 (4) 12

18. How many cubes have only one face painted with red?

(1) 4 (2) 8 (3) 16 (4) 2

110 OVERSEAS www.allenoverseas.com


Mental Ability

Direction (Q.19 to Q.21) : The six face of a cube are coloured black, brown, green red, white and blue, such
that :
(i) Red is opposite to black
(ii) Green is between red and black
(iii) Blue is adjacent to white
(iv) Brown is adjacent to blue
(v) Red is at the bottom
Answer the following questions based on this information :
19. Which colour is opposite brown?
(1) White (2) Red (3) Green (4) Blue
20. The four adjacent colours are
(1) Black, Blue, Brown, Red (2) Black, Blue, Brown, White
(3) Black, Blue, Red, White (4) Black, Brown, Red, White
21. Which of the following can be deduced from (i) and (v)?
(1) Black is on the top (2) Blue is on the top
(3) Brown is on the top (4) Brown is opposite to Black
Directions (Q.22 to Q.25) : A cube is painted red on two adjacent surface and black on the surfaces opposite
to red surfaces and green on the remaining faces. Now the cube is cut into sixty four smaller cubes of equal
size.
22. How many smaller cubes have only one surface painted?
(1) 8 (2) 16 (3) 24 (4) 32
23. How many smaller cubes will have no surface painted?
(1) 0 (2) 4 (3) 8 (4) 16
24. How many smaller cubes have less than three surfaces painted?
(1) 8 (2) 24 (3) 28 (4) 56
25. How many smaller cubes have three surfaces painted?
(1) 4 (2) 8 (3) 16 (4) 24

Q u e. 1 2 3 4 5 6 7 8 9 10 11 12 13 14 15
Ans. 3 2 3 1 4 3 2 3 3 3 4 3 4 2 4
Q u e. 16 17 18 19 20 21 22 23 24 25
Ans. 3 4 4 1 4 1 3 3 4 2

www.allenoverseas.com OVERSEAS 111


Class-VIII

Important Notes

112 OVERSEAS www.allenoverseas.com


Mental Ability

INSERTING THE MISSING


CHAPTER 15 CHARACTER
In such type of questions, a figure, a set of figures, an arrangement or a matrix is given, each of which bears certain
characters, be it numbers, letters or a group/combination of letters/numbers; following a certain pattern. The
candidate is required to decipher this pattern and accordingly find the missing character in the figure.
Solved examples
Ex.1 Which one will replace the question mark?

A2 C4 E6
G3 I5 ?
M5 O9 Q14

(1) L10 (2) K15 (3) I15 (4) K8

Sol. A2 +2 C4 +2 E6, M5 +2 O9 +2 Q14, G3 +2 I5 +2 K8


How the number is obtained?
2 + 4 = 6, 5 + 9 = 14. Similarly, 3 + 5 = 8. Hence, the answer is (4).
Ex.2 Which one will replace the question mark?

5 4 7 8 6 9 5 10 3 7 2 ?
(1) 1 (2) 4 (3) 3 (4) 6
Sol. (5 + 4 + 7)/2 = 8, (6 + 9 + 5)/2 = 10, (3 + 7 + 2)/2 = 6
Hence, the answer is (4).
Ex.3 Which one will replace the question mark?

(1) 18 (2) 12 (3) 9 (4) 6


Sol. (12 + 18 + 30)/10 = 6, (16 + 24 + 40)/10 = 8. Similarly, (45 + 18 + 27)/10 = 9
Hence, the answer is (3).
Ex.4 Which one will replace the question mark?

(1) 25 (2) 37 (3) 41 (4) 47


Sol. (5 × 3) + 4 = 19 and (6 × 4) + 5 = 29. Therefore, (7 × 5) + 6 = 41
Hence, the answer is (3).

www.allenoverseas.com OVERSEAS 113


Class-VIII

Ex.5 Which one will replace the question mark?

(1) 45 (2) 41 (3) 32 (4) 40


Sol. (15 × 2 – 3) = 27, (31 × 2 – 6) = 56 and (45 × 2 – 9) = 81
Hence, the answer is (1).
Ex.6 Which one will replace the question mark?

(1) 115 (2) 130 (3) 135 (4) 140


Sol. (5 × 6 × 8) + (7 × 4 × 9) = 492 and (7 × 5 × 4) + (6 × 8 × 9) = 572
Therefore (4 × 3 × 5) + (7 × 2 × 5) = 130
Hence, the answer is (2).
Ex.7 Which one will replace the question mark?

(1) 92 (2) 72 (3) 62 (4) 99


Sol. 21 + 31 = 52 and 39 + 46 = 85
Therefore, 16 + 83 = 99
Hence, the answer is (4).
Ex.8 Which one will replace the question mark?

(1) 36 (2) 48 (3) 38 (4) 30


Sol. (0 + 2 + 6 + 4) - 2 = 10 and (6 + 2 + 10 + 8) – 2 = 24
Therefore, (4 + 14 + 12 + 10) - 2 = 38
Hence, the answer is (3).

114 OVERSEAS www.allenoverseas.com


Mental Ability

Ex.9 Which one will replace the question mark?

(1) 41 (2) 64 (3) 65 (4) 61


Sol. (1) = 1, (2) = 8, (3) = 27
3 3 3

Therefore, (4)3 = 64
Hence, the answer is (2).
Ex.10 Which one will replace the question mark?

(1) 13 (2) 14 (3) 12 (4) 15


Sol. Sum of numbers in lower half of the circle = 11 + 9 + 3 + 7 = 30
Sum of numbers in upper half of the circle =? + 4 + 5 + 6 =? + 15
Upper half = Lower half
30 =? + 15,? = 30 – 15,? = 15
Hence, the answer is (4).

www.allenoverseas.com OVERSEAS 115


Class-VIII

Directions (Q.1 to Q.7) : Find the missing character in each of the following :

7
1.
? 15

127 31
63

(1) 190 (2) 221 (3) 236 (4) 255

? 14
2.
5 10
7 9

(1) 2 (2) 3 (3) 6 (4) 18

594 198
3.

? 66

(1) 11 (2) 12 (3) 22 (4) 33

? 1
4.
8 2
5 3

(1) 10 (2) 12 (3) 13 (4) 15


5
4 1

64 8
5.
? 2
25
(1) 1 (2) 2 (3) 3 (4) 4

116 OVERSEAS www.allenoverseas.com


Mental Ability

?
235 4

117 7
6.
59 15
29

(1) 327 (2) 386 (3) 438 (4) 469

0 7

7.
? 26

(1) 45 (2) 50 (3) 60 (4) 63


Directions (Q.8 to Q.18) : In each of the following questions, a set of figures carrying certain characters, is
given. Assuming that the characters in each set follow a similar pattern, find the missing character in each case.

4 9 9 16 16 ?

8.
6 12 20

(1) 21 (2) 25 (3) 35 (4) 45

18 19 22 24 26 27

9.
35 44 ?

(1) 51 (2) 76 (3) 89 (4) 94

27 54 42 84 ? 42

10.
9 14 7

(1) 12 (2) 21 (3) 24 (4) 35

www.allenoverseas.com OVERSEAS 117


Class-VIII

2 4 3 9 1 7

11.
20 90 ?

(1) 20 (2) 25 (3) 50 (4) 75

4 3 11 9 15 6

12.
144 9801 ?

(1) 1216 (2) 2250 (3) 8100 (4) 11036

3 5 5 13 ? 17
13.
4 12 15

(1) 2 (2) 6 (3) 8 (4) 64

25 17 38 18 89 16
14.
6 8 ?

(1) 13 (2) 15 (3) 17 (4) 19

84 81 88
15.
14 12 18 9 ? 11

(1) 16 (2) 21 (3) 61 (4) 81

25 4 81

16. 16 361 1 64 289 16 9 ? 25


81 9 64

(1) 260 (2) 269 (3) 625 (4) 429

118 OVERSEAS www.allenoverseas.com


Mental Ability

5 3 2 7 6 7
17. 63 41 ?
6 8 3 9 8 5

(1) 26 (2) 82 (3) 83 (4) 86

2 6 3 5 2 3
18. 168 120 ?
3 2 2 1 4 5

(1) 84 (2) 195 (3) 240 (4) None of these

Directions (Q. 19 & Q.20) : In each of these questions, which character when placed at the sign of interrogation
shall complete the matrix?

A D H
F I M
19.
? N R

(1) K (2) N (3) O (4) P

H K Q
C G O
20.
E J ?

(1) T (2) P (3) N (4) L


Direction (Q.21 to Q.25) : Find the missing character.

149 175

21.
? 203

(1) 148 (2) 208 (3) 213 (4) 233

? 9

22.
22 13

(1) 38 (2) 39 (3) 40 (4) 44

www.allenoverseas.com OVERSEAS 119


Class-VIII

3 ?

23.
6561 81

(1) 9 (2) 18 (3) 24 (4) 27


25 25 25 49

24. 100 6 100 25 5 81 25 4 25 36 ? 9


25 36 25 16
(1) 2 (2) 3 (3) 4 (4) 5

72 24 6
96 16 12
25.
108 ? 18

(1) 12 (2) 16 (3) 18 (4) 20

Que. 1 2 3 4 5 6 7 8 9 10 11 12 13 14 15
Ans. 4 4 3 3 1 4 4 2 1 2 3 3 3 2 1
Que. 16 17 18 19 20 21 22 23 24 25
Ans. 3 2 2 1 1 4 1 1 3 1

120 OVERSEAS www.allenoverseas.com


Mental Ability

Direction (Q.1 to Q.8) : Which number will replace the question mark in the following questions?

4 8 5

1.

6 14 6 14 6 ?
8 8 4
10 18 14 22 11 15

(1) 8 (2) 14 (3) 10 (4) 6

2.
4 6 8 9 11 13

52 145 ?

(1) 143 (2) 289 (3) 290 (4) 156

3. (1) 41
3
7
(2) 22
105
(3) 25
15
?
(4) 35

4. (1) 9
8 11
(2) 15
6
226
(3) 12 37 ?
122 65
(4) 17

5.
7 5 5 21 21 7

6 13 ?

(1) 4 (2) 8 (3) 20 (4) 14

www.allenoverseas.com OVERSEAS 121


Class-VIII

5 6 6 7 4 8
6. 12 21 ?
4 5 10

(1) 22 (2) 32 (3) 31 (4) 42

7. (1) 2765
2 2 256
(2) 3125
3 2 ?
(3) 8796
4 2 46656
(4) 30008

8. (1) 25

(2) 28 35 43 28 45 15 32

40 10 ?
(3) 35
20 28 11 16 15 40
(4) 38

9. The characters in given matrix follow a certain trend, row-wise or column-wise. Find out this trend and choose
the missing character from the given options.

(1) 3 31 17 58 87
(2) 6 68 19 61 56
(3) 7 91 22 70 50

(4) 9 10 142 11 ?

10. Find the missing number in Fig.(X).

(1) 65 97 86
(2) 58 8 73

(3) 60 19 ?
34 47
(4) 62
Fig. (X)
11. There is a certain rule followed in figures P , Q and R. Identify the rule and find the missing numbers a & b
respectively.

384 576 a

96 144 b

192 288 168


P Q R

(1) 336 , 84 (2) 676, 196 (3) 484 ,86 (4) 576, 194

122 OVERSEAS www.allenoverseas.com


Mental Ability

12. A figures carrying certain numbers. Assuming that the number in each set follows a similar pattern, find the
missing number.

15 2 9 7 13 16
80 65 ?
5 6 4 6 11 8

(1) 35 (2) 48 (3) 72 (4) 120

13. Find the missing number in the given number pattern.

(1) 25 3 15
(2) 125 2 6
25 441
289 ?
(3) 156 10 13
(4) 625 7 12

14. In the given question,number will replace the question mark?


(1) 2
18 24 32
(2) 3 12 14 16
(3) 4 3 ? 4

(4) 5 72 112 128

15. Find the missing number, if a certain rule is followed row-wise or column-wise in the given matrix.

3 ? 5
5 4 7
4 4 6
36 32 114

(1) 4 (2) 6 (3) 9 (4) 8

Que. 1 2 3 4 5 6 7 8 9 10 11 12 13 14 15
Ans. 3 3 4 2 4 2 2 4 3 2 1 2 4 2 1

www.allenoverseas.com OVERSEAS 123


Class-VIII

Important Notes

124 OVERSEAS www.allenoverseas.com


Mental Ability

CHAPTER 16 MIRROR & WATER IMAGES

16.1 Mirror Images


Mirror Image: The image of an object, as seen in a mirror, is called its mirror reflection or mirror image.
In such an image, the right side of the object appears on the left side and vice-versa. A mirror image is therefore
said to be laterally inverted and the phenomenon is called Lateral Inversion.
MIRROR-IMAGE OF CAPITAL LETTERS

Letter Mirror Letter Mirror Letter Mirror


Image Image Image

A J S
B K T
C L U
D M V
E N W
F O X
G P Y
H Q Z
I R –

Remark: The letters which have their mirror images identical to the letter itself are:
A, H, I, M, O, T, U, V, W, X, Y
Examples: Mirror-images of certain words are given below:
1. FUN :

2. STOP :

3. ZEBRA :

4. GOLKONDA :

5. XYLOPHONE :

www.allenoverseas.com OVERSEAS 125


Class-VIII

MIRROR-IMAGE OF SMALL LETTERS

Letter Mirror Letter Mirror Letter Mirror


Image Image Image

a j s
b k t
c l u
d m v
e n w
f o x
g p y
h q z
i r –

Examples: Mirror-images of certain words are given below:


1. arpit :
2. blade :
3. determine :

MIRROR-IMAGE OF NUMBERS

Number Mirror Number Mirror Number Mirror


Image Image Image

1 4 7
2 5 8
3 6 9

Examples: Mirror-images of certain combinations of alphabets and numbers are given below:
1. alpha348mz1 :
2. BMC49JN2317 :
3. 15bg82XQh :

126 OVERSEAS www.allenoverseas.com


Mental Ability

Directions (Q.1 to Q.20) : In each of the following questions, you are given a combination of alphabets and/
or numbers followed by four alternatives (1), (2), (3) and (4). Choose the alternative which most closely resembles
the mirror image of the given combination
1. WHITE
(1) (2)

(3) (4)
2. BRISK
(1) (2)

(3) (4)
3. PAINTED
(1) (2)

(3) (4)
4. NATIONAL
(1) (2)

(3) (4)
5. SUPERVISOR
(1) (2)

(3) (4)
6. JUDGEMENT
(1) (2)

(3) (4)
7. QUALITY
(1) (2)

(3) (4)
8. TERMINATE
(1) (2)

(3) (4)
9. FIXING
(1) (2)

(3) (4)
10. MALAYALAM

(1) (2)

(3) (4)
11. GEOGRAPHY
(1) (2)
(3) (4)

www.allenoverseas.com OVERSEAS 127


Class-VIII

12. INFORMATIONS
(1) (2)
(3) (4)
13. REASONING
(1) (2)
(3) (4)
14. EFFECTIVE
(1) (2)
(3) (4)
15. MAGAZINE
(1) (2)
(3) (4)
16. ANS43Q12
(1) (2)
(3) (4)
17. DL9CG4728
(1) (2)
(3) (4)
18. BR4AQ16HI
(1) (2)
(3) (4)
19. UTZFY6KH
(1) (2)
(3) (4)
20. TARAIN1014A
(1) (2)
(3) (4)

Que. 1 2 3 4 5 6 7 8 9 10 11 12 13 14 15 16 17 18 19 20
Ans. 3 4 2 2 1 3 3 3 4 2 1 3 2 1 4 2 3 1 4 4

128 OVERSEAS www.allenoverseas.com


Mental Ability

Directions (Q.1 to Q.10) : In each of the following questions, choose the correct mirror-image of the fig (X).
from amongst the four alternatives (1), (2), (3) and (4) given along with it.

1.

(X) (1) (2) (3) (4)

2.
(X) (1) (2) (3) (4)

3.
(X) (1) (2) (3) (4)

4.
(X) (1) (2) (3) (4)

5. ×

(X) (1) (2) (3) (4)

6.
(X) (1) (2) (3) (4)

7.
(X) (1) (2) (3) (4)

8.
(X) (1) (2) (3) (4)

9.
(X) (1) (2) (3) (4)

www.allenoverseas.com OVERSEAS 129


Class-VIII

10.

(1) (2) (3) (4)

11. Identify the mirror image of the question shape.

(1) (2) (3) (4)

12. Find the mirror image of Fig. (X) from amongst the given options, if the mirror is placed vertically on the left.

(X)

E
(1) (2) (3) (4)

13. Choose the correct mirror image of the given figure (X)

(X) (1) (2) (3) (4)

14. The correct mirror-image of the fig(X), if the mirror is placed vertically left is ______.

(X)

(1) (2) (3) (4)

130 OVERSEAS www.allenoverseas.com


Mental Ability

15. Find out from the options which is the mirror-image of the Fig (X), if mirror is placed along MR.
M

R
(X)

(1) (2) (3) (4)

Que. 1 2 3 4 5 6 7 8 9 10
Ans. 4 3 4 2 2 3 2 1 3 1
Que. 11 12 13 14 15
Ans. 3 2 4 2 2

www.allenoverseas.com OVERSEAS 131


Class-VIII

16.2 Water Images


Water Image: The reflection of an object, as seen in water, is called its water image. It is the inverted image
obtained by turning the object upside down.
WATER-IMAGE OF CAPITAL LETTERS

Letter

Water
Image

Letter

Water
Image
Letter

Water
Image
Remark 1 : The letters whose water-images are identical to the letter itself are:
C, D, E, H, I, O, X.
Remark 2 : Certain words which have water-images identical to the word itself are:
CHIDE, CODE, HIDE, HE

WATER-IMAGE OF SMALL LETTERS

Letter
Water
Image
Letter
Water
Image
Letter
Water
Image

WATER-IMAGE OF NUMBERS

Number 9
Water
Image

132 OVERSEAS www.allenoverseas.com


Mental Ability

Directions (Q.1 to Q.15) : In each of the following questions, you are given a combination of alphabets and/
or numbers followed by four alternatives (1), (2), (3) and (4). Choose the alternative which most closely resembles
the water-image of the given combination.
1. DISC

2. FROG

3. RECRUIT

4. ACOUSTIC

5. FAMILY

6. NUCLEAR

7. QUARREL

8. U4P15B7

9. PQ8AF5BZ9

www.allenoverseas.com OVERSEAS 133


Class-VIII

10. D6Z7F4

11. VAYU8436

12. BK50RP62

13. 96FSH52

14. RAJ589D8

15. GR98AP76ES

Que. 1 2 3 4 5 6 7 8 9 10
Ans. 3 1 2 2 4 4 1 3 4 3
Que. 11 12 13 14 15
Ans. 2 2 3 1 3

134 OVERSEAS www.allenoverseas.com


Mental Ability

Directions (Q.1 to Q.15) : In each of the following questions, choose the correct water-image of the fig. (X)
from amongst the four alternatives (1), (2), (3) and (4) given along with it.

1.
(X)

(1) (2) (3) (4)

2.
(X)

(1) (2) (3) (4)

3.
(X)

(a)
(1) (b)
(2) (c)
(3) (d)
(4)

4.
(X)

(1)
(a) (b)
(2) (c)
(3) (4)

5.
(X)

(a)
(1) (2) (3) (4)

www.allenoverseas.com OVERSEAS 135


Class-VIII

×
6.

(X)

× × × ×

(1) (2) (3) (4)


7.

(X) (1) (2) (3) (4)

8.

(X) (1) (2) (3) (4)


9.

(X) (1) (2) (3) (4)


10.

(X) (1) (2) (3) (4)

11.

(X) (1) (2) (3) (4)

12.

13.

(X) (1) (2) (3) (4)

136 OVERSEAS www.allenoverseas.com


Mental Ability

14.

(X) (1) (2) (3) (4)

15.

(X)

(1) (2) (3) (4)

Que. 1 2 3 4 5 6 7 8 9 10
Ans. 3 1 4 2 2 3 2 2 3 4
Que. 11 12 13 14 15
Ans. 4 4 4 2 2

www.allenoverseas.com OVERSEAS 137


Class-VIII

Important Notes

138 OVERSEAS www.allenoverseas.com


Mental Ability

CHAPTER 17 ARITHMETICAL REASONING

This chapter of reasoning deals with general arithmetical problems common in nature. Solutions of these problems
require common sense with slight amount of logical reasoning.
Solved examples
Ex.1 The number of boys in a class are three times the number of girls. Which one of the following number can’t
represent the total number of children in the class?
(1) 48 (2) 44 (3) 42 (4) 40
Sol. Let the number of girls be x, then from the question it is clear that number of boys are 3x,
Therefore, total no. of students = Number of boys + Number of girls = 3x + x = 4x
Now, the total number of children in the class must be a multiple of (4) does not qualify this condition. Therefore,
42 does not represent the total number of children in the class. Hence, the answer is (3).
Ex.2 In 10 years, A will be twice as old as B was 10 years ago. If at present A is 9 years older than B, the present
age of B is
(1) 19 years (2) 29 years (3) 39 years (4) 49 years
Sol. Let the present age of B be x years.
Then, the present age of A would be (x + 9) years.
After 10 years, the age of A would be (x+9+10) = (x + 19) years and before ten years, the age of B was (x
- 10) years. Now, from the information given in the question,
(x + 19) = 2(x – 10)
or x + 19 = 2x – 20 or x = 19 + 20 = 39 years
Therefore, the present age of B is 39 years. Hence, the answer is (3).
Ex.3 A shepherd had 17 sheep. All but nine died. How many was he left with?
(1) Nil (2) 8 (3) 9 (4) 17
Sol. "All but nine died" means "All except nine died". It means that nine sheep remained alive and other died.
Hence, the answer is (3).
Ex.4 In a chess tournament each of six players will play with all other players exactly once. How many matches will
be played during the tournament?
(1) 12 (2) 15 (3) 30 (4) 36
Sol. The situation in which the matches will be played would be as follows :
(i) 1st player will play matches with other 5 players.
(ii) 2nd player will play matches with 4 players other than the 1st player.
(iii) 3rd player will play matches with 3 players other than 1st and 2nd players.
(iv) 4th player will play matches with 2 players other than 1st, 2nd and 3rd players.
(v) 5th player will play match with 6th player only.
Therefore, the number of matches played during the tournament is
= 5 + 4 + 3 + 2 + 1 =15.
Hence, the answer is (2).

www.allenoverseas.com OVERSEAS 139


Class-VIII

Ex.5 In a group of 15 people, 7 read French, 8 read English while 3 of them read none of these two. How many
of them read French and English both?

(1) 0 (2) 3 (3) 4 (4) 5

Sol. Let circles x and y represent people who read French and English, respectively. Area A shows the people who
read French only. Area C shows the people who read English only. Area B represents people who read both
French and English.

Now, (A + B + C) + 3 = 15

or A + B + C = 12 ...(i) x y

Adding these two, we get A B

A + 2B + C = 15 ...(ii)
3
Subtracting (i) from (ii), we get

B = 15 – 12 = 3

Therefore, number of people who read French and English both is 3. Hence, the correct answer is (2).

1. At the end of a business conference, ten people present shake hands with each other once. How many handshakes
will be there all together?
(1) 20 (2) 45 (3) 55 (4) 90
2. A worker may claim Rs 15 for each km which he travels by taxi and Rs 5 for each km which he drives his
own car. If in one week he claimed Rs 500 for travelling 80 km, how many km did he travel by taxi?
(1) 10 (2) 20 (3) 30 (4) 40
3. Pushpa is twice as old as Rita was two years ago. If difference between their ages be 2 years, how old is Pushpa
today?
(1) 6 years (2) 8 years (3) 10 years (4) 12 years
4. After five years, the age of a father will be thrice the age of his son, whereas five years ago, he was seven
times old as his son was. What is father's present age?
(1) 35 years (2) 40 years (3) 45 years (4) 50 years
5. In a group of person travelling in a bus, 6 persons can speak Tamil, 15 can speak Hindi and 6 can speak Gujarati.
In that group, none can speak any other language. If 2 persons in the group can speak two language and one
person can speak all the three languages, then how many persons are there in the group?
(1) 21 (2) 22 (3) 23 (4) 24
6. What is the product of all the numbers in the dial of a telephone?
(1) 1,58,480 (2) 1,59,450 (3) 1,59,480 (4) None of these

140 OVERSEAS www.allenoverseas.com


Mental Ability

7. A man wears socks of two colours black and brown. He has altogether 20 blacks socks and 20 brown socks
in a drawer. Supposing he has to take out the socks in the dark, how many must he take out to be sure that
he has a matching pair?
(1) 2 (2) 3 (3) 20 (4) 21
8. The total number of digits used in numbering the pages of a book having 366 pages is
(1) 732 (2) 990 (3) 1098 (4) 1305
9. A monkey climbs 30 feets at the beginning of each hour and rests for while when he slips back 20 feet before
he again start climbing in the beginning of the next hour. If he begins his ascent at 8:00 a.m., at what time
will he first touch a flag at 8:00 a.m. at what time will he first touch a flag at 120 feet from the ground?
(1) 4 P.M. (2) 5 P.M. (3) 6 P.M. (4) None of these
10. If a clock takes seven second to strick seven, how long will it take to strike ten?
(1) 7 seconds (2) 9 seconds (3) 10 seconds (4) None of these
11. Out of a total of 120 musicians in a club, 5% can play all the three instruments – guitar, violin and flute. It
so happens that the number of musicians who can play any two and only two of the above instruments is 30.
The number of musicians who can play the guitar alone is 40. What is the total number of those who can play
violin alone or flute alone?
(1) 30 (2) 38 (3) 44 (4) 45
12. There are 50 students admitted to a nursery class. Some students can speak only English and some can speak
only Hindi. Ten students can speak both English and Hindi. If the number of students who can speak English
is 21, then how many students can speak Hindi, only Hindi and only English
(1) 39, 29 and 11, respectively (2) 37, 27 and 13, respectively
(3) 28, 18 and 22, respectively (4) 21, 11 and 29, respectively
Directions (Q.13 to Q.15) : Read the following information carefully and then answer the questions that follow
A publishing firm publishes newspapers A, B and C. In an effort to persuade advertisers to insert advertisements
in these newspapers, the firm sends out the following statements to possible advertisers
A survey of representative sample of the whole population shows that –
Newspaper A is read by 26% Newspaper B is read by 25%
Newspaper C is read by 14% Newspaper A and B are read by 11%
Newspaper B and C are read by 10% Newspaper C and A are read by 9%
Newspaper C only is read by 0%
13. The percentage of readers who read all the three newspapers is
(1) 1 (2) 4 (3) 5 (4) 6
14. The percentage of readers who read Newspapers A and B but not C, is
(1) 2 (2) 4 (3) 5 (4) 6
15. The percentage of readers who read at least one of the three newspapers is
(1) 40 (2) 50 (3) 60 (4) 65
16. The number of two digit numbers which are both perfect squares and perfect cubes is______ .
(1) 0 (2) 2 (3) 1 (4) 3

www.allenoverseas.com OVERSEAS 141


Class-VIII

17. The sum of the digits of a 2-digit number is 12. If the new number formed by reversing the digits is greater
than the original number by 54, then the original number is ____.
(1) 39 (2) 93 (3) 49 (4) 23
18. If a man left one-third of his money to his daughter, one-fourth to his son and the remaining ` 20,000 to his
wife, then the money man has left is ______.
(1) `36,000 (2) `48,000 (3) `28,000 (4) `54,000
19. In a zoo, there are rabbits and pigeons. If heads are counted, there are 340 heads and if legs are counted
there are 1060 legs. How many pigeons are there?
(1) 120 (2) 150 (3) 180 (4) 170
20. In a survey, 240 parents of a school were asked about the number of hours spent by their children on their
study (including homework) in a day. The data obtained is represented by given pie. chart From this pie chart,
find the number of students in each sector in terms of percentage.

(1) P = 20%, Q = 20%, R = 60% (2) P = 15%, Q = 60%, R = 25%


(3) P = 15 %, Q = 25 %, R = 60 % (4) P = 20%, Q = 60%, R = 20%
21. Thirty six vehicles are parked in a parking lot in a single row. After the first car, there is one scooter. After
the second car, there are two scooters. After the third car, there are three scooters and so on. Find the number
of scooters in the second half of the row.
(1) 10 (2) 12 (3) 15 (4) 17
22. A restaurant manager bought 20 packages of bagels. Some packages contained 6 bagels each and the rest
contained 12 bagels each. There were 168 bagels in all. How many packages of 12 bagels did the manager
buy?
(1) 6 (2) 8 (3) 9 (4) 12
23. A group of students decided to collect as many paise from each member of the group as is the number of
members. If the total collection amounts to ` 59.29, the number of members in the group is
(1) 57 (2) 67 (3) 77 (4) 87
24. If 7 spiders make 7 webs in 7 days, then 1 spider will make 1 web in how many days?
(1) 1 (2) 7/2 (3) 7 (4) 49
25. Present ages of X and Y are in the ratio 5 : 6 respectively. Seven years hence this ratio will become 6 : 7
respectively. What is X's present age?
(1) 35 years (2) 42 years (3) 49 years (4) Can't be determined

Que. 1 2 3 4 5 6 7 8 9 10 11 12 13 14 15 16 17 18 19 20
Ans. 2 1 1 2 2 4 2 2 3 4 3 1 3 4 1 3 1 2 2 3
Que. 21 22 23 24 25
Ans. 3 2 3 3 1

142 OVERSEAS www.allenoverseas.com


Mental Ability

ALPHA-NUMERIC SEQUENCE PUZZLE &


CHAPTER 18 LOGICAL SEQUENCE OF WORDS

18.1 Alpha-Numeric Sequence Puzzle


In this type of questions, a jumbled sequence of some letters, numbers and symbols is given, followed by
certain questions based on it.
The questions asked may be any of the following types :
1. To find elements (letters, numbers or symbols) satisfying certain conditions in the given sequence.
2. To determine the element at a certain given relative position in the sequence.
3. To complete a series consisting of terms formed by a combination of the elements of the sequence, by
deciphering the hidden pattern in relation to the position of the elements in the sequence.
4. To find the odd combination from among a given set of combinations of elements, by finding the common
property which other combinations share as per their positions in the given sequence.
5. To find the analogical relationship between a given pair of combinations of elements, and then find a
combination which holds same relationship with another given combination.
Solved examples
Study the following arrangement of symbols, letters and numbers to answer the questions given
below it :
d= b F 2 K S 7 5 # $ P L V 8 @ M U E 6 ­ Q G © 9 3 & T Y £
Ex.1 How many such letters are there in the arrangements each of which is either immediately preceded by a
symbol or immediately followed by a number, but not both?
(1) Three (2) Four (3) Five (4) None of these
Sol. The letters satisfying the given conditions are shown as under :

d = b F 2 K S 7 5 # $ P L V 8 @ M U E 6 ­ Q G © 9 3 & T Y £

Clearly, there are 8 such letters.


Hence, the answer is (4).
Ex.2 How many such consonants are there in the above arrangement each of which is immediately followed by
a consonant but not immediately preceded by a symbol?
(1) Nil (2) One (3) Two (4) Three
Sol. We know that of the 26 letters of English alphabet, five letters namely A, E, I, O, U are vowels, while remaining
are consonants.
The consonants satisfying the given conditions may be shown as under :

d= b F 2 K S 7 5 # $ P L V 8 @ M U E 6 ­ Q G © 9 3 & T Y £
Clearly, there is only one such consonant.
Hence, the answer is (2).
Ex.3 Which of the following is exactly in the middle of the ninth from the left end and the fifteenth from the right
end in the above arrangement?
(1) P (2) U (3) E (4) L

www.allenoverseas.com OVERSEAS 143


Class-VIII

Sol. Clearly, counting from the left, the ninth element is 7. Counting from the right, the fifteenth element is @.
The character midway between 7 and @ is P.
Hence, the answer is (1).
Ex.4 If all the elements after the middle element in the above arrangement are written in the reverse order, which
of the following will be seventh towards right of the twelfth element from the left end?
(1) U (2) 2 (3) T (4) 6
Sol. The middle element in the given arrangement is 8. Writing all the elements after 8 in the reverse order,
we get the following sequence :
d= b F 2 K S 7 5 # $ P L V 8 £ Y T & 3 9 © G Q ­ 6 E U M @
Counting from the left in the above sequence, the twelfth element is $. The seventh element to the right
of $ is T. Hence, the answer is (3).
Ex.5 Which of the following groups of elements will come in place of the question mark in the series of elements
given below?
K 2 d 5 S F P # K ? U @ P
(1) 8 L 5 (2) @ V # (3) V P 7 (4) 8 $ 5
Sol. Clearly, first and second elements of each term move two steps and four steps backward respectively to give
the second and third elements of the same term. The third element of each term moves nine steps forward
to give the first element of the next term. The pattern is repeated in the subsequent terms.
Now, the letter 9 steps ahead of K is 8; the letter 2 steps behind 8 is L and the letter 4 steps behind L
is 5. Thus, the missing term is 8 L 5. Hence, the answer is (1).

Ex.6 Based on the positions in the above arrangement, if b F d : T & £, then K 7 :?


(1) © 9 Q (2) © G Q (3) 9 © Q (4) © 9 ­
Sol. Observing the given analogous pair, we find that the corresponding elements of both the terms occupy the
same position from the beginning and end of the given arrangement.

Now, K, and 7 are respectively 7th, 6th and 9th elements from the left end in the given arrangement.
The 7th, 6th and 9th elements from the right end are ©, 9 and Q respectively. Thus, the missing term is
© 9 Q.
Hence, the answer is (1).

Directions (Q.1 to Q.5) : Study the following digit letter symbol sequence carefully and answer the questions
given below:
R * T J L 2 $ D = M # 8 C % B < K 1 & A W? P E + Q @ 7 F 6
1. How many such numbers are there in the above sequence, each of which is immediately preceded by a conso-
nant and immediately followed by a symbol?
(1) Nil (2) One (3) Three (4) None of these

144 OVERSEAS www.allenoverseas.com


Mental Ability

2. Which of the following is sixth to the left of eighteenth element from the left?
(1) % (2) C (3) 1 (4) 8
3. If the above sequence is written in reverse order then which of the following will be sixth to the right of sixteenth
element from the right end?
(1) M (2) A (3) B (4)?
4. If the first fifteen elements in the above sequence are written in reverse order, then which of the following will be
twenty first from the right end?
(1) 2 (2) $ (3) = (4) L
5. What should come in place of question mark in the following on the basis of above sequence?
* R J : F 6 @ : : L J $ : Q @ E : : D $ M :?
(1) # M C (2) P E W (3) P? + (4) P E?
Directions (Q.6 to Q.10) : Study the following series of alpha-numeric symbol combination and answer the
questions that follow:
SK6£Q2R*CF8E$G2#49LN3UV5YaB7W9
6. How many symbols are there in the above arrangement each of which is immediately preceded by but not
immediately followed by a letter of English alphabet?
(1) Nil (2) One (3) Two (4) Three
7. What should come in place of question mark in the following series?
S 9 K 6 W £ Q 7 2? C a F
(1) R Y * (2) *Y C (3) 2 B R (4) R B *
8. If the positions of # and $ are interchanged, also the positions of £ and *,Q and K and F and V are interchanged,
which of the following will be the ninth element to the left of eighteenth element from the left?
(1) C (2) V (3) 8 (4) *
9. Three of the following four are alike in respect of their positions in the above series. Which is the one that does
not belong to that group?
(1) Q * 8 (2) £ R F (3) 6 2 C (4) 2 * 8
10. S K £ is related to 9 W B in the same way as Q 2 * is related to :
(1) a V U (2) 7 B Y (3) B Y V (4) a Y V
Direction (Q.11 to Q.15) : Observe the following sequence and answer the questions that follows:
@#$#@$@#$$$@##@@@@$###$$
11. How many times three @'s occur together?
(1) 2 (2) 1 (3) 4 (4) 5
12. How many times exactly two #'s occur together?
(1) 2 (2) 1 (3) 4 (4) 5
13. How many times $ is followed by exactly one @?
(1) 2 (2) 1 (3) 4 (4) 5
14. How many times is a # not followed by a $?
(1) 2 (2) 1 (3) 4 (4) 5

www.allenoverseas.com OVERSEAS 145


Class-VIII

15. How many times is a @ followed by exactly two #?


(1) 2 (2) 1 (3) 4 (4) 5
Direction (Q.16 to Q.20) : Observe the following sequence and answer the questions that follows:
XXYYXYXYYYXYXYXYYYXYYXXXY
16. How many times three Y's occur together?
(1) 2 (2) 1 (3) 3 (4) 6
17. How many times three X's occur together?
(1) 2 (2) 1 (3) 3 (4) 6
18. How many times is X followed by exactly two Y's?
(1) 2 (2) 1 (3) 3 (4) 6
19. How many times is X not followed by Y?
(1) 2 (2) 1 (3) 3 (4) 6
20. How many times is a Y followed by exactly one X?
(1) 2 (2) 1 (3) 3 (4) 6

Q ue . 1 2 3 4 5 6 7 8 9 10
A ns . 4 4 1 1 2 1 4 1 4 4
Q ue . 11 12 13 14 15 16 17 18 19 20
A ns . 1 2 1 4 2 1 2 1 3 4

146 OVERSEAS www.allenoverseas.com


Mental Ability

18.2 Logical Sequence of Words


In this type of questions, certain inter-related words are given and numbered, followed by various sequences of
the numbers denoting them, as alternatives. The candidate is required to arrange these words in a logical
sequence based on a common property and then choose the correctly graded sequence from the given alternatives.
Some common sequences have been discussed below:
1. Sequence of occurrence of events
or various stages in a process
Ex.1 Arrange the following in a meaningful sequence:
(1) Consultation (2) Illness (3) Doctor (4) Treatment
(5) Recovery
(1) 2, 3, 1, 4, 5 (2) 2, 3, 4, 1, 5 (3) 4, 3, 1, 2, 5 (4) 5, 1, 4, 3, 2
Sol. Clearly, illness occurs first. One then goes to a doctor and after consultation with him, undergoes treatment
to finally attain recovery.
Thus, the correct order is 2, 3, 1, 4, 5. Hence the answer is (1)
Ex.2 Arrange the following in a logical order:
(1) Birth (2) Death (3) Funeral (4) Marriage
(5) Education
(1) 1, 3, 4, 5, 2 (2) 1, 5, 4, 2, 3 (3) 2, 3, 4, 5, 1 (4) 4, 5, 3, 1, 2
Sol. Clearly the given words when arranged in the order of various events as they occur in a man's life, form the
sequence : Birth, Education, Marriage, Death, Funeral.
So the correct order becomes 1, 5, 4, 2, 3
Hence the answer is (2)

2. Sequence of objects in a class


or group, from part to the whole
Ex.3 Arrange the following in a meaningful order, from particular to general :
(1) Family (2) Community (3) Member (4) Locality
(5) Country
(1) 3, 1, 2, 4, 5 (2) 3, 1, 2, 5, 4 (3) 3, 1, 4, 2, 5 (4) 3, 1, 4, 5, 2
Sol. Clearly a member is a part of a family, which in turn is a part of community. The community lives in a
locality which lives within a country.
Thus, the correct order is 3, 1, 2, 4, 5.
Hence the answer is (1)
Ex.4 Arrange the following in a logical order :
(1) Shoulder (2) Wrist (3) Elbow (4) Palm
(5) Finger
(1) 2, 4, 5, 3, 1 (2) 3, 1, 4, 2, 5 (3) 3, 4, 5, 2, 1 (4) 5, 4, 2, 3, 1

www.allenoverseas.com OVERSEAS 147


Class-VIII

Sol. Clearly, we are given the names of parts of a hand, which may be arranged
(1) From top to bottom, i.e., Shoulder, Elbow, Wrist, Palm, Finger, which is 1, 3, 2, 4, 5 or
(2) From bottom to top, i.e., Finger, Palm, Wrist, Elbow, Shoulder, which is 5, 4, 2, 3, 1
Out of these, the sequence 5, 4, 2, 3, 1 is given in the alternatives provided.
Hence the answer is (4)

3. Sequence of increasing / decreasing size,


value, intensity etc.

Ex.5 Arrange the following in a logical sequence from small to big :

(1) Bungalow (2) Flat (3) Cottage (4) House

(5) Palace (6) Mansion

(1) 3, 2, 1, 4, 6, 5 (2) 3, 2, 4, 1, 5, 6 (3) 3, 2, 4, 1, 6, 5 (4) 5, 6, 4, 1, 2, 3

Sol. Clearly, the names of various dwelling units, when arranged in increasing order of their sizes form the sequence :
Cottage, Flat, House, Bungalow, Mansion, Palace.

Thus, the correct order is 3, 2, 4, 1, 6, 5

Hence the answer is (3)

Ex.6 Arrange the following in a logical order :

(1) Gold (2) Iron (3) Sand (4) Platinum

(5) Diamond

(1) 2, 4, 3, 5, 1 (2) 3, 2, 1, 5, 4 (3) 4, 5, 1, 3, 2 (4) 5, 4, 3, 2, 1

Sol. Clearly, the given names when arranged in order of increasing values, i.e., from cheapest to the most expensive,
form the sequence : Sand, Iron, Gold, Diamond, Platinum.

Thus, the correct order is 3, 2, 1, 5, 4

Hence the answer is (2)

4. Sequence in which a chain of given


objects is formed
Ex.7 Arrange the following in a meaningful sequence :

(1) Phrase (2) Letter (3) Word (4) Sentence

(1) 1, 2, 3, 4 (2) 1, 3, 2, 4 (3) 2, 3, 1, 4 (4) 2, 3, 4, 1

Sol. A group of letters makes a word. A group of words makes a phrase. A group of phrases makes a sentence.

Thus, the correct order is 2, 3, 1, 4

Hence the answer is (3)

148 OVERSEAS www.allenoverseas.com


Mental Ability

Ex.8 Arrange the following in a logical order:

(1) Frog (2) Eagle (3) Grasshopper (4) Snake

(5) Grass

(1) 1, 3, 5, 2, 4 (2) 3, 4, 2, 5, 1 (3) 5, 3, 1, 4, 2 (4) 5, 3, 4, 2, 1

Sol. Clearly, a grasshopper feeds on grass, a frog feeds on a grasshopper, a snake feeds on frog and an
eagle feeds on a snake. Thus, a food-chain is formed.

So, the correct order is 5, 3, 1, 4, 2

Hence the answer is (3)

1. Arrange the words given below in a meaningful sequence.


1. Leaf 2. Fruit 3. Stem 4. Root
5. Flower
(1) 3, 4, 5, 1, 2 (2) 4, 3, 1, 5, 2 (3) 4, 1, 3, 5, 2 (4) 4, 3, 1, 2, 5
2. Arrange the words given below in a meaningful sequence.
1. Nation 2. Village 3. City 4. District
5. State
(1) 2, 3, 4, 5, 1 (2) 2, 3, 4, 1, 5 (3) 1, 3, 5, 4, 2 (4) 1, 2, 3, 4, 5
3. Arrange the words given below in a meaningful sequence.
1. Caste 2. Family 3. Newly married Couple 4. Clan
5. Species
(1) 2, 3, 1, 4, 5 (2) 3, 4, 5, 1, 2 (3) 3, 2, 1, 4, 5 (4) 4, 5, 3, 2, 1
4. Arrange the words given below in a meaningful sequence.
1. Cut 2. Put on 3. Mark 4. Measure
5. Tailor
(1) 3, 1, 5, 4, 2 (2) 2, 4, 3, 1, 5 (3) 1, 3, 2, 4, 5 (4) 4, 3, 1, 5, 2
5. Arrange the words given below in a meaningful sequence.
1. House 2. Street 3. Room 4. Town
5. District
(1) 3, 2, 1, 4, 5 (2) 3, 1, 4, 2, 5 (3) 3, 1, 2, 4, 5 (4) 3, 1, 2, 5, 4
6. Arrange the words given below in a meaningful sequence.
1. Wall 2. Clay 3. House 4. Room
5. Bricks
(1) 5, 2, 1, 4, 3 (2) 2, 5, 4, 1, 3 (3) 2, 5, 1, 4, 3 (4) 1, 2, 3, 4, 5

www.allenoverseas.com OVERSEAS 149


Class-VIII

7. Arrange the words given below in a meaningful sequence.


1. Probation 2. Interview 3. Selection 4. Appointment
5. Advertisement 6. Application
(1) 5, 6, 3, 2, 4, 1 (2) 5, 6, 4, 2, 3, 1 (3) 5, 6, 2, 3, 4, 1 (4) 6, 5, 4, 2, 3, 1
8. Arrange the words given below in a meaningful sequence.
1. Elephant 2. Cat 3. Mosquito 4. Tiger
5. Whale
(1) 5, 3, 1, 2, 4 (2) 3, 2, 4, 1, 5 (3) 1, 3, 5, 4, 2 (4) 2, 5, 1, 4, 3
9. Arrange the words given below in a meaningful sequence.
1. Yarn 2. Plant 3. Saree 4. Cotton
5. Cloth
(1) 2, 4, 5, 1, 3 (2) 2, 4, 3, 5, 1 (3) 2, 4, 1, 5, 3 (4) 2, 4, 5, 3, 1
10. Arrange the words given below in a meaningful sequence.
1. Infant 2. Old 3. Adult 4. Adolescent
5. Child
(1) 5, 4, 3, 2, 1 (2) 3, 4, 2, 1, 5 (3) 2, 3, 4, 5, 1 (4) 1, 5, 4, 3, 2
11. Arrange the words given below in a meaningful sequence.
1. Leaves 2. Branch 3. Flower 4. Tree
5. Fruit
(1) 4, 3, 1, 2, 5 (2) 4, 2, 5, 1, 3 (3) 4, 3, 2, 1, 5 (4) 4, 2, 1, 3, 5
12. Arrange the words given below in a meaningful sequence.
1. Cutting 2. Dish 3. Vegetable 4. Market
5. Cooking
(1) 1, 2, 4, 5, 3 (2) 3, 2, 5, 1, 4 (3) 4, 3, 1, 5, 2 (4) 5, 3, 2, 1, 4
13. Arrange the words given below in a meaningful sequence.
1. Hecto 2. Centi 3. Deca 4. Kilo
5. Deci
(1) 1, 3, 4, 5, 2 (2) 1, 5, 3, 4, 2 (3) 2, 5, 3, 1, 4 (4) 5, 2, 1, 4, 3
14. Arrange the words given below in a meaningful sequence.
1. Protect 2. Pressure 3. Relief 4. Rain
5. Flood
(1) 2, 4, 3, 1, 5 (2) 2, 4, 5, 1, 3 (3) 2, 5, 4, 1, 3 (4) 3, 2, 4, 5, 1
15. Arrange the words given below in a meaningful sequence.
1. Presentation 2. Recommendation 3. Arrival 4. Discussion
5. Introduction
(1) 5, 3, 4, 1, 2 (2) 3, 5, 4, 2, 1 (3) 3, 5, 1, 4, 2 (4) 5, 3, 1, 2, 4

Q ue . 1 2 3 4 5 6 7 8 9 10
An s. 2 1 3 4 3 3 3 2 3 4
Q ue . 11 12 13 14 15
An s. 4 3 3 2 3

150 OVERSEAS www.allenoverseas.com


Mental Ability

FIGURE MATRIX, FIGURE FORMATION,


CHAPTER 19 CONSTRUCTION OF SQUARES &
GROUPING OF IDENTICAL FIGURES

In this Type of questions, more than one set of figure is given in the form of a matrix, all of them following the
same rule. The candidate is required to analyse the complete sets; find out the common rule and then on the
basis, find the missing figure in the incomplete set.
Solved example

FIGURE MATRIX
In such type of problems a 2 × 2 or 3 × 3 grid is given. This grid has some design or symbols to form a pattern.
But a cell of the grid is left empty. A candidate requires to fill up the cell. Now, one needs to analyse the
grid and identify a rule along row-wise or column-wise in the grid.
Ex.1 Complete the given pattern.

?
(1) (2)

(3) (4)

Sol. Let us consider horizontally. The second figure is obtained from the first figure by moving the line segment to the
opposite side of the square boundary and replacing it with two similar line segments. Also, the element in the
lower-left corner gets replaced by two similar elements – one placed in the upper-left and the other placed in
the lower right corner. Hence, the answer is (1).

FIGURE FORMATION
This section deals with the following types of problems.

1. Formation of a Figure from its Segments: In such type of problems all the parts to form a figure are
given. A candidate requires to identify the figure so formed out of the four options.

2. Choosing a Pattern which has the same components as a given Pattern: In such type of problems,
a pattern of several components are given. Only one pattern out of four option patterns contains the same
components. A candidate requires to choose such pattern.

www.allenoverseas.com OVERSEAS 151


Class-VIII

Solved Examples
Ex.2 Find out which of the figures (1), (2), (3) and (4) can be formed from the pieces given in figure (X).

(X)

(1) (2) (3) (4)

Sol. The parts of figure in option (1) are in figure (X). Hence, the answer is (1).
Ex.3 Select that option which has the same components as the given figure (X).

(X)

(1) (2) (3) (4)

Sol. Components of figure (X) and figure (2) are exactly the same. Hence, the answer is (2).

CONSTRUCTION OF SQUARES
Such type of problems are on the basis of the geometrical figures.
A candidate is required to identify the figure which can fit into each other to form a square.
Ex.4 Select a figure from the given four aternatives which fits exactly into figure (X) to form a complete square.

(X)

(1) (2) (3) (4)

Sol. Figures (X) and (3) together form a square.

Hence, the answer is (3).

152 OVERSEAS www.allenoverseas.com


Mental Ability

GROUPING OF IDENTICAL FIGURES


In such type of problems a set of some figures is given. A candidate is required to classify these figures into
groups on the basis of some common properties amongst them.
EX.5 Group the following figures into three classes regarding common properties amongst them.

1 2 3

4 5 6

7 8 9

(1) 4 , 7, 9; 2, 5, 8; 1, 3, 6 (2) 4 , 7, 9; 2, 5, 6; 1, 3, 8
(3) 1 , 2, 3; 4, 5, 6; 7, 8, 9 (4) 1 , 2, 3; 4, 7, 9; 5, 6, 8
Sol. 4, 7, 9 are blank figures.
2,5,6 each is divided into 4 parts by two mutually perpendicular lines.
1, 3, 8 each is divided into 6 parts. Hence, the answer is (2).

Directions (Q.1 & Q.2): In each of the following questions, group the given figures into three classes using
each figure only once.

1.
1 2 3

4 5 6

(1) 1, 4; 2, 3; 5, 6 (2) 1, 5; 2, 6; 4, 3

(3) 1, 6; 2, 3; 4, 5 (4) 1, 2; 3, 6; 4, 5

2.
1 2 3 4

5 6 7

(1) 1, 5, 6; 3, 4, 7; 2 (2) 1, 3; 2, 6; 4, 5, 7

(3) 1, 2, 6; 7, 3; 4, 5 (4) 1, 3; 2, 4, 5; 6, 7

www.allenoverseas.com OVERSEAS 153


Class-VIII

Directions (Q.3 & Q.4) : Group the given figures into three classes using each figures only once.

3.

1 2 3

4 5 6

7 8 9

(1) 1, 2, 5 ; 3, 7, 8 ; 4, 6, 9 (2) 1, 7, 2 ; 3, 9, 6 ; 4, 5, 8
(3) 2, 3, 8 ; 4, 6, 9 ; 1, 5, 7 (4) 5, 6, 9 ; 3, 4, 1 ; 2, 7, 8
4.

1 2 3

4 5 6

7 8 9

(1) 2, 4, 7 ; 1, 8, 9 ; 3, 5, 6 (2) 2, 6, 9 ; 1, 5, 7 ; 3, 4, 8
(3) 2, 6, 7 ; 1, 5, 8 ; 3, 4, 9 (4) 2, 8, 7 ; 1, 5, 9 ; 3, 4, 6
Directions (Q. 5 & Q.6): In each of the following questions, select the figure from a set of four figures (1), (2),
(3) and (4) that can be formed by joining the figures given in box marked (X).

5.

(X)

(1) (2) (3) (4)

6.

(X)

(1) (2) (3) (4)


Directions (Q.7 to Q.18) : In each of the following questions, find out which of the answer figures (1), (2), (3)
and (4) completes the figure matrix?

154 OVERSEAS www.allenoverseas.com


Mental Ability

7.

(1) (2) (3) (4)

8.

(1) (2) (3) (4)

9.

(1) (2) (3) (4)

10.

(1) (2) (3) (4)

www.allenoverseas.com OVERSEAS 155


Class-VIII

11.

(1) (2) (3) (4)

12.

(1) (2) (3) (4)

13.

(1) (2) (3) (4)


14.

(1) (2) (3) (4)

156 OVERSEAS www.allenoverseas.com


Mental Ability

15.

(1) (2) (3) (4)

16.

(1) (2) (3) (4)

17.

(1) (2) (3) (4)

18.

(1) (2) (3) (4)

www.allenoverseas.com OVERSEAS 157


Class-VIII

Direction (Q.19 & Q.20) : In each of the following questions, find out which of the figure (1), (2), (3) and
(4) can be formed from the pieces given in a question figures.

Question Figure Answer figures

19.

(1) (2) (3) (4)

Question Figure Answer figures

20.

(1) (2) (3) (4)

21. In the following question, find out which of the figures (1), (2), (3) and (4) can be formed from the pieces
given in the question figure.

Question Figure Answer figures

(1) (2) (3) (4)

Directions (Q.22 to Q.26): In each of the following questions, find out which of the answer figures (1), (2), (3)
and (4) completes the figure matrix?
22.

(1) (2) (3) (4)


23.

(1) (2) (3) (4)

158 OVERSEAS www.allenoverseas.com


Mental Ability

24.

(1) (2) (3) (4)


25.

(1) (2) (3) (4)


26.

(1) (2) (3) (4)

Direction (Q.27 to Q.30) : Select a figure from the given four alternatives which fits exactly into Figure (X) to
form a complete square.

27. (1) (2) (3) (4)


(X)

28. (1) (2) (3) (4)

(X)

www.allenoverseas.com OVERSEAS 159


Class-VIII

29. (1) (2) (3) (4)


(X)

30. (1) (2) (3) (4)


(X)

Que. 1 2 3 4 5 6 7 8 9 10 11 12 13 14 15 16 17 18 19 20
Ans. 1 4 1 2 2 3 1 3 1 2 3 1 1 1 2 3 4 3 3 4
Que. 21 22 23 24 25 26 27 28 29 30
Ans. 2 1 3 2 3 1 3 3 1 3

160 OVERSEAS www.allenoverseas.com


Mental Ability

CHAPTER 20 MATHEMATICAL OPERATIONS

This section deals with questions on simple mathematical operations. Here, the four fundamental operations –
addition, subtraction, multiplication and division and also statements such as 'less than', 'greater than', 'equal to',
'not equal to', etc. are represented by symbols, different from the usual ones. The questions involving these
operations are set using artificial symbols. The candidate has to substitute the real signs and solve the questions
accordingly, to get the answer.

20.1 Problem solving by substitution


In this type, you are provided with substitutes for various mathematical symbols or numerals, followed by a
question involving calculation of an expression or choosing the correct/incorrect equation. The candidate is
required to put in the real signs or numerals in the given equation and then solve the questions as required.
Note : While solving a mathematical expression, proceed according to the rule BODMAS i.e., Brackets, Of,
Division, Multiplication, Addition, Subtraction.
Solved examples
Ex.1 If '+' means 'minus', ' × ' means 'divided by', ' ¸ ' means 'Plus' and '–' means 'multiplied by', then which of the
following will be the value of the expression 252 × 9 – 5 + 32 ¸ 92?
(1) 95 (2) 168 (3) 192 (4) 200
Sol. Putting the proper signs in the given expression, we get :
252 ¸ 9 × 5 – 32 + 92 = 28 × 5 – 32 + 92 = 140 – 32 + 92 = 232 – 32 = 200.
Hence, the answer is (4).
Ex.2 If L stands for +, M stands for –, N stands for × , P stands for ¸ , then 14 N 10 L 42 P 2 M 8 =?
(1) 153 (2) 216 (3) 248 (4) 251
Sol. Putting the proper signs in the given expression, we get :
14 × 10 + 42 ¸ 2 – 8 = 14 × 10 + 21 – 8 = 140 + 21 – 8 = 161 – 8 = 153.
Hence, the answer is (1).
Ex.3 If 20 – 10 means 200, 8 ¸ 4 means 12, 6 × 2 means 4 and 12 + 4 means 3 then 100 – 10 × 1000 ¸ 1000 + 100
× 10 =?
(1) 0 (2) 20 (3) 1090 (4) 1900
Sol. Given that : 20 – 10 = 200. But, actually 20 × 10 = 200. So, – means ×.
Given that : 8 ¸ 4 = 12. But, actually 8 + 4 = 12. So, ¸ means +.
Given that : 6 × 2 = 4. But, actually 6 – 2 = 4. So, × means –.
Given that : 12 + 4 = 3. But, actually 12 ¸ 4 = 3. So, + means ¸.
Thus, in the given mathematical language, – means × , ¸ means +, × means – & + means ¸ . Putting the
correct signs, we have :
Given expression = 100 × 10 – 1000 + 1000 ¸ 100 – 10 = 1000 – 1000 + 10 – 10 = 0.
Hence, the answer is (1).

www.allenoverseas.com OVERSEAS 161


Class-VIII

Ex.4 It being given that : > denotes +, < denotes –, + denotes ¸ , – denotes =, = denotes 'less than' and × denotes
'greater than ', find which of the following is a correct statement.
(1) 3+2>4=9+3<2 (2) 3 > 2 > 4 = 18 + 3 < 1
(3) 3>2<4× 8+4<2 (4) 3 + 2 < 4 × 9 + 3 < 3
Sol. Using proper notations, we have :
11
(1) Given statement is 3 ¸ 2 + 4 < 9 ¸ 3 – 2 or < 1, which is not true.
2
(2) Given statement is 3 + 2 + 4 < 18 ¸ 3 – 1 or 9 < 5, which is not true.
(3) Given statement is 3 + 2 – 4 > 8 ¸ 4 – 2 or 1 > 0, which is true.
5
(4) Given statement is 3 ¸ 2 – 4 > 9 ¸ 3 – 3 or – > 0, which is not true.
2
Hence, the answer is (3).

1. If '<' means 'minus', '>' means 'plus', '=' means 'multiplied by ' and '$' means 'divided by', then what would be the
value of 27 > 81 $ 9 < 6?
(1) 6 (2) 33 (3) 36 (4) None of these
2. If × means –, + means ¸ , – means × and ¸ means +, then 15 – 2 ¸ 900 + 90 × 100 =?
(1) 190 (2) 180 (3) 90 (4) None of these
3. If '+' means 'divided by', '–' means 'add', '×' means 'minus' and '/' means 'multiplied by', what will be the value of
the following expression?
[{(17 × 12) – (4/2)} + (23 – 6)]/0
(1) Infinite (2) 0 (3) 118 (4) 219
4. If '–' stands for 'division', '+' for 'multiplication', ' ¸ ' for 'subtraction' and '×' for 'addition', then which one of the
following equations is correct?
(1) 4 × 5 + 9 – 3 ¸ 4 = 15 (2) 4 × 5 × 9 + 3 ¸ 4 = 11
(3) 4 – 5 ¸ 9 × 3 – 4 = 17 (4) 4 ¸ 5 + 9 – 3 + 4 = 18
5. If '+' stands for 'division', ' ¸ ' stands for 'multiplication', '×' stands for 'subtraction' and '–' stands for 'addition', which
one of the following is correct?
(1) 18 ¸ 6 × 7 + 5 – 2 = 22 (2) 18 × 6 + 7 ¸ 5 – 2 = 16
(3) 18 ¸ 6 – 7 + 5 × 2 = 20 (4) 18 + 6 ¸ 7 × 5 – 2 = 18
6. If L denotes ¸ , M denotes ×, P denotes + and Q denotes –, then which of the following statements is true?
3 173
(1) 32 P 8 L 16 Q 4 = – (2) 6 M 18 Q 26 L 13 P 7 =
2 13
38
(3) 11 M 34 L 17 Q 8 L 3 = (4) 9 P 9 L 9 Q 9 M 9 = – 71
3

162 OVERSEAS www.allenoverseas.com


Mental Ability

Directions (Q.7 & Q.8) : In each of the following questions, some symbols are represented by letters as
shown below.

+ – × –·· = > <


B G E C D A F

Now, identify the correct expression in each case.


7. (1) 18 C 3 D 6 B 8 C 4 G 12 (2) 18 A 3 E 6 B 8 G 4 B 12
(3) 18 C 3 G 6 B 8 B 4 D 12 (4) 18 F 3 B 6 E 8 G 4 E 12
8. (1) 15 B 5 G 8 B 4 G 6 F 3 (2) 15 C 15 B 8 F 4 B 6 C 3
(3) 15 A 5 E 8 C 4 B 6 E 3 (4) 15 C 5 F 8 C 4 B 6 C 3
Directions (Q.9 to Q.11) : In each of the following questions, different alphabets stand for various symbols
as indicated below :
Addition : O Subtracton : M Multiplication : A
Division : Q Equal to : X Greater than : Y
Less than : Z
Out of the four alternatives given in these questions, only one is correct according to the above letter symbols.
Identify the correct answer.
9. (1) 2 Z 2 A 4 O 1 A 4 M 8 (2) 8 Y 2 A 3 A 4 Q 2 A 4
(3) 10 X 2 O 2 A 4 O 1 M 2 (4) 12 X 4 O 2 Q 1 A 4 A 2
10. (1) 1 O 1 Q 1 M 1 Y 3 Q 1 (2) 2 Q 1 O 10 A 1 Z 6 A 4
(3) 3 O 2 O 10 Q 2 X 10 A 2 (4) 5 Q 5 A 5 O 5 Y 5 A 2
11. (1) 3 O 2 X 2 Q 1 A 3 O 1 (2) 6 M 2 Y 10 Q 2 A 3 O 1
(3) 10 A 2 Z 2 Q 2 A 10 Q 2 (4) 10 A 2 Y 2 Q 1 A 10 Q 2
12. If ¸ means × , × means +, + means – and – means ¸ , 'find the value of 16 × 3 + 5 – 2 ¸ 4.
(1) 9 (2) 10 (3) 19 (4) None of these
13. If + means ¸ , ¸ means – , – means × , × means +, then 12 + 6 ¸ 3 – 2 x 8 =?
(1) –2 (2) 2 (3) 4 (4) 8
14. If P denotes ¸ , Q denotes × , R denotes + and S denotes – , – then what is the value of 18 Q 12 P 4 R 5 S 6?
(1) 53 (2) 59 (3) 63 (4) 65
15. If P means 'division', T means 'addition', M means 'subtraction' and D means 'multiplication', then what will be the
value of the expression 12 M 12 D 28 P 7 T 15?
(1) – 30 (2) – 15 (3) 15 (4) None of these
16. If P means × , R means +, T means ¸ and S means –, then 18 T 3 P 9 S 8 R 6 =?
1 2
(1) – 1 (2) (3) 46 (4) None of these
3 3

www.allenoverseas.com OVERSEAS 163


Class-VIII

17. If P denotes 'multiplied by', T denotes 'subtracted from', M denotes 'added to' and B denotes 'divided by', then
28 B 7 P 8 T 6 M 4 =?
3
(1) – (2) 30 (3) 32 (4) 34
2
18. If '×' stands for 'addition', '<' for 'subtraction', '+' for 'division', '>' for 'multiplication','–' for 'equal to' , ' ¸ ' for 'greater
than' and '=' for 'less than', then state which of the following is true?
(1) 3 × 4 > 2 – 9 + 3 < 3 (2) 5 × 3 < 7 ¸ 8 + 4 × 1
(3) 5 > 2 + 2 = 10 < 4 × 8 (4) 3 × 2 < 4 ¸ 16 > 2 + 4

Que. 1 2 3 4 5 6 7 8 9 10
Ans . 4 4 2 1 4 4 3 4 1 2
Que. 11 12 13 14 15 16 17 18
Ans . 4 1 3 1 4 4 2 3

164 OVERSEAS www.allenoverseas.com


Mental Ability

20.2 Interchange of signs and number


Solved examples
Ex.1 If the given interchanges namely : signs + and ¸ and numbers 2 and 4 are made in signs and numbers, which
one of the following four equations would be correct?
(1) 2 + 4 ¸ 3 = 3 (2) 4 + 2 ¸ 6 = 1.5 (3) 4 ¸ 2 + 3 = 4 (4) 2 + 4 ¸ 6 = 8
Sol. Interchanging (+ and ¸) and (2 and 4), we get :
(1) 4 ¸ 2 + 3 = 3 or 5 = 3, which is false. (2) 2 ¸ 4 + 6 = 1.5 or 6.5 = 1.5, which is false.
10
(3) 2 + 4 ¸ 3 = 4 or = 4, which is false. (4) 4 ¸ 2 + 6 = 8 or 8 = 8, which is true.
3
Hence, the answer is (4).
Ex.2 Which one of the four interchanges in signs and numbers would make the given equation correct?
3+5–2=4
(1) + and –, 2 and 3 (2) + and –, 2 and 5 (3) + and –, 3 and 5 (4) None of these
Sol. By making the interchanges given in (1), we get the equation as 2 – 5 + 3 = 4 or 0 = 4, which is false.
By making the interchanges given in (2), we get the equation as 3 – 2 + 5 = 4 or 6 = 4, which is false.
By making the interchanges given in (3), we get the equation as 5 – 3 + 2 = 4 or 4 = 4, which is true.
Hence, the answer is (3).

Directions (Q.1 to Q.4) : In each of the following questions, if the given interchanges are made in signs
and numbers, which one of the four equations would be correct?
1. Given interchanges : Signs – and ¸ and numbers 4 and 8.
(1) 6 – 8 ¸ 4 = – 1 (2) 8 – 6 ¸ 4 = 1 (3) 4 ¸ 8 – 2 = 6 (4) 4 – 8 ¸ 6 = 2
2. Given interchanges : Signs + and × and numbers 4 and 5.
(1) 5 × 4 + 20 = 40 (2) 5 × 4 + 20 = 85 (3) 5 × 4 + 20 = 104 (4) 5 × 4 + 20 = 95
3. Given interchanges : Signs + and – and numbers 4 and 8.
(1) 4 + 8 – 12 = 18 (2) 4 – 8 + 12 = 0 (3) 8 + 4 – 12 = 24 (4) 8 – 4 + 12 = 8
4. Given interchanges : Signs – and × and numbers 3 and 6.
(1) 6 – 3 × 2 = 9 (2) 3 – 6 × 8 = 10 (3) 6 × 3 – 4 = 15 (4) 3 × 6 – 4 = 33
5. By applying which of the following meanings of arithmetical signs, will the value of
1
700 – 10 ¸ × 35 + 70 be zero?
2
(1) × means ¸ , + means ×, ¸ means +, – means –
(2) × means ¸ , + means –, ¸ means ×, – means +
(3) × means +, + means –, ¸ means ×, – means ¸
(4) × means –, + means ¸ , ¸ means ×, – means +

www.allenoverseas.com OVERSEAS 165


Class-VIII

Directions (Q.6 to Q.11) : In each of the following questions, the given equation becomes correct due
to the interchange of two signs. One of the four alternatives under it specifies the interchange of signs in
the equation which when made will make the equation correct. Find the correct alternative.
6. 5 + 6 ¸ 3 – 12 × 2 = 17
(1) ¸ and × (2) + and × (3) + and ¸ (4) + and –
7. 2 × 3 + 6 – 12 ¸ 4 = 17
(1) × and + (2) + and – (3) + and ¸ (4) – and ¸
8. 16 – 8 ¸ 4 + 5 × 2 = 8
(1) ¸ and × (2) – and ¸ (3) ¸ and + (4) – and x
9. 9 + 5 ¸ 4 x 3 – 6 = 12
(1) + and × (2) ¸ and × (3) ¸ and – (4) + and –
10. 12 ¸ 2 – 6 × 3 + 8 = 16
(1) ¸ and + (2) – and + (3) × and + (4) ¸ and x
11. 10 + 10 ¸ 10 – 10 × 10 = 10
(1) + and – (2) + and ¸ (3) + and × (4) ¸ and +
Directions (Q.12 to Q.15) : In each of the following questions, which one of the four interchanges in signs
and numbers would make the given equation correct?
12. 6 × 4 + 2 = 16
(1) + and ×, 2 and 4 (2) + and ×, 2 and 6 (3) + and ×, 4 and 6 (4) None of these
13. (3 ¸ 4) + 2 = 2
(1) + and ¸ , 2 and 3 (2) + and ¸ , 2 and 4 (3) + and ¸ , 3 and 4 (4) No interchange, 3 and 4
14. 4 × 6 – 2 = 14
(1) × to ¸ , 2 and 4 (2) – to ¸ , 2 and 6 (3) – to +, 2 and 6 (4) × to +, 4 and 6
15. (6 ¸ 2) × 3 = 0
(1) ¸ and ×, 2 and 3 (2) × to –, 2 and 6 (3) ¸ and ×, 2 and 6 (4) × to –, 2 and 3

Que. 1 2 3 4 5 6 7 8 9 10
Ans . 3 3 2 2 3 1 1 2 3 2
Que. 11 12 13 14 15
Ans . 3 3 1 3 4

166 OVERSEAS www.allenoverseas.com


Mental Ability

CHAPTER 21 SPOTTING OUT THE EMBEDDED FIGURE

The problem figure in embedded figure is designed to measure disembedding as a restructuring skill, which
results from the use of style. In this test, students are required to discern simple geometric figures from more
complicated patterns. This test measures the general intelligence or some specific ability of the person.
Solved Examples
Directions : In each of the following questions, fig. (X) is embedded in any one of the four alternative figures (1),
(2), (3) and (4). Find the alternative which contains fig. (X) as its part.
Ex.1 Figure (X) is embedded in anyone of the four alternative figures. Choose the alternative which contains figure
(X).

(X)

(1) (2) (3) (4)

Sol.

Clearly figure (X) is embeded in alternative figure (2). The portion which figure (X) occupies in the alternative
figure has been shown in the adjacent figure. Hence, the correct answer figure is (2).
Ex.2 Choose the alternative which has figure (X) embedd in it.

(X)

(1) (2) (3) (4)

Sol.

Clearly figure (X) is embedded in alternative figure (1) and the portion it occupies in alternative figure has been
shown in the adjacent figure. Hence the correct answer figure is (1).

www.allenoverseas.com OVERSEAS 167


Class-VIII

Directions (Q.1 to Q.24) : In each of the following questions, you are given a fig. (X) followed by four
alternative figures (1), (2), (3) and (4) such that fig. (X) is embedded in one of them. Trace out the alternative
figure which contains fig. (X) as its part.

1.

(X)

(1) (2) (3) (4)

2.

(X)

(1) (2) (3) (4)

3.

(X)

(1) (2) (3) (4)

4.

(X)

(1) (2) (3) (4)

168 OVERSEAS www.allenoverseas.com


Mental Ability

5.

(X)

(1) (2) (3) (4)

6.

(X)

(1) (2) (3) (4)

7.

(X)

(1) (2) (3) (4)

8.

(X)

(1) (2) (3) (4)

9.

(X)

(1) (2) (3) (4)

www.allenoverseas.com OVERSEAS 169


Class-VIII

10.

(X)

(1) (2) (3) (4)

11.

(X)

(1) (2) (3) (4)

12.

(X)

(1) (2) (3) (4)

13.

(X)

(1) (2) (3) (4)

170 OVERSEAS www.allenoverseas.com


Mental Ability

14.

(X)

(1) (2) (3) (4)

15.

(1) (2) (3) (4)

16.

(X)

(1) (2) (3) (4)

17.

(X)

(1) (2) (3) (4)

www.allenoverseas.com OVERSEAS 171


Class-VIII

18.

(X)

(1) (2) (3) (4)

19.

(X)

(1) (2) (3) (4)

20.

(X)

(1) (2) (3) (4)

21.

(X)

(1) (2) (3) (4)

22.

(X)

(1) (2) (3) (4)

172 OVERSEAS www.allenoverseas.com


Mental Ability

23.

(X)

(1) (2) (3) (4)

24.

(X)

(1) (2) (3) (4)

25. Which figure is embedded in the pattern given in figure (X)?

(X)

(1) (2) (3) (4)

Que. 1 2 3 4 5 6 7 8 9 10 11 12 13 14 15 16 17 18 19 20
Ans. 3 3 2 2 1 1 3 4 2 4 1 4 1 1 2 4 1 2 2 4
Que. 21 22 23 24 25
Ans. 2 4 1 3 1

www.allenoverseas.com OVERSEAS 173


Class-VIII

Important Notes

174 OVERSEAS www.allenoverseas.com


Mental Ability

CHAPTER 22 DOT SITUATION


Questions on dot situation relate to a process in which we have to find a common characteristic between
a dot placed in between some figures and enclosed area by the same figures.

In these questions, one or more dot/s is/are placed somewhere in the area common to figures such as circle,
square, triangle and rectangle. The placement of dot shows a common relationship among these figures.
We have to understand this common relationship and search for the same in the answer figure.

Solved examples

Direction : In the following question a dot is placed in the figure marked as (A), this figure is followed by
four alternatives marked as (1), (2), (3) and (4). One out of these four options contains the common region
to circle, squares triangle similar to that of marked by dot in figure (A). Select that option.

Ex.

(A) (1) (2) (3) (4)

Sol. One dot occupies the region which is common to circle and triangle and other dot occupies the region which
is common to triangle and square. These two characteristics as shown by these two dots are found in figure
(1), it possesses region which is common to circle and triangle and also the region which is common in triangle
and square. Hence, the answer is (1).

common to Common to
circle and Triangle and
triangle Square

www.allenoverseas.com OVERSEAS 175


Class-VIII

Direction (Q.1 to Q.15) : In the following question a dot is placed in the figure marked as (X), this figure
is followed by four alternatives marked as (1), (2), (3) and (4). One out of these four options contains the
common region to circle, square, triangle and rectangle similar to that of marked by dot in figure (X). Select
that option.

1.

(X)

(1) (2) (3) (4)

2.

(X)

(1) (2) (3) (4)

3.

(X)

(1) (2) (3) (4)

176 OVERSEAS www.allenoverseas.com


Mental Ability

4.

(X)

(1) (2) (3) (4)

5.

(X)

(1) (2) (3) (4)

6.

(1) (2) (3) (4)

7.

(X)

(1) (2) (3) (4)

www.allenoverseas.com OVERSEAS 177


Class-VIII

8.

(X)

(1) (2) (3) (4)

9.

(X)

(1) (2) (3) (4)

10.

(X)

(1) (2) (3) (4)

11.

(X) (1) (2) (3) (4)

178 OVERSEAS www.allenoverseas.com


Mental Ability

12.
(X) (1) (2) (3) (4)

13.

(X) (1) (2) (3) (4)

14.

(X) (1) (2) (3) (4)

15.

(X) (1) (2) (3) (4)

Que 1 2 3 4 5 6 7 8 9 10 11 12 13 14 15
.
Ans 1 4 1 4 2 4 2 1 3 4 4 1 1 1 3
.

www.allenoverseas.com OVERSEAS 179


Class-VIII

Important Notes

180 OVERSEAS www.allenoverseas.com


Mental Ability

COMPLETION OF
CHAPTER 23 INCOMPLETE PATTERN

In this type of problems, a figure or a matrix, containing a set of figures following a particular sequence or
pattern is given, in which a part, generally a quarter, is left blank. This problem figure is followed by four alternative
figures. The candidate is required to select the one which best fits into the blank space of problem figure so
as to complete the original pattern.
Solved examples
Ex.1 Select a figure from amongst the four alternatives, which when placed in the blank space of fig. (X) would
complete the pattern.

?
(X) (1) (2) (3) (4)

Sol. Clearly, fig. (2) will complete the pattern when placed in the blank space of fig. (X) as shown below :

Hence, the answer is (2).


Ex.2 Complete the pattern in fig. (X) by selecting one of the figures from the four alternatives.

(X) (1) (2) (3) (4)


Sol. Clearly, fig. (1) will complete the pattern when placed in the blank space of fig. (X) as shown below :

Hence, the answer is (1).

www.allenoverseas.com OVERSEAS 181


Class-VIII

Directions (Q.1 to Q.20) : In each of the following questions, select a figure from amongst the four alternatives,
which when placed in the blank space of fig. (X) would complete the pattern.

1. ?
(X) (1) (2) (3) (4)

2. ?

(X) (1) (2) (3) (4)

3. ?

(X) (1) (2) (3) (4)

4.

(X) (1) (2) (3) (4)

5.
?

(X) (1) (2) (3) (4)

6.
?
(X) (1) (2) (3) (4)

182 OVERSEAS www.allenoverseas.com


Mental Ability

7.
?
(X) (1) (2) (3) (4)

8. ?

(X) (1) (2) (3) (4)

9.

(X) (1) (2) (3) (4)

10. ?
(X) (1) (2) (3) (4)

11. ?
(X) (1) (2) (3) (4)

12.
?
(X) (1) (2) (3) (4)

?
13.

(X) (1) (2) (3) (4)

www.allenoverseas.com OVERSEAS 183


Class-VIII

14. ?
(X) (1) (2) (3) (4)

?
15.

(X) (1) (2) (3) (4)

16. ?
(X) (1) (2) (3) (4)

17.

(X) (1) (2) (3) (4)

18. ?
(X) (1) (2) (3) (4)

19.

(?)

(X) (1) (2) (3) (4)

20.

?
(X) (1) (2) (3) (4)

Que. 1 2 3 4 5 6 7 8 9 10 11 12 13 14 15 16 17 18 19 20
A ns. 3 4 2 4 3 3 1 1 4 1 2 1 2 2 3 4 2 3 1 4

184 OVERSEAS www.allenoverseas.com


Mental Ability

CHAPTER 24 SITTING ARRANGEMENT

Questions of sitting arrangement are based on a set of information containing certain conditions. Students
are required to arrange the objects either in a row or in circle on the basis of such conditions. Information
given in the questions is presented in distorted form to create confusions and to test candidate's ability to
analyse the information step by step in order to answer the questions correctly. One may find the situation
such as five men standing in a line or six people sitting around a table in a particular order.
Solved examples
Ex.1. Read the following information carefully to answer the question given below :
Nine cricket fans are watching a match in a stadium. Seated in one row, they are J, K, L, M, N, O, P, Q and
R. L is at the right of M and at third place at the right of N, K is at one end of the row. Q is seated adjacent to
both O and P. O is at the third place at the left of K. J is right next to left of O.
1. Who is sitting at the centre of the row?
(1) I (2) J (3) O (4) Q
2. Who is at the other end of the row?
(1) J (2) N (3) P (4) R
3. Which of the following statements is true?
(1) N is two seats away from J. (2) M is at one extreme end.
(3) R and P are neighbours. (4) There is one person between L and O.
Sol. L is to the right of M and at third place at the right of N. So, the sequence becomes :
N£ML. O is at third place at the left of K and J is right next to left of O i.e. JO££K. Q is adjacent to both O
and P i.e. JOQPK Thus, the only blank space left must be occupied by R i.e. NRML.
Since K is at one end of the row, so the entire seating sequence becomes: NRMLJOQPK
1. J is sitting at the centre of the row. Hence, the answer is (2).
2. N is-at the other end of the row. Hence, the answer is (2).
3. Clearly, there is one person J between Land O.
So, only (4) is true. Hence, the answer is (4).
Ex.2 Six persons are sitting in a circle facing the center of the circle. Umesh is between Rajesh and Ravi, Mahesh
is between Sanjay and Ganesh. Sanjay is to the immediate left of Rajesh. Who is to the immediate right
of Rajesh?
(1) Umesh (2) Ganesh (3) Ravi (4) Sanjay'
Sol. On the basis of two information that Umesh is between Rajesh and Ravi and Mahesh is between Sanjay and
Ganesh we can determine the exact positions of persons sitting adjacent to Mahesh and Umesh. However,
the last information that Sanjay is to the immediate left of Rajesh fixes the position of all the six persons
as in the given figure below. On the basis of which we can determine that Umesh is sitting immediate right
of Rajesh. Hence, the answer is (1).

www.allenoverseas.com OVERSEAS 185


Class-VIII

Ex.3 Read the following statements and answer the question that follow:
(i) P and Q like football and hockey. (ii) P and S like hockey and badmintion.
(iii) Q and R like cricket and football. (iv) R and S like badminton and cricket.
1. Which player does not like cricket?
(1) P (2) Q (3) R (4) S
2. Who likes hockey, football and badmintion?
(1) P (2) Q (3) R (4) S
3. Who likes badminton, cricket and football?
(1) P (2) Q (3) R (4) S
4. Who likes hockey, cricket and football?
(1) P (2) Q (3) R (4) S
5. How many players like all the four games?
(1) P (2) Q (3) R (4) None
Sol. From the given information, we can make the following table:

From the above, we get


1. According to the above table, P does not like cricket. Hence, the answer is (1).
2. According to the above table, P likes hockey, football and badminton.Hence, the answer is (1).
3. According to the above table, R likes badminton, cricket and football. Hence, the answer is (3).
4. According to the above table, Q likes hockey, cricket and football. Hence, the answer is (2).
5. According to the above table, none of the players likes all the four games. Hence, the answer is (4).

Directions (Q.1 to Q.4) : Some friends are sitting on a bench, Sunil is sitting next to Sunita and Sanjay is
sitting next to Bindu. Bindu is not sitting with Sumit, Sumit on the left end of the bench and Sanjay is on second
position from right hand side. Sunil is on the right side of Sunita. Sunil and Sanjay are sitting together. Based on
the above sitting arrangements answer the following questions.
1. Who is sitting in the centre?
(1) Sumit (2) Sunil (3) Bindu (4) Sanjay
2. Sanjay is sitting between?
(1) Bindu and Sunita (2) Sunil and Sumit (3) Sunita and Bindu (4) Sunil and Bindu
3. Sumit is sitting on the?
(1) Second place from right (2) Second place from left
(3) Extreme left (4) Extreme right

186 OVERSEAS www.allenoverseas.com


Mental Ability

4. Bindu is sitting on the?


(1) Extreme left side (2) Extreme right side
(3) Second from left side (4) Third from left side
Directions (Q. 5 to Q.9) : Read the following information carefully and answer the questions given below it :
In a car exhibition, seven cars of seven different companies viz. Cardilace, Ambassador, Fiat Maruti, Mercedes,
Bedford and Fargo were displayed in a row, facing east such that :
(1) Cardilace car was to the immediate right of Fargo.
(2) Fargo was fourth to the right of Fiat.
(3) Maruti car was between Ambassador and Bedford.
(4) Fiat, which was third to the left of Ambassador car, was at one of the ends.
5. Which of the following was the correct position of the Mercedes?
(1) Immediate left of Cardilace (2) Immediate left of Bedford
(3) Between Bedford and Fargo (4) Fourth to the right of Maruti
6. Which of the following is definitely true?
(1) Fargo car is between Ambassador and Fiat. (2) Cardilace car is to the immediate left of Mercedes.
(3) Fargo is to the immediate right of Cardilac. (4) Maruti is fourth to the right of Mercedes.
7. Which cars are on the immediate either sides of the Cardilace car?
(1) Ambassador and Maruti (2) Maruti and Fait
(3) Fiat and Mercedes (4) None of these
8. Which of the following is definitely true?
(1) Maruti is to the immediate left of Ambassador. (2) Bedford is to the immediate left of Fiat.
(3) Bedford is at one of the ends. (4) Fiat is second to the right of Maruti.
9. Which of the following groups of cars is to the right of the Ambassador car?
(1) Cardilace, Fargo and Maruti (2) Maruti, Bedford and Fiat
(3) Mercedes, Cardilace and Fargo (4) Bedford, Cardilace and Fargo
Direction (Q. 10 & Q.11) : A, B, C and D are to be seated in a row. But C and D cannot be together. Also
B cannot be at the third place.
10. Which of the following must be false?
(1) A is at the first place (2) A is at the second place
(3) A is at third place (4) A is at the fourth place
11. If A is not at the third place, then C has which of the following option?
(1) The first place only (2) The third place only
(3) The first and third place only (4) Any of the places

www.allenoverseas.com OVERSEAS 187


Class-VIII

12. There are nine chairs in a row, each numbered 1 to 9 from left to right. Six friends are sitting on these
chairs. Megha, Sapna and Riya are neither sitting at chair 1 nor at chair numbered 9. Beena and Megha
does not have anybody sitting adjacent to them. There is only one empty chair between Megha and Riya.
Charu is adjacent to both Jiya and Riya. Sapna is sitting the seat numbered 2. On which chair number Megha
is sitting?

(1) 4 (2) 5 (3) 7 (4) 8

13. Six friends are sitting around a circular table at equal distance from each other. Ramola is sitting two places
right of Komolika who is equally opposite to Anu. Anu is sitting on the immediate left of Pallavi, who is exactly
opposite to Mandira. Natasha is also sitting at the table.

Which of the following statements is not correct?

(1) Natasha and Ramola are exactly opposite to each other

(2) Mandira and Natasha are at equal distance is from Komolika.

(3) Angle subtended by Mandira and Natasha is same as the angle subtended by Ramola and Pallavi at the
centre of the table.

(4) Natasha is on the immediate left of Pallavi

Direction (Q.14 to Q.17) : Study the following information to answer the given questions.

(i) Eight friends A, B, C, D, E, F, G and H are seated in a circle facing centre

(ii) D is between B and G and F is between A and H

(iii) E is second to the right of A.

14. Which of the following is A's position?

(1) left of F (2) Right of F

(3) Between E and F (4) can't be determined

15. Which of the following is C's position?

(1) Between E and A (2) Between G and E

(3) Second to the left of B (4) Can't be determined

16. Who are the neighbours of D?

(1) B and C (2) C and E

(3) B and G (4) B and G or B and H

17. If the positions of B and G and D and A are interchanged then who is sitting between B and G in new position.

(1) D (2) A (3) H (4) E

188 OVERSEAS www.allenoverseas.com


Mental Ability

Direction (Q. 18 to Q.21 ) : Study the following information carefully and answer the questions given below it
(i) Eleven students A, B, C, D, E, F, G, H, I, J, and K are sitting in a row of the class facing the teacher.
(ii) D, who is to the immediate left of F, is second to the right of C.
(iii) A, is second to the right of E, who is at one of the ends.
(iv) J is the immediate neighbour of A and B and third of the left of G.
(v) H is to the immediate left of D and third to the right of I.
18. Who is sitting in the middle of the row?
(1) C (2) I (3) B (4) G
19. Which of the following groups of friends is sitting to the right of G?
(1) IBJA (2) ICHDF (3) CHDF (4) CHDE
20. In the above sitting arrangement, which of the following statements is superfluous?
(1) (i) (2) (ii) (3) (iii) (4) None is superfluous
21. Which of the following statements is true in the context of the above sitting arrangements?
(1) There are three studends sitting between D and G.
(2) G and C are neighbour sitting to immediate right of H.
(3) B is sitting between J and I.
(4) K is sitting between A and J
22. Five cities A, B, C, D and E are famous for their lovely gardens, fancy jewellery, educational institutes, blue
pottery and scents but not in the same order and one city is famous for only one feature.
(i) A and C are neither educational centres nor have gardens.
(ii) B and E are not famous for jewellery or pottery.
(iii) Scents and jewellery have nothing to do with A.
(iv) D and E are not famous for gardens and jewellery.
(v) D is not famous for educational institutes.
City E is famous for which of the following features?
(1) Jewellery (2) Educational institutes (3) Scents (4) Gardens
23. Six people P, Q, R, S, T and U are sitting on the ground in a hexagonal shape. All the sides of the hexagon,
so formed, are of same length. P is not adjacent to Q or R; S is not adjacent to R or T; Q and R are adjacent;
U is in the middle of S and R. Who is at the same distance from S as T is from S?
(1) Q (2) R (3) U (4) Can't be determined
24. Persons X, Y, Z and Q live in red, green, yellow or blue coloured houses placed in sequence on a street.
Z lives in a yellow house. The green house is adjacent to the blue house. X does not live adjacent to Z.
The yellow house is in between the green and red houses. The colour of the house X lives in is ______.
(1) Blue (2) Green (3) Red (4) Data inadequate
25. P, Q, R, S, T, V and W are sitting around a circle facing the centre. R is third to the right of V, who is
second to the right of P. T is second to the left of Q, who is second to the left of W. V is sitting between
S and W. Who is sitting between T and V?
(1) W (2) R (3) S (4) Can't be determined

www.allenoverseas.com OVERSEAS 189


Class-VIII

26. Six friends – Mohit, Arun, Amit, Samrath, Aman and Siddharth are studying six different specialisations which
are – metallurgy, telecommunication, humanities, mechanical, electrical and electronics not necessarily in the
same order. Each one likes a different sport which are hockey, cricket, swimming, football, badminton and
tennis again not in the same order. Samrath is not studying electronics. Aman is studying humanities and
likes hockey. Amit likes swimming and is not studying electronics. The one who likes football is studying electrical.
Siddharth is studying mechanical and does not like tennis. The one who likes badminton is studying
telecommunication. Mohit and Arun do not like badminton. Mohit does not like tennis. Which sport does Mohit
like?
(1) Football (2) Cricket (3) Hockey (4) None of these
27. Five girls are sitting in a row facing south. Priya is not adjacent to Shalini or Niti. Priti is not adjacent to
Shalini and is at extreme right end. Priya is adjacent to Mahi. Mahi is at the middle in the row. Then, Priti
is adjacent to whom out of the following?
(1) Priya (2) Shalini (3) Niti (4) Mahi
28. A, B, C, D, E, F, G and H are sitting around a circle facing the centre. D is second to the left of H, who
is third to the left of A. B is fourth to the right of C, who is the immediate neighbour of H. G is not a neighbour
of B or C. F is not a neighbour of B. How many of them are sitting between C and B?
(1) Two (2) Four (3) Two or Four (4) Three
29. Read the following information and answer the following question:
1. Seven students P, Q, R, S, T, U and V take a series of tests.
2. No two students get the same marks.
3. V always scores more than P.
4. P always scores more than Q.
5. Each time either R scores the highest and T gets the least, or alternatively S scores the highest and
U or Q scores the least.
If R gets most, V should be ranked not lower than .
(1) Second (2) Third (3) Fourth (4) Fifth
30. Aakash, Priya, Shikha, Mohit, Riya, Giya and Mukta are sitting around a circle facing the centre. Aakash
is third to the left of Mukta and to the immediate right of Riya. Priya is second to the left of Giya, who
is not an immediate neighbour of Mohit. Who is to the immediate right of Priya?
(1) Mohit (2) Shikha (3) Mukta (4) Can't be determined

Que. 1 2 3 4 5 6 7 8 9 10 11 12 13 14 15 16 17 18 19 20
Ans. 2 4 3 2 4 2 4 1 3 1 3 3 4 2 1 3 2 2 3 4
Que. 21 22 23 24 25 26 27 28 29 30
Ans. 3 2 2 1 3 1 1 4 3 3

190 OVERSEAS www.allenoverseas.com

You might also like